73
S.S.M.ROMÂNIA - Filiala Mehedinți 2014 1 REVISTA MEHEDINȚEANĂ DE MATEMATICĂ NR. 14 SOCIETATEA DE ȘTIINȚE MATEMATICE DIN ROMÂNIA Filiala Mehedinți REVISTA DE MATEMATICĂ MEHEDINȚEANĂ R.M.M. Nr.14-2014

RMM 14.pdf

Embed Size (px)

Citation preview

Page 1: RMM 14.pdf

S.S.M.ROMÂNIA - Filiala Mehedinți 2014

1 REVISTA MEHEDINȚEANĂ DE MATEMATICĂ NR. 14

SOCIETATEA DE ȘTIINȚE MATEMATICE DIN ROMÂNIA

Filiala Mehedinți

REVISTA DE MATEMATICĂ MEHEDINȚEANĂ

R.M.M.

Nr.14-2014

Page 2: RMM 14.pdf

S.S.M.ROMÂNIA - Filiala Mehedinți 2014

2 REVISTA MEHEDINȚEANĂ DE MATEMATICĂ NR. 14

SOCIETATEA DE ȘTIINȚE MATEMATICE DIN ROMÂNIA

Filiala Mehedinți

COMITETUL DIRECTOR- 2010–2014

Prof.Dr. Căiniceanu Gheorghe PREȘEDINTE Prof. Dr. Prajea Manuela VICEPREȘEDINTE Prof. Dr. Stretcu Daniel SECRETAR

Prof. Grecu Vasile CASIER Prof. Nănuţi Dan MEMBRU

Prof. Nedeianu Dan MEMBRU Prof.Săceanu Victor MEMBRU

Prof.Ungureanu Mihai Octavian MEMBRU Prof.Gimoiu Iuliana CENZOR

Prof.Giugiuc Constantin CENZOR Expert contabil Borugă Ion CENZOR

COMITETUL DIRECTOR- 2015–2020

Prof.Daniel Sitaru PREȘEDINTE Prof.Dr. Căiniceanu Gheorghe VICEPREȘEDINTE

Prof. Dan Nedeianu VICEPREȘEDINTE Prof. Emilia Răducan SECRETAR

Prof. Grecu Vasile CASIER Prof. Leonard Giugiuc MEMBRU Prof. Claudia Nănuți MEMBRU Prof. Ovidiu Ticuși MEMBRU Prof. Dan Nănuți MEMBRU

Prof. Iuliana Gimoiu CENZOR Prof.Ionela Răducan CENZOR

Expert contabil Ion Borugă CENZOR

Page 3: RMM 14.pdf

S.S.M.ROMÂNIA - Filiala Mehedinți 2014

3 REVISTA MEHEDINȚEANĂ DE MATEMATICĂ NR. 14

MEMBRII S.S.M.R.-Filiala Mehedinți:

Nr.crt. Numele și prenumele Nr.crt. Numele și prenumele 1 Prof. Drinceanu Anda 45 Prof. Răducan Ionela 2 Prof. Filip Ștefania 46 Prof. Achimescu Florian 3 Prof. Preşneanu Doru 47 Prof. Andrei Ionică 4 Prof. Dăniasă Cora Ionela 48 Prof. Bălășoiu Daniela 5 Prof. Chirfot Carmen 49 Prof. Bondoc Lucian 6 Prof.Dr. Căiniceanu Gheorghe 50 Prof. Cernovici Ileana 7 Prof. Nedeianu Dan 51 Prof. Chiriţă Mircea 8 Prof. Ticuşi Ovidiu 52 Prof. Cioarec Radu 9 Prof. Giugiuc Leonard 53 Prof. Coadă Carmen

10 Prof. Paponiu Dana 54 Prof. Constantin Magdalena 11 Prof. Pașov Nicoleta 55 Prof. Diaconescu Roxanda 12 Prof. Sitaru Daniel 56 Prof. Drulă Ileana 13 Prof. Şuţă Marian 57 Prof. Farago Alexandru 14 Prof. Nănuţi Dan 58 Prof. Fluerașu Anghel 15 Prof. Pătuleanu Magdalena 59 Prof. Fluerașu Gabriela 16 Prof. Dr.Stretcu Daniel 60 Prof. Fritea Eugen 17 Prof. Căpraru Dorel 61 Prof. Giugiuc Constantin 18 Prof. Teiș Alina 62 Prof. Gorun Sanda 19 Prof. Dr. Tomiţă Vasile 63 Prof. Hinoveanu Sorin 20 Prof. Dr. Lupu Adrian 64 Prof. Ionică Constantin 21 Prof. Bălu Nicoleta 65 Prof. Irimia Constantin 22 Prof.Bondoc Gabriela 66 Prof. Jugănaru Ion 23 Prof. Răducan A. Emilia 67 Prof. Mălineanu Gabriela 24 Prof. Draga Tătucu Mariana 68 Prof. Marica Ștefan 25 Prof. Gimoiu Iuliana 69 Prof. Moclea Adriana 26 Prof. Șimandan Gileta 70 Prof. Osăin Victoria 27 Prof. Grecu Vasile 71 Prof. Pandioniu Aristia 28 Prof. Nițoiu Angela 72 Prof. Piț-Rada Marica 29 Prof. Dr. Prajea Manuela 73 Prof.Piț-Rada Ionel-Vasile 30 Prof. Ungureanu Mihai Octavian 74 Prof. Popescu Eleodor 31 Prof. Bejenaru Laviniu 75 Prof. Popescu Marcel 32 Prof. Vasilcanu Florentina 76 Prof. Popescu Virgil 33 Prof.Ciucă Ionel 77 Prof. Pupăză Ecaterina 34 Prof. Cristel Ecaterina 78 Prof. Rîmniceanu Elena 35 Prof. Bădescu Emilia 79 Prof. Semen Valentin 36 Prof. Dr. Grecu Luminița 80 Prof. Șerbănescu Ileana 37 Prof. Lădaru Daniela 81 Prof. Simionescu Loredana 38 Prof. Nănuţi Claudia 82 Prof. Stancu Oana 39 Prof. Ionescu Adela 83 Prof. Tomoescu Loredana 40 Prof. Chilea Ion 84 Prof. Tudose Elena 41 Prof. Dan Daniel 85 Prof. Tudosie Virginia 42 Prof. Grecu Adela 86 Prof. Untaru Ilie 43 Prof. Antonie Rodica 87 Prof. Ușurelu Ionela 44 Prof. Mariș Marinela 88 Prof. Pop Veronica

Page 4: RMM 14.pdf

S.S.M.ROMÂNIA - Filiala Mehedinți 2014

4 REVISTA MEHEDINȚEANĂ DE MATEMATICĂ NR. 14

AUTORII MATERIALELOR DIN R.M.M.-14

Nr.crt. Numele și prenumele Nr.crt. Numele și prenumele 1 Prof. Bondoc Gabriela 15 Prof. Piț-Rada Marica 2 Prof. Dr. Căiniceanu Gheorghe 16 Prof.Piț-Rada Ionel-Vasile 3 Prof. Chirfot Carmen 17 Prof. Preșneanu Doru 4 Prof. Dăniasă Cora Ionela 18 Prof. Pupăză Ecaterina 5 Prof. Draga Tătucu Mariana 19 Prof. Rîmnicianu Elena 6 Prof. Giugiuc Leonard 20 Prof. Șimandan Gileta 7 Prof. Grecu Adela 21 Prof. Dr. Stretcu Daniel 8 Prof. Grecu Vasile 22 Prof. Sitaru Daniel 9 Prof. Heuberger Dana 23 Prof. Ticuși Ovidiu

10 Prof. Ionică Constantin 24 Elev Bușe Iasmina 11 Prof. Nănuți Claudia 25 Elev Ceaușene Patricia 12 Prof. Nedeianu Dan 26 Elev Grecu Bogdan 13 Prof. Nițoiu Angela 14 Prof. Palașcă Vladimira

COLECTIVUL DE REDACȚIE

Nr.crt. Numele și prenumele Nr.crt. Numele și prenumele 1 Prof. Dr. Căiniceanu Gheorghe 11 Prof.Octavian Mihai Ungureanu 2 Prof. Dan Daniel 12 Prof. Popescu Eleodor 3 Prof. Sitaru Daniel 13 Prof. Săceanu Victor 4 Prof. Paponiu Dana 14 Prof. Grecu Vasile 5 Prof. Răducan Emilia 15 Prof. Dr. Prajea Manuela 6 Prof. Nănuți Claudia 16 Prof. Nănuți Dan 7 Prof. Rîmniceanu Elena 17 Prof. Nedeianu Dan 8 Prof. Draga Tătucu Mariana 18 Prof. Dr. Stretcu Daniel 9 Prof. Antonie Rodica 19 Prof. Dr. Lupu Adrian

10 Prof. Giugiuc Leonard 20 Prof. Ionică Constantin

Page 5: RMM 14.pdf

S.S.M.ROMÂNIA - Filiala Mehedinți 2014

5 REVISTA MEHEDINȚEANĂ DE MATEMATICĂ NR. 14

CUPRINS Cuvânt înainte din partea colectivului redacţional ............................................................. 6

În sprijinul participanților la concursurile școlare.................................................................... 7 Foşti elevi mehedinţeni ......................................................................................................... 8 Note matematice - O aplicație a Teoremei lui Milnor .......................................................... 10 Teme pentru grupele de performanţă................................................................................ 11 Clasa a V-a ......................................................................................................................... 11 Probleme de mișcare ............................................................................................................. 11 Clasa a VI-a ........................................................................................................................ 18 Divizorii unui număr natural ................................................................................................. 18 Principiul „cutiei” ................................................................................................................. 19 Clasa a IX-a ........................................................................................................................ 21 Generalizarea unei probleme de Olimpiadă ........................................................................... 21 Relaţii de recurenţă generate de bisectoarea unui unghi ........................................................ 23 Clasa a X-a .......................................................................................................................... 25 Convexitate Jensen ............................................................................................................... 25 Asupra numărului de permutări fără puncte fixe .................................................................. 29 Clasa a XI-a ........................................................................................................................ 31 Calculul limitelor unei clase de șiruri .................................................................................... 31 Puncte laticiale în plan și spațiu ............................................................................................ 33 Clasa a XII-a ....................................................................................................................... 35 Teoremele lui Lagrange și Cauchy în algebra superioară ...................................................... 35 Transport de structură .......................................................................................................... 42 Probleme propuse .............................................................................................................. 44 Ciclul primar ...................................................................................................................... 44 Clasa a V-a .......................................................................................................................... 46 Clasa a VI-a ........................................................................................................................ 47 Clasa a VII-a ....................................................................................................................... 49 Clasa a VIII-a ..................................................................................................................... 51 Clasa a IX-a ........................................................................................................................ 53 Clasa a X-a .......................................................................................................................... 55 Clasa a XI-a ........................................................................................................................ 57 Clasa a XII-a ....................................................................................................................... 59 Subiecte Olimpiadă ............................................................................................................ 62 Premianții concursurilor din 2013-2014 ............................................................................ 69

Page 6: RMM 14.pdf

S.S.M.ROMÂNIA - Filiala Mehedinți 2014

6 REVISTA MEHEDINȚEANĂ DE MATEMATICĂ NR. 14

Cuvânt înainte…

Și în această toamnă grupul de profesori entuziaști care formează colectivul redacțional al revistei de Matematică a Mehedințiului vă propune dragi cititori un nou număr al revistei noastre care a ajuns la 14. Ca de fiecare dată, simțim nevoia să precizăm că revista noastră se dorește un fel de completare de tip local a Gazetei Matematice, publicația de prestigiu a Societății de Știinte Matematice din România. Filiala Mehedinți a SSMR vă propune în paginile acestei reviste: situația membrilor cu cotizația la zi la 31.12.2013, teme pentru cercuri de elevi, probleme propuse pentru clasele 4-12, listele de onoare cu numele elevilor care s-au remarcat în cadrul concursurilor de matematică, subiectele olimpiadei municipale din 2014, clasele 5-12. În sprijinul viitorilor colaboratori revista noastră vă pune la dispoziție o listă cu temele pentru grupele de performanță ce nu au apărut încă în primele 13 numere. Este vorba despre programa suplimentară a OJM si ONM . Cu ocazia pensionării dorim si din paginile acestei reviste să aducem mulțumiri colegilor Antonie Rodica, Saceanu Victor, Băloi Valeria colaboratori activi ai Filialei și ai R.M.M. Totodată le adresăm rugămintea de a ne fi alături în continuare și le dorim sănătate și pensie lungă. Și în acest an cu prilejul „Zilei copilului” Filiala Mehedinți a SSMR a acordat premii elevilor care au obținut rezultate bune la Olimpiada Națională de matematică: Lungu Vlad, Crăciunescu Emanuel, Marghescu Bogdan la Concursul Național de Matematică Aplicată “Adolf Haimovici”: Dorobanțu Adina, Pîlșu Georgiana precum și la Olimpiada Națională de Lingvistică: Bușe Iasmina, Grecu Bogdan, Raicu Iulia (concurs interdisciplinar de limba română și matematică). Dorim tuturor cititorilor noștri un an plin de satisfacții.

Președinte Filiala Mehedinți a SSMR Profesor Dr. Gheorghe Căiniceanu

Page 7: RMM 14.pdf

S.S.M.ROMÂNIA - Filiala Mehedinți 2014

7 REVISTA MEHEDINȚEANĂ DE MATEMATICĂ NR. 14

În sprijinul participanților la concursurile școlare Prof. Nedeianu Dan , C.N. ,,Gh.Țițeica”

Prin efortul remarcabil al profesorilor și elevilor colaboratori ai revistei R.M.M, în toate cele 13 reviste anterioare, au fost publicate diverse articole de specialitate, apărute în deja tradiționala rubrică ,,Teme pentru grupele de performanță”.

Întrucât se apropie perioada olimpiadelor școlare, prezentăm în cele ce urmează o sinteză a titlurilor acestor teme, publicate în numerele anterioare (nr.9-2009, nr.10-2010, nr.11-2011, nr.12-2012, nr.13-2013). Lista de mai jos se dorește a fi o continuare a unei prezentări similare apărută în R.M.M. nr.9-2009, ce cuprinde titluri de articole publicate în primele opt numere ale revistei R.M.M (2002-2009).

Pentru fiecare temă publicată anterior, încadrată pe nivel corespunzător, se face referire (trimitere) la numărul revistei, de unde se poate studia, aprofunda tema respectivă:

CLASA a-V-a *Compararea puterilor *Cuburi perfecte *Probleme de perspicacitate *Probleme de mişcare *Metoda reducerii la absurd *Partiții

CLASA a-VI-a *Proprietăţi ale relaţiei de divizibilitate în N, Z (cele din programa specifică olimpiadei) *Numere raţionale (pozitive şi negative) -periodicitate, operaţii, ecuaţii *Probabilităţi *Linia mijlocie în triunghi (teorema directă şi teorema reciprocă)

CLASA a-VII-a *Proprietăţi ale modulului unui număr real *Calcul algebric: formule pentru 푎 ± 푏 ,푛 par sau impar, identitatea Lagrange,(푎 + 푏) = 푀푎 + 푏 *Teorema medianei *Probleme de maxim şi de minim (algebră) *Teorema sinusurilor *Teoremele lui Ceva, Menelaus (cu reciproce) *Formule pentru aria triunghiului, aplicaţii *Patrulater ortodiagonal *Patrulatere inscriptibile (condiţii necesare şi suficiente) *Dreapta Simson, cercul Euler *Probleme elementare de loc geometric

CLASA a-VIII-a *Patrulater ortodiagonal *Patrulatere inscriptibile (condiţii necesare şi suficiente) *Arc capabil de unghi dat *Poziţiile relative a două cercuri *Inegalităţi geometrice (inegalitatea triunghiului, oblica în relaţie cu perpendiculara, etc.) *Construcţii simple cu rigla gradată şi compasul *Probleme elementare de loc geometric *Perpendiculara comună a două drepte *Teorema lui Menelaus, teorema lui Thales în spaţiu *Corpuri care admit axe de simetrie, simetria faţă de un plan

CLASA a-IX-a *Densitatea în ℝ a mulțimilor ℚ și ℝ−ℚ, teorema lui Kronecker *Funcţii injective, surjective, bijective *Teoremele Euler, Fermat, Wilson *Indicatorul lui Euler *Teorema împărţirii cu rest in ℤ *Principiul includerii şi excluderii *Recurenţe liniare de ordinul I şi II, recurenţe omografice *Congruenţe modulo 푛 *Relaţii metrice *Teoreme clasice (Van Aubel, Stewart, Steiner), dreapta Euler, dreapta Simson, etc. *Teoreme de concurenţă şi coliniaritate *Puncte şi linii importante

CLASA a-X-a *Convexitate în sensul lui Jensen, inegalități. *Polinoame (c.m.m.d.c., c.m.m.m.c., algoritmul lui Euclid, teorema fundamentală a algebrei, teorema lui Bezout, rădacini multiple, polinomul Taylor, polinoame ireductibile, polinom minimal, relaţiile lui Viete).

CLASA a-XI-a *Descompunerea unei permutări în produs de cicli disjuncţi, respectiv transpoziţii *Determinanţi de ordinul 푛 *Rangul unei matrice, inegalitatea lui Sylvester *Mulţimi dense în ℝ, lema Cantor *Puncte limită pentru şiruri, limită inferioară, limi superioară *Mulţimi numărabile, mulţimi nenumărabile *Formula lui Taylor cu restul Lagrange *Lema Cesaro-Stolz;Criteriul Cauchy-d'Alembert *Discontinuități de prima și a doua speță , funcții Darboux *Teorema lui Cauchy

CLASA a-XII-a *Grupuri finite, teorema lui Cauchy *Grupuri finit generate *Orice corp finit este comutativ *Sume Darboux, sume Riemann, integrabilitate *Mulţimi neglijabile Lebesgue, criteriul lui Lebesgue

Page 8: RMM 14.pdf

S.S.M.ROMÂNIA - Filiala Mehedinți 2014

8 REVISTA MEHEDINȚEANĂ DE MATEMATICĂ NR. 14

ȘERBAN COSTEA-prezentare de Prof. Dr. Gheorghe Căiniceanu Numele Costea are o legătură lungă cu Colegiul Național Traian. „Bătrânul” Costea a

fost profesor al CNT, fiul său Nicu Costea a fost o perioadă profesor de fizică al Liceului Traian și apoi profesor de fizică la Liceul Domnul Tudor, fiind mulți ani director al acestui liceu. Soția lui Nicu Costea, Dorina Costea a funcționat ca să zic așa o viață ca profesor de limba franceză în CNT. Ultimul Costea care a pășit pe coridoarele CNT a fost Șerban Costea, în anii 1989-1993. Participant la 5 editii ale ONM în perioada 1987-1993, a obținut de mai multe ori premiul I pe județ și premii la Concursul interjudețean „Gheorghe Țițeica”. În liceu am avut plăcerea să-i fiu profesor. Profunzimea în raționament, ușurința cu care făcea calcule mintale, înclinația spre însușirea limbilor străine, sunt câteva din lucrurile care l-au făcut „vedetă” în școala. Pasionat de matematică, a urmat o carieră în domeniu și arătăm în continuare CV- ul său așa cum ni l-a trimis.

Page 9: RMM 14.pdf

S.S.M.ROMÂNIA - Filiala Mehedinți 2014

9 REVISTA MEHEDINȚEANĂ DE MATEMATICĂ NR. 14

Page 10: RMM 14.pdf

S.S.M.ROMÂNIA - Filiala Mehedinți 2014

10 REVISTA MEHEDINȚEANĂ DE MATEMATICĂ NR. 14

O aplicație a Teoremei lui Milnor Prof. Claudia Nănuţi, Colegiul Naţional Economic „Theodor Costescu”

TEOREMĂ: Pentru orice 푛 ∈ 푁 notăm 푆 = {푥 ∈ 푅 |‖푥‖ = 1}. O funcție continuă 푣: 푆 → 푅 se numește câmp de vectori tangenți la sfera 푆 dacă 푥 ∙ 푣(푥) = 0, pentru orice 푥 ∈ 푆 . Fie 푛 ∈ 푁. Dacă 푛 este par atunci câmpul de vectori tangenți la sfera 푆 se anulează în cel puțin un punct. APLICAȚIE: Să se arate că în orice moment la suprafața Pământului există un punct unde nu bate vântul. (Un astfel de punct se numește „ochi de ciclon”). Demonstrație: Identificăm suprafața Pământului cu sfera 푆 = {푥 ∈ 푅 |‖푥‖ = 1} și definim pe 푆 câmpul de vectori 푣 dat de viteza vântului la suprafața Pamântului la un moment dat. Aplicația 푣: 푆 →푅 este continuă. Vom demonstra că există 푥 ∈ 푆 încât 푣(푥 ) = 0. Fie 퐴 = 푥 ∈ 푅 ≤ ‖푥‖ ≤ și 퐷 = 푅 /{0}. În ipoteza că 푣 este de clasa 퐶 presupunem prin absurd că există un câmp de vectori tangenți la 푆 cu proprietatea că 푣 este continuu diferențiabil și nu se anulează pe 푆 . Înlocuim în 푣 pe 푥 cu ( )

‖ ‖ (normăm) și în acest caz ‖푣(푥)‖ = 1, pentru ‖푥‖ = 1.

Deoarece 푣 este de clasa 퐶 rezultă că 푐 = 푠푢푝 ∈ ‖푣 (푥)‖ < +∞. Fie 푤:퐷 → 푅 ;푤(푥) = ‖푥‖ ∙ 푣

‖ ‖;푥 ∈ 퐷.

Pentru orice 푡 ∈ 푅 ; |푡| < 푎 = 푚푖푛 , definim 푓 :퐷 → 푅 ; 푓 (푥) = 푥 + 푡푤(푥), pentru 푥 ∈ 퐷. Pentru orice 푟 > 0, notăm 푆(0, 푟) = {푟푥|푥 ∈ 푆 }. Demonstrăm că pentru |푡| < 푎 și 푟 > 0, avem: 푓 (푆(0, 푟)) = 푆(0, 푟√1 + 푡 ) (1)

푓 (푟푥) = 푟푥 + 푡푤(푟푥) = 푟푥 + 푡‖푟푥‖푣푟푥‖푟푥‖ = 푟푥 + 푡푟‖푥‖푣

푟푥‖푟푥‖ =

= 푟푥 + 푡푟푤(푥) = 푟푓 (푥). Pentru a demonstra (1) este suficient să arătăm că: 푓 (푆 ) = 푆(0,√1 + 푡 ) (2) Dacă 푥 ∈ 퐷 rezultă: ‖푓 (푥)‖ = ‖푥 + 푡푤(푥)‖ = ‖푥‖ + 2푡푥푤(푥) + 푡 ‖푤(푥)‖ = = ‖푥‖ + 2푡 ∙ 0 + 푡 ‖푥‖ = (1 + 푡 ) ∙ ‖푥‖ ‖푓 (푥)‖ = √1 + 푡 ∙ ‖푥‖ ⟹ 푓 (푆 ) ⊆ 푆 0,푟√1 + 푡 (3) Invers: Fie ∈ 푆 . Definim ℎ :퐴 → 푅 ; ℎ (푥) = 푦 − 푡푤(푥); 푥 ∈ 퐴. ‖ℎ (푥)‖ = ‖푦 − 푡푤(푥)‖. Deducem: 1 − |푡| ∙ ‖푥‖ ≤ ‖ℎ (푥)‖ ≤ 1 + |푡| ∙ ‖푥‖

1 −32

|푡| ≤ ‖ℎ (푥)‖ ≤ 1 +32

|푡|

Din |푡| < ⇒ ≤ ‖ℎ (푥)‖ ≤ ; 푥 ∈ 퐴, de unde ℎ (퐴) ⊆ 퐴. Dacă |푡| < 푎 ⟹ |푡| ∙ 푐 < 1. ‖ℎ (푥)− ℎ (푦)‖ ≤ |푡| ∙ 푐 ∙ ‖푥 − 푦‖ ; 푥,푦 ∈ 퐴. Rezultă că ℎ este o contracție și atunci există un unic 푥 ∈ 퐴 ; ℎ (푥 ) = 푥 .

Page 11: RMM 14.pdf

S.S.M.ROMÂNIA - Filiala Mehedinți 2014

11 REVISTA MEHEDINȚEANĂ DE MATEMATICĂ NR. 14

‖ℎ (푥 )‖ = ‖푦 − 푡푤(푥 )‖ ⇒ ‖푥 ‖ = 푦 − 푡 ∙ ‖푥 ‖푣푥‖푥 ‖

푓 (푥 ) = 푥 + 푡푤(푥 ) = 푦 ; 푓 √1 + 푡 푥 = √1 + 푡 ∙ 푦. Din (3) rezultă că: ‖푓 (푥)‖ = √1 + 푡 ∙ ‖푥‖⟹ ‖푓 (푥 )‖ = √1 + 푡 ∙ ‖푥 ‖ ⟹ ⟹ ‖푦‖ = √1 + 푡 ‖푥 ‖ ⟹ 푓 (푆 ) ⊇ 푆 0,√1 + 푡 (4) Din (3), (4) ⟹푓 (푆 ) = 푆 0,√1 + 푡 . Fie 퐵 = 푥 ∈ 푅 √1 + 푡 ≤ ‖푥‖ ≤ √1 + 푡 ; 푡 ∈ 푅 . Din (1) deducem că |푥| < 푎 ⟹ 푓 (퐴) = 퐵 , de unde

1 + 푡 휇(퐵 ) = 휇(푓 (퐴)) Am notat cu 휇 măsura Lebesgue în 푅 . Admitem cunoscut următorul rezultat: Dacă 퐴 este o mulțime compactă din 푅 cu frontiera de măsură nulă, 푣:퐴 → 푅 o funcție de clasă 퐶 și 푐 = 푠푢푝 ∈ ‖푣 (푥)‖ și dacă 푡 ∈ 푅,푓 :퐴 ⟶ 푅 , 푓 (푥) = 푥 + 푡푣(푥) atunci funcția 푡 ⟶ 휇푓 ((퐴)) este o funcție polinomială într-o vecinătate a originii. Datorită faptului că funcția 푡 ⟶ 휇푓 ((퐴)) , √1 + 푡 휇(퐵 ) = 휇(푓 (퐴)) nu este polinomială rezultă că presupunerea este falsă. Prin urmare există 푥 ∈ 푆 încât 푣(푥 ) = 0. Punctul 푥 se numește „ochi de ciclon”. Bibliografie: 1. Daniel Sitaru, Claudia Nănuți - Probleme de concurs – Editura Ecko-Print - Drobeta Turnu Severin - 2011 2. Daniel Sitaru, Claudia Nănuți - Matematici pentru olimpiade – Editura Ecko - Print-Drobeta Turnu Severin - 2014 3. Sorin Rădulescu, Marius Rădulescu - Teoreme și probleme de analiză matematică – EDP - București

Temă pentru grupa de performanță la clasa a V-a

PROBLEME DE MIŞCARE Notă metodică pentru Cercul de Matematică - gimnaziu

Prof. Ecaterina Pupăză, Colegiul Național „Traian” O adevarată „piatră de încercare” în concursurile de matematică o reprezintă problemele de mişcare. Acestea îi ajută pe elevi să îşi dezvolte creativitatea, modalităţile de rezolvare, fiind variate (metode aritmetice sau abordări algebrice). Deși apar începând cu ciclul primar, continuă cu cel gimnazial si liceal, au enunțuri familiare, cu autoturisme, trenuri, autobuze, biciclete, ambarcațiuni ş.a. care se deplasează în diverse direcţii, dar oricum, în linie dreaptă, cu viteze constante sau cu o anumită viteză medie, într-un timp dat sau ce trebuie aflat, problemele din această categorie aduc totdeauna provocarea gradului lor de dificultate .Prezenta Notă metodică se adresează elevilor de gimnaziu care

Page 12: RMM 14.pdf

S.S.M.ROMÂNIA - Filiala Mehedinți 2014

12 REVISTA MEHEDINȚEANĂ DE MATEMATICĂ NR. 14

indrăgesc matematica ,profesorilor de matematică și îşi propune o sistematizare a diferitelor tipuri de probleme legate de mişcare, însoțite de probleme propuse la concursurile şcolare. De regulă, problemele de mișcare vizează,în gimnaziu, mişcarea rectilinie uniformă a unui mobil, caracterizată prin cei 3 parametri(distanţă, viteză, timp) ce variază după formula: 푑 = 푣푡. Problemele de mişcare pot fi clasificate în 3 categorii, eventual combinate: 1 - determinări de parametri specifici - distantă, viteză, timp (în care se dau valori pentru două dintre ele şi se cere cea de-a treia); 2 - probleme de „urmărire” (mobile care se deplasează în acelaşi sens); 3 - probleme de „întâlnire” (mobile care se deplasează în sens contrar); 1. Probleme cu determinări de parametri specifici – distanță, viteză, timp Dată fiind relaţia care leagă cele 3 mărimi (푑 = 푣푡), sunt posibile 3 tipuri de probleme simple, însoțite sau nu de transformări în unități de măsură standard: se cunosc 푣 şi 푡 şi se cere 푑(푑 = 푣푡) sau 푑 şi 푡 şi se cere 푣(푣 = 푑 푡⁄ ), sau 푑 şi 푣 şi se cere 푡(푡 = 푑 푡⁄ ).

Exemple: 1. Un biciclist străbate cu viteza constantă o distanţă de 9 km în 30 min. Cu ce viteză s-a deplasat? 2. Printr-o localitate , in acelaşi moment, trec: un automobil cu viteza constantă de 12 m/s, o motocicletă cu viteza constantă de 72 km/h şi un autobuz cu viteza constantă de 900 m/min. Care dintre cele trei vehicule va ajunge primul în următoarea localitate? 3. În cât timp va ajunge la destinaţie o maşină care se deplasează rectiliniu cu viteza constantă de 30 m/s pe un traseu de 102 km? În general, problemele din aceasta categorie pot fi reductibile la una dintre cele 3 scheme de mai sus, doar după succesiuni de raţionamente . Ex:1 Un motociclist pleacă de la cabana unde inoptase spre un canton silvic, pe un drum forestier, mergând cu aceeaşi viteză. După două ore de mers, constata ca nu ajunsese la cantonul silvic. Mai avea până acolo 14 km. După 5 ore de mers, trecuse de acel canton şi era la 25 km de el. Care este distanţa dintre cabana şi canton?

Rezolvare: Situaţia prezentată în enunţ: A B I____________I_____I_______I D 14km C 25km Motociclistul pleacă din 퐷, după două ore ajunge în punctul 퐴, situat la 14 km de 퐶 (canton). După 5 ore de mers a trecut de 퐶, ajungând într-un punct 퐵, aflat la 25 km de 퐶. Lungimea drumului 퐴퐵 este evident 39 km, pentru parcurgerea distanţei 퐴퐵 a avut nevoie de 3 ore. Dacă distanţa este de 39 km şi timpul de 3 ore, atunci viteza este de 13 km/oră, iar distanţa 퐷퐶 cerută este 40 km. Ex:2 Un biciclist urcă cu 6 km/h şi coboară aceeaşi pantă cu 20 km/h. Ştiind că drumul urcat şi coborât a durat 3h şi 15 minute, să se afle lungimea drumului. Rezolvare(metoda falsei ipoteze): Dacă lungimea pantei este de 60km, spre exemplu,timpul de urcare va fi

Page 13: RMM 14.pdf

S.S.M.ROMÂNIA - Filiala Mehedinți 2014

13 REVISTA MEHEDINȚEANĂ DE MATEMATICĂ NR. 14

푡 = 60 푘푚: 6 푘푚/ℎ = 10 ore iar timpul de coborâre va fi 푡 = 60 푘푚 ∶ 20 푘푚 ℎ⁄ = 3 ore, timpul total fiind 푡 = 13 ℎ. Comparăm timpul total presupus cu cel real din problemă şi avem

= ⁄ = . Cum 푡 este de patru ori mai mic decât timpul presupus rezultă că drumul real este mai mic de patru ori decât drumul presupus (60 푘푚) adică 푑 = 15 푘푚. Probleme propuse 1(cu determinari de parametri specifici): 1. Un tren, mergând uniform, fără oprire, parcurge 60 푘푚, în 50 minute. Azi, a fost oprit 5 minute la jumătatea drumului. Cu ce viteză trebuie să meargă în continuare, pentru a ajunge la destinație fără întârziere? 2. Un autoturism pleacă din localitatea 퐴 spre localitatea 퐵, mergând cu o viteză medie de 40 푘푚 표푟ă⁄ . Ajungând în 퐵, se întoarce, fără oprire şi, mergând cu o viteză medie de 60 푘푚 표푟ă⁄ , ajunge în 퐴 după 5 ore de la plecare. Să se afle distanţa dintre cele două localităţi. 3. Un ciclist urcă o pantă cu viteza de 4 푘푚 표푟ă⁄ şi coboară aceeaşi pantă cu viteza de 16 푘푚 표푟ă⁄ . Ştiind că drumul, urcat şi coborât, adurat două ore, aflați lungimea drumului. Problemele de mișcare de tipul 2 și 3(probleme de „urmărire” pentru mobile care se deplasează în acelaşi sens sau probleme de „întâlnire” pentru mobile care se deplasează în sens contrar) au un algoritm de rezolvare: a) Identificarea modului de mișcare, a locului și momentului plecării. Pot apare situațiile: 2.1 - mobilele se mișcă în același sens și pornesc simultan în mișcare Modelul: Notăm: 퐷 = distanța între 퐴 și 퐵, 푡 = timpul de întălnire și folosim relația: 퐷 =푡(푣 − 푣 ),푣 ,푣 − vitezele constante ale celor două mobile. 2.2 - mobilele se mișcă în același sens și pornesc într-un interval de timp unul după altul Modelul: Notăm: 퐷 = distanța între퐴 și 퐵,푡 = timpul de întâlnire față de plecarea mobilului 2퐷 = (푡 − 푡 ) și folosim relația: 푡(푣 − 푣 ) = 푣 퐷 , unde 푣 ,푣 sunt vitezele constante ale celor două mobile 3.1- mobilele se mișcă în sens contrar și pornesc simultan în mișcare Modelul: Notăm: 퐷 = distanța între 퐴 și 퐵, 푡 = timpul de întâlnire și folosim relația: 퐷 = 푡(푣 + 푣 ), 푣 ,푣 - vitezele constante ale celor două mobile 3.2 - mobilele se mișcă în sens contrar și pornesc într-un interval de timp unul după altul Modelul: Notăm: 퐷 = distanța între퐴 și 퐵, 푡 =timpul de întâlnire față de plecarea mobilului 2, 퐷 = (푡 − 푡 )și folosim relația:퐷 = 푡(푣 + 푣 ) + 푣 퐷 , unde 푣 ,푣 sunt vitezele constante ale celor două mobile b) Reprezentarea pe o semidreaptă a pozițiilor mobilelor la diferite momente cu obiectiv: corelarea lor. c) Stabilirea expresiilor algebrice/legăturilor între distanțe și timpii corespunzatori d) Rezolvarea matematică/grafică a problemei

Exemplificăm problemele de mișcare de tipul 2 si 3 2.Probleme de „urmărire”/întâlnire a mobilelor care se deplasează în acelaşi sens 2.1 Mobilele pornesc (în acelaşi moment), în același sens „urmăritorul” se apropie de „urmărit”, într-o oră, cu o distanţă egală cu diferenţele

Page 14: RMM 14.pdf

S.S.M.ROMÂNIA - Filiala Mehedinți 2014

14 REVISTA MEHEDINȚEANĂ DE MATEMATICĂ NR. 14

vitezelor. distanţa ce-i separă va fi „recuperată” de „urmăritor” într-un timp egal cu câtul dintre această distanţă şi diferenţa vitezelor, acesta fiind timpul cerut, necesar „urmăritorului” pentru a-l ajunge. Din aceeaşi categorie fac parte problemele în care două mobile, mergând în acelaşi sens, străbat un acelaşi drum, cunoscându-se timpul de parcurs al fiecăruia şi diferenţa de viteză şi cerându-se lungimea drumului. Ex1 Două mobile au pornit (în acelaşi moment) din A spre B. Primul mobil face distanţa AB în 12 ore, iar cel de-al doilea în 15 ore. Ştiind ca viteza celui dintâi este cu 10 km/oră mai mare decât a celui de-al doilea, să se afle distanta AB

Rezolvare I______________________I__________I A O B După 12 ore de la plecarea din퐴a primului mobil, al doilea se află undeva pe drum (în punctul 푂) şi mai are de mers15− 12 = 3 ore. În fiecare oră, primul mobil a luat un avans de 10 푘푚 faţă de al doilea deci în cele 12 ore, avansul a fost de 12 × 10 = 120 푘푚. Cei 120 푘푚 reprezintă chiar distanţa 퐴푂 pe care o mai are de parcurs cel de-al doilea mobil cu viteza de 120 ÷ 3 = 40 푘푚 표푟ă⁄ . În cele 15 ore, distanţa 퐴퐵 este de 15 × 40 = 600 푘푚. 2.2 Mobilele se deplasează în acelaşi sens, astfel încât primul (a cărui viteză estecunoscută), plecând cu un timp dat înaintea celui de-al doilea, este ajuns de cel de-al doilea după un alt timp dat şi se cere viteza celui de-al doilea. Ex 2 La 4 ore de la plecarea dintr-un port a unui vapor care se deplasează cu viteza de 25 km/oră, porneşte o şalupă care ajunge vaporul după 5 ore. Care este viteza şalupei?

Rezolvare În cele 4 ore de la plecare, vaporul parcurge4 × 25 = 100 푘푚(avansul faţă de şalupa).Salupa ajunge vaporul după 5 ore, înseamnă că, în fiecare oră de mers, acesta a recuperat 100 ÷ 5 =20 푘푚, adică viteza sa este cu20 푘푚 ℎ⁄ mai mare decât viteza vaporului,adică viteza şalupeieste 25 + 20 = 45 푘푚 표푟ă⁄ . Ex 3 Un motociclist pleacă din oraşul A cu o viteză medie de 30 km/h. După 4 ore pleacă din acelaşi oraş şi în acelaşi sens un autoturism care are o viteză medie de 60 km/h. După cât timp autoturismul va ajunge din urmă motociclistul?

Rezolvare: (metoda aritmetică) moto v1=30km/h A auto v2=60km/h B C I I Motociclistul în cele 4 ore parcurge4 × 30 = 120 푘푚(se află în punctul 퐵). Pentru ca autoturismul să ajungă motociclistul, el trebuie să recupereze distanţa de 120 km.Când autoturismul pleacă la drum, motociclistul îşi continuă mişcarea spre punctul 퐶.Autoturismul recuperează într-o oră 60 − 30 = 30푘푚 표푟ă⁄ . Cei 120 km vor fi recuperați în 120 ÷ 30 = 4 ore.

Page 15: RMM 14.pdf

S.S.M.ROMÂNIA - Filiala Mehedinți 2014

15 REVISTA MEHEDINȚEANĂ DE MATEMATICĂ NR. 14

Rezolvare:(metoda algebrică) Se ştie că푑 = 푣푡. Deci 퐴퐶 = 60푡,퐵퐶 = 30푡de unde 퐴퐶 − 퐵퐶 = 60푡 − 30푡 = 4 × 30 de unde 푡 = 4ℎ. Probleme propuse 2-Probleme de „urmărire”/întâlnire a mobilelor care se deplasează în acelaşi sens 1. Un tren pleacă din oraşul퐴la ora 12, cu viteza de 35 푘푚 ℎ⁄ , iar altul pleacă tot din 퐴, la ora 12 şi 30 min, cu viteza de 70 푘푚 표푟ă⁄ . Când va ajunge trenul al doilea pe primul şi la ce distanţa de oraşul퐴? 2. De pe un aeroport a decolat un avion ce se deplasează cu viteza de 240 푘푚 표푟ă⁄ . După două ore, decolează un alt avion, care-l ajunge pe primul după 4ore. Care este viteza de zbor a celui de-al doilea avion? 3.Distanţa dintre localităţile 퐴 şi 퐵 este de120 푘푚 ℎ⁄ .Din 퐴 pleacă spre퐵 un motociclist cu viteza de 40 푘푚 ℎ⁄ .După o oră şi jumătate pleacă din 퐴 o maşină care ajunge în 퐵 în acelaşi timp cu motociclistul.Aflaţi viteza maşinii. 4.Un tren mergând cu o viteză constantă, parcurge distanţa de 300 km între oraşele 퐴 şi 퐵 în 5ore. După o oră, pleacă din 퐴 un alt tren care îl ajunge pe primul într-o staţie intermediară aflată la120 km de 퐵.Aflaţi viteza celui de-al doilea tren. 3.Probleme de „întâlnire” a mobilelor când deplasarea se face în sens contrar 3.1 Mobilele au pornit (în acelaşi moment), mergând în sens contrar, se apropie unul de altul, într-o oră, cu o distanţă egală cu suma vitezelor. Distanta ce-i separa va fi„acoperită” într-un timp egal cu câtul dintre acestă distanţă şi suma vitezelor. Ex 1 Distanţa dintre două localităţi A si B este de 160 km. Din A si B pleacă, în acelaşi timp, unul spre celălalt, un motociclist, care merge cu viteza medie de 30 km/oră şi respectiv un automobilist, care se deplasează cu viteza medie de 50 km/oră. După cât timp se întâlnesc cei doi?

Rezolvare: La momentul iniţial, între cele două mobile este o distanţă de160 km.După fiecare oră, distanţa dintre ei se micşorează cu 30 푘푚 + 50 푘푚 = 80 푘푚,până când, la întâlnire, distanţa va fi 0. Deci, după fiecare oră, din cei 160 km se scad 80 km adică, cei doi se întâlnesc după două ore(160 ÷ 80 = 2). Ex 2 Din oraşele A şi B aflate la o distanţă de 210 km, pornesc unul spre altul în acelaşi timp, doi

motociclişti. Viteza medie a motociclistului care pleacă din A este 43 din viteza celuilalt.

După două ore de la pornire, cei doi mai aveau de parcurs, până la întâlnirea lor, 70 km. Aflaţi viteza medie a fiecărui motociclist.

Rezolvare:(metoda aritmetică):

A v1 70 km v2 B I I I I I I I I I

11 v2d 22 v2d

Deoarece 푣 = 푣 ⇒ 푑 = 푑 , timpul fiind acelaşi. Reprezentarea grafică a distanţelor parcurse:

Page 16: RMM 14.pdf

S.S.M.ROMÂNIA - Filiala Mehedinți 2014

16 REVISTA MEHEDINȚEANĂ DE MATEMATICĂ NR. 14

d2 I I I I I

210 km d1 I I I I

Cele 7 părţi, fiecare parte fiind egală cu din푑 reprezintă 210 푘푚 − 70 푘푚 = 140 푘푚. Primul în două ore a parcurs140 ÷ 7 × 3 = 60 (km), al doilea în două ore a parcurs140 ÷7 × 4 = 80(km),sau × 푑 = 60 km⇒ 푑 = 60 ÷ 3 × 4 = 80(km). Viteza medie a motociclistului care a plecat din 퐴,a fost 60 ÷ 2 = 30 (km/h),viteza medie a motociclistului care a plecat din 퐵,a fost 80 ÷ 2 = 40(km/h) sau 푣 = 30km/h⇒ 푣 = 30 ÷3 × 4 = 40(km/h). Rezolvare: (metoda algebrică) Dacă 푑 = 2푣 şi푑 = 2푣 , iar 푣 = 푣 ⇒ 2푣 + 2푣 + 70 푘푚 = 210 푘푚 ⇒ 2 푣 +2푣 = 140 푘푚 ⇒ 3푣 + 4푣 = 2 × 140 푘푚 ⇒ 7푣 = 280 푘푚 ⇒ 푣 = 40 푘푚 ℎ⁄ ⇒ 푣 =30푘푚 ℎ⁄ . 3.2- mobilele se mișcă în sens contrar și pornesc într-un interval de timp unul după altul

Ex 1 La ora 12 si 35 minute pleacă trenul personal care se deplasează din A spre B cu viteza de 40 km/oră. La ora 17 si 45 minute pleacă un tren accelerat, din B spre A, cu viteza de 72 km/oră. La ora 18 si 35 minute,ele se încrucişează. La ce oră ajunge personalul în B?

Rezolvare: situaţia din problemă: p a I__________________I________I A I B Iniţial, personalul se află în 퐴, iar acceleratul în 퐵,퐼 fiind viitorul punct de întâlnire.În momentul întâlnirii, personalul mai are de străbătut distanţa 퐼퐵 parcursă de accelerat. Deplasarea sa a durat de la ora 17 şi45 minute până la ora 18 şi 35 minute, adică 50 min = 5 6⁄ ore.Deci,distanţa parcursă de accelerat, până la punctul de întâlnire, a fost de 72 ×5 6⁄ = 60 km. Trenul personal mai are de străbătut până în 퐵, o distanţă de60 km, cu viteza de 40km/oră. Deci, timpul necesar este 60 ÷ 40 = 3 2⁄ ore, adică 1 oră și 30 min. Din punctul de întâlnire퐼, unde se afla la ora18 și 35 minute,personalul mai are de mers1 oră şi 30 minute, deci ora la care ajunge în퐵 este:18 ore şi 35 minute+1 oră şi30minute= 20 ore 5 minute. Probleme propuse -3 Probleme de „întâlnire” a mobilelor când deplasarea se face în sens contrar 1. La ora 12 şi 35 minute, pleacă un tren din 퐴 spre퐵, cu viteza de 60 km/oră, iar la ora14 și 5 minute pleacă din 퐵 spre퐴 un alt tren, cu viteza de 80 km/oră.Distanţa dintre gările 퐴 și퐵 este de 398 km. La ce oră şi la ce distanţă de 퐴 se întâlnesc cele două trenuri? 2. Pe autostrada dintre două oraşe, la aceeaşi oră, pornesc unul către altul doi motociclişti. După 2 ore și 40 minute de la plecare, primul motociclist a parcurs 7 10⁄ din drum, al doilea3 5⁄ , iar distanţa dintre ei era de 60 km. Se cere: a) distanţa dintre cele două oraşe; b) distanţa parcursă de fiecare motociclist; c) viteza fiecărui motociclist.

Page 17: RMM 14.pdf

S.S.M.ROMÂNIA - Filiala Mehedinți 2014

17 REVISTA MEHEDINȚEANĂ DE MATEMATICĂ NR. 14

3.Distanţa dintre două localităţi 퐴 şi 퐵 este de 60 km.Din 퐴 pleacă un pieton cu viteza de 5 km/h şi în acelaşi timp pleacă din 퐵 un biciclist cu viteza de25 km/h, mergând unul spre celălat.După cât timp se întâlnesc pietonul cu biciclistul? Care este distanţa faţă de localitatea 퐵 a punctului de întâlnire? 4.O barcă cu motor mergând în sensul apei străbate distanţa de la 퐴 la 퐵 în 8 ore, iar în contra apei de la퐵 la 퐴 în 10 ore. În câte ore va străbate distanţa de la 퐴 la퐵 o plută care este luată de curentul apei? Probleme propuse pentru concursurile şcolare 1) Din Bucureşti și Ploieşti pleacă simultan, unul spre celălalt, câte un autobuz cu vitezele constante 푣 = 60 푘푚 ℎ⁄ şi respectiv 푣 = 40 푘푚 ℎ⁄ . În acelaşi moment, dintr-unul din autobuze îşi ia zborul spre celălalt autobuz un porumbel călător, care continuă să zboare neîntrerupt, între cele 2 autobuze, de la unul la celălalt, cu viteza constantă 푣 = 70 푘푚 ℎ⁄ , până la întâlnirea autobuzelor. Ce drum total străbate porumbelul? Distanța Bucureşti- Ploieşti este de 60 km. 2) Un biciclist calculează că dacă va merge cu 35 푘푚 ℎ⁄ va ajunge la timp la destinaţie.El parcurge jumătate de drum cu 40 푘푚 ℎ⁄ .A ajuns cu o oră mai devreme.Să se afle lungimea drumului.

ONM , clasa a VIII-a , 1980 Cezar se antrenează pentru un concurs de atletism.În

fiecare zi parcurge un traseu asemănător cu cel din figură , dus-întors , în total 5 ore.Ştiind că viteza pe drum drept

este 4 푘푚 ℎ⁄ şi că la urcare este de 3 푘푚 ℎ ⁄ iar la coborâre de 6 푘푚 ℎ⁄ să se afle câţi kilometri parcurge elevul zilnic.

Concursul Interjudeţean „Florica T. Câmpan” 3) Trei persoane 퐴, 퐵, şi 퐶 trebuie să parcurgă o distanţă de10 푘푚 între două localităţi.Viteza lor de deplasare , mergând pe jos, este de 5 푘푚 ℎ⁄ .퐴 are o motocicletă cu două locuri, a cărei viteză este de 25 푘푚 ℎ⁄ . Ei pornesc în acelaşi moment, 퐴 şi퐵 pe motocicletă, iar 퐶 pe jos.Într-un punct al traseului, 퐴 opreşte, iar 퐵 coboară şi îşi continuă drumul pe jos, spre destinaţia lor.퐴 se întoarce şi când se întâlneşte cu 퐶, acesta urcă pe motocicletă şi ambii pleacă spre destinaţie.Interesant este că cei trei sosesc simultan la destinaţie.(Se neglijează timpii necesari opririi motocicletei, coborârii sau urcării unui pasager).Care este distanţa parcursă de 퐴 cu motocicleta?Care este timpul în care cei trei au ajuns la destinaţie?

Bibliografie: 1) „Probleme de mişcare”- Dan Gurgui, Ed. Sitech, 2010 2) „Concursul de matematică Florica T. Câmpan”, Ed. Taida, 2007 3) Olimpiade şi concursuri şcolare 4) Cherata V., Voicilă J., Mîndruleanu S., „Metode şi tehnici de

rezolvare a problemelor de aritmetică”, Editura Sibila, 1997 5) Roşu M., „Matematică pentru formarea profesorilor din

învăţământul primar”, Editura Meteor Press, 2005

Page 18: RMM 14.pdf

S.S.M.ROMÂNIA - Filiala Mehedinți 2014

18 REVISTA MEHEDINȚEANĂ DE MATEMATICĂ NR. 14

Temă pentru grupa de performanță la clasa a VI-a DIVIZORII UNUI NUMĂR NATURAL

Prof. Draga Tătucu Mariana, Colegiul Național „Gheorghe Țițeica”

INTRODUCERE Tema propusă: ,,Divizorii unui număr natural” constituie o extindere a programei analitice obligatorii de matematică și parcurgerea ei este necesară pentru abordarea unor probleme mai dificile. O categorie aparte de probleme, ignorată de manualele unice din anii precedenți și prezentată sumar în actualele manuale alternative, problemele de numărare apar destul de des în ultimul timp ca subiecte de olimpiadă, concursuri.

NUMĂRUL DIVIZORILOR UNUI NUMĂR NATURAL Fie 푎 un număr natural compus ce are următoarea descompunere în factori primi: 푎 = 푝 ∙푝 ∙ … ∙ 푝 ,unde 푝 , 푝 , … , 푝 sunt numere prime, iar 훼 ,훼 , … , 푎 ∈ ℕ (forma canonică a lui 푎). Pentru a obține numărul divizorilor lui 푎, formăm tabelul: 푝 푝 푝 … … … … …푝 훼 + 1 termeni 푝 푝 푝 … … … … …푝 훼 + 1 termeni … … … … … … … … … … … … … … … … … … … … … … … 푝 푝 푝 … … … … …푝 훼 + 1 termeni Observăm că fiecare număr din tabel este un divizor pentru훼; linia푖 conține훼 + 1 termeni. Dacăînmulțim fiecare număr din prima linie cu fiecare număr din a doua linie, obținem (훼 + 1)(훼 + 1)divizori ai lui 푎. Înmulțim apoi fiecare din aceste numere cu fiecare număr din linia 3 și obtinem (훼 + 1)(훼 + 1)(훼 + 1)divizori ai lui 푎. Continuând raționamentul obținem(훼 + 1)(훼 +1) … (훼 + 1) numere care sunt divizori ai lui 푎. În numărul acestor divizori este inclus și divizorul 1. Am obținut astfel următoarea Teorema: Numărul divizorilor numărului 푎 = 푝 ∙ 푝 ∙… ∙ 푝 este

휏(푛) = (훼 + 1)(훼 + 1) … (훼 + 1). Pentru 푛 număr natural, vom nota cu 흉(풏)- numărul divizorilor naturali ai lui 푛. Exemple. 1) Determinați numărul divizorilor lui360. Soluție: 3600 = 2 ∙ 3 ∙ 5 . Rezultă (4 + 1)(2 + 1)(2 + 1) = 45 divizori. 2) Câți divizori în mulțimea numerelor naturale are numărul 2 ∙ 5 + 2 ∙ 5. Soluție: Numărul dat se poate scrie2 ∙ 5 ∙ 3 . Rezultă (8 + 1)(1 + 1)(3 + 1) = 72 divizori. 3) Determinați toate numerele de forma 푎 = 2 ∙ 3 , unde 푚și푛 sunt numere naturale care au exact7 divizori. Solutie:(푚 + 1)(푛 + 1) = 7; rezultă numerele3 , 2 . 4) Să se determine toate numerele scrise în baza 10 care sunt divizibile cu 15și au14 divizori. Soluție: 15|퐴 ⇒ 퐴 = 3 ∙ 5 ∙ 푝 ∙ 푝 … ; 푥,푦 ≠ 0. 휏(퐴) = (푥 + 1)(푦 + 1)(푘 + 1)(푘 + 1) …

Page 19: RMM 14.pdf

S.S.M.ROMÂNIA - Filiala Mehedinți 2014

19 REVISTA MEHEDINȚEANĂ DE MATEMATICĂ NR. 14

Cum푥 + 1 ≠ 1 și 푦 + 1 ≠ 1 ⇒ 푥 + 1 = 2 și 푦 + 1 = 7 sau 푥 + 1 = 7 și 푦 + 1 = 2. Deci푥 =1 și 푦 = 6 sau 푥 = 6 și 푦 = 1. Numerele care satisfac condiția problemei sunt퐴 = 3 ∙ 5 sau 퐴 = 3 ∙ 5.

SUMA DIVIZORILOR UNUI NUMĂR NATURAL (σ(n)). Sa calculăm mai întâi suma S =1+x+x2 +………………….xn. Se știe că S= ( x n+1 -1 )/ ( x – 1 ) ; cu x≠1. Scriem produsul de n sume având termenii pe cele n linii din table și obținem (1+p1+p1

2………+p1

α1)(1+p2+p22+……….+p2

α)……….(1+pn+pn2+……………pn

α)= p 1α1+1 - 1 · p 2α2+1 -1 …………· pn

αn -1 P1 - 1 p2 – 1 pn -1 Am obținut astfel teorema : suma divizorilor numărului a=p1

α1 · p2α ·……..pn

αn este S = p1

α1 -1 - p2α2 - 1……….pn

αn - 1 p1 -1 p2 - 1 pn - 1

Exemplu .Fie S suma divizorilor naturali ai nr 2001. Să se arate că 5S este un număr natural pătrat perfect. Soluție: 2001 = 3· 23 ·29 de unde σ(n)=3² - 1 . 23² - 1 . 29² - 1 = 4·24·30 3-1 23 – 1 29 - 1 Atunci 5S=(2³ · 3 · 5 )² = 120² , deci 5S este pătrat perfect. Lemă. Dacă d1 ,d 2,…..dk sunt toți divizorii naturali ai numărului n atunci avem relația : ( d1 · d2 · ………….dk )2 = nk.

Soluție.Considerăm d1< d2<…….<dkși obținem d1=n/dk ,d2 = n/dk-1…. dk= n/d1 ,relații care înmulțite membru cu membru dau d1.d2 ……dk =(n/d1 )(n/d2)……..(n/dk), de unde ( d1 · d2 ·….dk)2= nk . Exemplu. Să se calculeze produsul tuturor divizorilor naturali ai numărului 2001. Soluție.2001= 3.23.29 , iar numărul divizorilor este (1+1)(1+1)(1+1)=8 ;rezultă relația ( d1· d2··….d8 )2 =20018 și d1 · d 2 · ….d8 = 20014

Bibliografie

,,Probleme de numărare”-Amalia Romilă; Editura EduSoft ,,Matematică pentru grupele de performanță”-Gheorghe Lobonț;

Editura Dacia Educațional Revista „Arhimede” ,Gazeta matematică, Internet

Temă pentru grupa de performanță la clasa a VI-a

Principiul „cutiei”

Elevii Bușe Iasmina, Grecu Bogdan, Ceaușene Patricia Îndrumător Prof. Dr. Căiniceanu Gheorghe

Acest principiu este mai mult o observaţie de bun simţ care, de-a lungul timpului s-a dovedit foarte utilă în rezolvarea unor probleme. Se pare că pentru prima oară a fost utilizat de către matematicianul german Dirichlet. Din acest motiv i se mai spune și principiul lui Dirichlet.

Dar care este acest principiu? Pentru a-l înţelege bine vom porni de la un exemplu foarte simplu.Dacă aveţi trei mere pe care trebuie să le puneţi în două coşuri, atunci va trebui ca

Page 20: RMM 14.pdf

S.S.M.ROMÂNIA - Filiala Mehedinți 2014

20 REVISTA MEHEDINȚEANĂ DE MATEMATICĂ NR. 14

într-un coş să puneţi două mere. Despre aceasta este vorba; dacă am mai multe obiecte de aşezat în mai puţine cutii, atunci într-o cutie trebuie să aşez mai multe obiecte. Iată însă cum se enunţă acest principiu la modul general: Dacă avem un număr N de cutii si un număr N+1 de obiecte, atunci cel puţin într-o cutie vor fi două obiecte. Să numerotăm cutiile cu:퐶 ,퐶 ,퐶 … … …퐶 şi obiectele cu:푂 ,푂 ,푂 … … …푂 ,푂 Să presupunem că în cutia 퐶 se afla obiectul 푂 , în cutia 퐶 obiectul 푂 şi aşa mai departe, în cutia 퐶 se află obiectul 푂 . În acest fel am terminat cutiile, dar ne-a mai rămas un obiect. Ce facem cu el ? Îl introducem într-o cutie unde se mai află deja un obiect. Aşadar, într-o cutie se vor afla două obiecte.

Prezentăm în continuare câteva exemple interesante. Vom vedea că cel mai important aspect legat de aplicarea acestui principiu constă în a descoperi de fapt care sunt cutiile şi, respectiv, care sunt obiectele.

Începem cu o problemă foarte simplă dar utilă pentru a înţelege mecanismul după care funcționează principiul cutiei.

Problema 1 : Dacă întrebăm la întâmplare 367 de persoane care este ziua lor de naștere vom găsi cel puţin două persoane născute în aceeaşi zi.

Soluţie: În aceasta problemă "cutiile" sunt zilele anului. Înseamnă că avem 365 sau 366 de "cutii". Obiectele sunt cele 367 de persoane. Se observă că avem mai multe obiecte decât cutii. Atunci, conform principiului cutiei, vom avea cel puţin o cutie cu două obiecte. Aşadar, vor fi cel puţin două persoane născute în aceeaşi zi.

Problema 2: La o lucrare de matematică, cei 25 de elevi ai unei clase au luat note de la 5 la 10, inclusiv. Arătaţi că există cel puţin 5 elevi care au luat aceeaşi notă .

Soluţie: Elevii puteau obţine notele: 5, 6, 7, 8, 9 sau 10. Acestea vor fi cutiile; în total avem 6 cutii. Obiectele sunt reprezentate de cei 25 de elevi. Dacă în fiecare cutie vom pune câte 4 obiecte înseamnă că am fi folosit 6*4 = 24 (obiecte). Dar erau 25 de elevi (adică de obiecte), aşadar într-o cutie vor fi 5 obiecte (elevi). În concluzie, cel puţin 5 elevi au obţinut aceeaşi notă.

Problema 3. Demonstraţi că, printre 81 de numere naturale ai căror divizori primi se află în mulţimea {2, 3, 5}, există 4 al căror produs este puterea a patra a unui număr natural.

Soluţie: Observăm că numerele sunt de forma: n = 2 ∙ 3 ∙ 5 Se caută 4 numere 푛 , 푖 ∈ {1,2,3,4} astfel încât: 푛 ∙ 푛 ∙ 푛 ∙ 푛 = 2 ∙ 3 ∙ 5 = 푚 ⇔ 푎 + 푎 + 푎 + 푎 = 4 ∙ 푘 ,푏 + 푏 + 푏 + 푎 = 4 ∙ 푘 şi 푐 + 푐 + 푐 + 푐 = 4 ∙ 푘

Putem identifica fiecare număr n cu tripletul puterilor la care se ridică factorii săi primi, adică cu tripletul (푎,푏, 푐). Oricărui număr îi corespunde un unic triplet şi orice triplet determină un singur număr. Deci avem 81 triplete. Printre acestea trebuie să demonstrăm că există 4 astfel încât (푎 + 푎 + 푎 + 푎 , 푏 + 푏 + 푏 + 푏 , 푐 + 푐 + 푐 + 푐 ) =(4 ∙ 푘 , 4 ∙푘 , 4 ∙ 푘 )

Împărţim numerele în 9 grupe de câte 9 elemente. Tripletului (a,b,c) îi asociem tripletul (푟 , 푟 , 푟 ) , pe care îl denumim tipar, format cu resturile la împărţirea cu 2 ale lui a,b,c.

Avem în fiecare grupă nouă tipare care pot lua 8 forme diferite. Conform principiului cutei, vor exista două cu acelaşi tipar. Tripletele corespunzătoare acestor tipare au componentele care se corespund de aceeaşi paritate. Adunându-le obţinem un triplet cu toate componentle pare de forma: (2 ∙ 푘 , 2 ∙ 푘 , 2 ∙ 푘 ).

Page 21: RMM 14.pdf

S.S.M.ROMÂNIA - Filiala Mehedinți 2014

21 REVISTA MEHEDINȚEANĂ DE MATEMATICĂ NR. 14

Acest lucru se va întâmpla în toate grupele. Găsim astfel 9 triplete cu componente pare.

Asociem fiecărui triplet astfel obţinut câte un tipar, considerând că acum tiparul este tripletul format cu resturile la împărţirea cu 4.

Cum resturile nu pot lua decât valori pare: 0 sau 2, avem doar 8 variante diferite de astfel de tipare. Cum sunt 9 numere, vom obţine două cu acelaşi tipar. Adunând tripletele corespunzătoare, obţinem un triplet ce are toate elementele multiplii de patru, deci găsim patru numere ce respectă cerinţa problemei.

Problema 4. Se consideră 25 de numere naturale care divid pe 2014 . Să se arate că există patru printre acestea având produsul pătratul unui pătrat perfect.

Soluţie: 2014 = 2 ∙ 19 ∙ 53 Fiecare din cele 25 de numere va avea forma 2 ∙ 19 ∙ 53 , cu 푥,푦, 푧휖ℕ şi 푥,푦, 푧 ≤ 2014. Exponentul fiecărui număr prim poate fi par sau impar. Avem 8 posibilităţi: 2 ∙ 19 ∙53 , 2 ∙ 19 ∙ 53 , 2 ∙ 19 ∙ 53 , 2 ∙ 19 ∙ 53 , 2 ∙ 19 ∙ 53 , 2 ∙ 19 ∙ 53 , 2 ∙ 19 ∙ 53 , 2 ∙19 ∙ 53 .

Considerăm 9 numere din cele 25. Aplicând principiul cutiei, din cele 9 numere naturale oarecare, există cel puţin două care au aceeaşi formă a exponenţilor. Fie 푎 si 푏 cele două numere. Rezultă că există 푝 pătrat perfect, astfel încât 푝 = 푎 ∙ 푏

Separăm cele două numere 푎 ş푖 푏 , le păstrăm pe celelalte 7 şi mai adăugăm două din cele 25-9=16 numere rămase, obţinând din nou 9 numere.

Aplicând principiul cutiei si procedând analog de încă 7 ori, obţinem 푝 = 푎 ∙ 푏 , 푝 = 푎 ∙ 푏 , … , 푝 = 푎 ∙ 푏 .

Formăm grupa 푝 ,푝 , … … , 푝 . Toate cele 9 pătrate perfecte au fiecare din cei trei exponenţi (corespunzători bazelor 2, 19, şi 53 ) de forma 4푘 sau 4푘 + 2.

Aplicând principiul cutiei, există două pătrate perfecte 푝 ş푖 푝 cu aceeaşi formă a exponenţilor. Atunci produsul 푝 ∙ 푝 = 푝 = 푎 ∙ 푏 ∙ 푎 ∙ 푏 , ceea ce trebuia demonstrat.

Temă pentru grupa de performanță la clasa a- IX -a

O generalizare a problemei nr.3 propusă la clasa a IX-a pentru Olimpiada de Matematică –Etapa locală 2014

Prof. Preșneanu Doru - Colegiul Național Pedagogic „Ștefan Odobleja”

Enunţ: Se consideră ABC , oarecare , 푀휖(퐵퐶), 푁휖(퐴퐶) si 푃휖(퐴퐵), astfel încât

MCBM , NACN unde R , date. Ştiind că AB=c, AC=b, BC=a, AM, BN si CP

concurente și 0 PFNEMD , unde D, E si F sunt punctele de intersecţie a cercului circumscris ABC cu AM, BN și respectiv CP, atunci are loc relaţia:

AMabc

a222

2

)1()1(

+ BN

bcab

222

2

)1()1(

+

CPcba

c222

2

)1()1(

=0 .

Page 22: RMM 14.pdf

S.S.M.ROMÂNIA - Filiala Mehedinți 2014

22 REVISTA MEHEDINȚEANĂ DE MATEMATICĂ NR. 14

Rezolvare: Cum ABCADC , DMCAMB , rezultă că CMDAMB deci

MDBM

CMAM

BMCMMDAM . Dar MCBM , atunci MCBM . Avem şi

,aMCBM deci

1

aMC . Cum A, M, D coliniare , rezultă

xAMMDAMxMD . Avem

2

22

2

22

)1()1(

aAMxaMCMDAM 22

2

)1( AMax

(1).

Cum MCBM ACABAMACABAM

1

11

1

ACABACABAM 2

)1(1 222

22 . Dar 22

cAB , 22bAC ,

cos ACABACAB . Deci AbcbcAM cos2)1(

1 2222

2

. Din

teorema cosinusului avem 222cos2 acbAbc , deci

2222222

2

)1(1 acbbcAM

2

2222

)1()1()1(

abcAM (2).

Din (1) si (2) avem 222

2

2

2

)1()1()1(

)1( abcax

222

2

)1()1( abcax

Analog se calculează 2

2222

)1()1()1(

bcaBN . Dacă BNyNE , atunci

22

2

)1( BNby

222

2

)1()1( bcaby

Analog se calculează 2

2222

)1()1()1(

cabCP , unde CBAP .

Page 23: RMM 14.pdf

S.S.M.ROMÂNIA - Filiala Mehedinți 2014

23 REVISTA MEHEDINȚEANĂ DE MATEMATICĂ NR. 14

Conform teoremei lui Ceva, avem

11 , deci

2

2222

)1()1()1(

cabCP . Dacă CPzPF , atunci 22

2

)1( CPcz

222

2

)1()1( cbacz

. Cum 0 PFNEMD

0 CPzBNyAMx , de unde rezultă ceea ce trebuia demonstrat.

Relaţii de recurenţă bine definite, generate de bisectoarea interioară a unui unghi într-un triunghi

Prof. Carmen - Victoriţa Chirfot - Colegiul Tehnic „Domnul Tudor”

Fie triunghiul dreptunghic isoscel ABC, dreptunghic în B

şi catetă a, 0a . Construim bisectoarea 1AM interioară

unghiului BAC, 1 ( )M BC . Bineînţeles, ipotenuza 2BC a

şi 1ˆ

8m BAM

. Fie 1 1BM x , 10 x a şi 1 1M C y ,

10 y a .

Vom scrie tangenta unghiului 1ˆBAM . Deci, 1

1ˆ xtg BAM

a .

Aplicând teorema bisectoarei interioare corespunzătoare

unghiului ˆBAC , avem 1 1

2a ax y , adică

1 1

2a ax y

1 1

2a ax y

2a aa

1 2a

a

1 2 . Deci,

1 1

1 2 1 2

8

a ax ytg

1 2 18 1 2

tg , 1 2 1x a şi 1 2 2 1y a 2 2a . Deci, un prim

rezultat dedus este că 2 18

tg

. Să calculămşi lungimea segmentului 1AM . Evident,

22 2 2 2 2 2 2

1 1 1 2 1AM AB BM a x a a 1 4 2 2AM a .

Construim, în continuare bisectoarea interioară unghiului 1ˆBAM , pe care o notăm

2 2 1, ( )AM M BM şi fie 2 2AM x , 2 10 x x şi 2 1 2M M y , 2 10 y x . Avem

Page 24: RMM 14.pdf

S.S.M.ROMÂNIA - Filiala Mehedinți 2014

24 REVISTA MEHEDINȚEANĂ DE MATEMATICĂ NR. 14

22

ˆ16

xtg tg BAMa

şi aplicând teorema bisectoarei interioare unghiului 1ˆBAM , avem

şi 2 2

4 2 2a ax y

2 2

4 2 2a ax y

4 2 22 1

a aa

1 4 2 22 1

. Deci,

2

2 1

4 2 2 1

ax

, adică

2

2 1 4 2 2 1

4 2 2 1 4 2 2 1

ax

2

2 1 4 2 2 1

3 2 2

ax

2

2 1 4 2 2 1

3 2 2

ax

2 2

2 1 4 2 2 1

2 1

ax

2

4 2 2 1

2 1

ax

2 4 2 2 1 2 1x a 4 2 2 1 2 116

tg

.

Construim din aproape în aproape bisectoarele unghiurilor 2ˆBAM , 3

ˆBAM , …,

respectiv 1ˆ

nBAM , şi anume, 3AM , 4AM , …, nAM , unde 3 4, , ..., ( )nM M M BC , şi

notăm segmentele i iBM x , 1i i iM M y , *i N , 2i (notaţie folosită şi pentru segmentele deja construite; pentru 1i avem 1 1BM x şi 1 1M C y ) şi i n . Aplicând teorema bisectoarei pentru bisectoarea nAM în triunghiul 1nBAM , deducem relaţia

1

1

n

n n n

AMABBM M M

1n

n n

AMax y

(1). Dar, 2 2 21 1n nAM AB BM , în triunghiul 1nABM

dreptunghic în B. Deci, 2 2 21 1n nAM a x 2 2

1 1n nAM a x . Revenim în relaţia (1) şi

găsim 2 2 2 2

1 1n n

n n n n

a x a a xax y x y

. Evident că 1n n nx y x , deci

2 2 2 21 1

1

n n

n n n

a x a a xax y x

. Dacă luăm primul şi ultimul termen al egalităţii, obţinem

relaţia de recurenţă 2 2

1 *

1

, , 2n

n n

a a xa n N nx x

, cu primul termen

1 2 1 , 0x a a . Dar, 2

2

12

2

n nn

n

a x a tgx tg

.

Probleme propuse: 1) Să se calculeze

12tg ,

10tg şi

5tg , folosind teorema bisectoarei într-un triunghi.

Page 25: RMM 14.pdf

S.S.M.ROMÂNIA - Filiala Mehedinți 2014

25 REVISTA MEHEDINȚEANĂ DE MATEMATICĂ NR. 14

2) Se dă 2 21n n na y x a x , 0a şi 1n n nx y x , * , 2n N n şi 1 2 1x a . Să se

găsească formula termenului general al şirului 1n nx

, respectiv al şirului 1n n

y

. Să se cerceteze monotonia celor două şiruri! 3) Să se găsească formula termenului general al şirului 1n n

z

definit prin recurenţa 2

1 1 11n n n nz z z z , *n N , 2n , unde 1 1z . Care este formula termenului dacă

1 , 0,2

z ctg

. Dar dacă 1 , 0,

2z tg

?

4) Să se găsească formula termenului general al şirului 1n nz

definit prin recurenţa

21 1 11n n n nz z z z , *n N , 2n , unde 1 0z a (ce se întâmplă dacă 0a ?) şi să

se studieze monotonia şirului. Bibliografie: [1]. Colecţia Revistei de Matematică din Mehedinţi. [2]. Colecţia Revistei de Matematică din Timişoara. [3]. Colecţia Gazetei Matematice.

Temă pentru grupa de performanță la clasa a- X -a Convexitate Jensen

Prof.Dr. Gheorghe Căiniceanu 1.ABSTRACT Convexitatea și convexitatea în sens Jensen vor fi prezentate, urmărind cu predilecție tehnici prin care se pot deduce diverse inegalități implicate adesea în probleme de concurs. Articolul conține prezentarea noțiunilor, teoreme de „propagare” a convexității prin compuneri și aplicații. 2.CONVEXITATE Din punct de vedere intuitiv, o funcție este convexă pe intervalul I dacă pe acest interval reprezentarea sa grafică se află „sub” coarda AB a capetelor A(a,f(a)), B(b,f(b)), C((1-t)a+tb,(1-t)f(a)+tf(b)), D((1-t)a+tb, f[(1-t)a+tb]), t[0,1]. Avem , de fapt yD<yC . Se folosește adesea exprimarea ca un astfel de grafic “ține apa”. OBSERVAȚIA 1 Un punct arbitrar al intervalului [a.b] se obține prin expresia

(1-t)a+tb, luând un t arbitrar în [0,1] DEFINIȚIA 2 Spunem că funcția

Rbaf ],[: este convexă dacă)()()1(])1[(],[,],1,0[ ytfxfttyxtfbayxt

.

A

C

B

D

a b u

Page 26: RMM 14.pdf

S.S.M.ROMÂNIA - Filiala Mehedinți 2014

26 REVISTA MEHEDINȚEANĂ DE MATEMATICĂ NR. 14

În articolul de față vom considera pentru început o formă mai slabă a conceptului de funcție convexă și anume convexă în sens Jensen: DEFINIȚIA 3. Fie I un interval închis de numere reale. O funcție RIf : , cu proprietatea

2

)()()2

(, bfafbafIba

se numește funcție convexă (Jensen). În cazul schimbării sensului inegalității spunem că avem o funcție concavă. TEOREMA 4.Fie RIf : o funcție convexă.Atunci :

(1)n

afafn

aafIaaa nnn

)(...)()...(,..., 1121

Demonstrație: Se găsește într-o formă amănunțită și accesibilă în [3], dar voi puncta o parte din ea pentru învățăturile ce se pot extrage în practica rezolvării de probleme. In prima fază se arată că dacă (1) este adevărată pentru n numerate, atunci este adevarată și pentru 2n numere

nafaf

n

aafaaf

n

aaaa

fn

aaf

n

nnnn

n

2)(...)(

)2

(....)2

()2

...2()

2...(

21

2122121221

21

În a doua fază vom face demonstrarea că dacă (1) este adevărată pentru n+1 numere, va fi adevarată și pentru n numere.În limbajul de la școală am putea sa-i spunem “varianta a III-a de inducție”. (In [2] metoda se numește Inducție Cauchy). Doresc să arăt că (1) este adevărată pentru numerele a1,a2,…an. Voi folosi că este adevărată

pentru n+1 numere și anume a1,a2,…an, n

aa n ...1 .Scrierea efectivă a relațiilor este un bun

exercițiu pentru tinerii noștri cititori la cercul de matematică. În mod analog se poate demonstra și : PROPOZIȚIA 5.Dacă funcția Rbaf ],[: satisface condiția 0],,[,),(2)()( abayxxyfyfxf atunci pentru oricare ],[,..., 21 baxxx n avem:

nnn xxnfxfxf )...()(....)( 11 .

Nu vom încheia paragraful dedicat cunoașterii funcțiilor convexe înainte de a preciza că pentru o funcție convexă în sensul definiției 2 este cunoscută : TEOREMA 6.Fie Raaa n,...,, 21 cu suma 1 și ],[,...,, 21 baxxx n , Rbaf ],[: . Inegalitatea: )...()(...)( 1111 nnnn xaxafxfaxfa este adevărată pentru orice n număr natural nenul dacă și numai dacă este adevărată pentru n=2. 3. CONVEXITATEA ÎN COMPUNERI DE FUNCȚII Vom începe acest paragraf cu o listă de funcții convexe necesară în primul rând elevilor de clasa a X-a mai puțin familiarizați cu noțiunea de funcție convexă. 1. nxxfRf )(,),0[: , este convexă și crescătoare

Page 27: RMM 14.pdf

S.S.M.ROMÂNIA - Filiala Mehedinți 2014

27 REVISTA MEHEDINȚEANĂ DE MATEMATICĂ NR. 14

2. xaxfRf )(),,0(: , 1a este convexă, crescătoare pentru a supraunitar și descrescătoare convexă pentru a subunitar

3. Rf )2

,0(: , tgxxf )( convexă, crescătoare

4. n xxfRf )(,),0[: , concavă, crescătoare

5. ]1,1[],0[: f ,f(x)=sinx ,concavă, crescătoare pe ]2

,0[ si descrescătoare pe cealaltă

jumătate de domeniu 6. xxfRf alog)(,),0(: convexă, descrescătoare pentru a subunitar și concavă crescătoare pentru a supraunitar

7. xxff arcsin)(],2

,2

[]1,1[: ,concavă, crescătoare pe [-1,0] și convexă, crescătoare

pe [0,1]

8. *,1)(,),0(: Nnx

xfRf n , convexă, descrescătoare.

Toate aceste funcții pot fi analizate prin lectura grafica, graficele lor fiind studiate la clasă. TEOREMA 7. Dacă RIgf :, sunt convexe iar g este crescătoare, atunci compunerea lor ℎ: 퐼 → ℝ,ℎ = 푔 ∘ 푓 este convexă. Demonstrație:

)2

(2))2

((2)2

)()((2))(())(()()( yxhyxfgyfxfgyfgxfgyhxh

.

Următoarele propoziții nu le vom mai demonstra, ele fiind un bun exercițiu la cercul de mate sau ca muncă individuală. PROPOZIȚIA 8.Dacă f este convexă iar g este concavă descrescătoare, atunci ℎ = 푔 ∘ 푓 este concavă. PROPOZIȚIA 9.Dacă f și g sunt concave iar g este crescătoare, atunci ℎ = 푔 ∘ 푓 este concavă. PROPOZIȚIA 10. Dacă f este concavă, g este convexă descrescătoare atunci ℎ = 푔 ∘ 푓 este convexă. PROPOZIȚIA 11.Dacă ],[],[: dcbaf este bijectivă, atunci:

a) f convexă, crescătoare implică f-1 concavă, crescătoare b) f concavă, crescătoare implică f-1 convexă, crescătoare.

OBSERVAȚIA 12.Pentru demonstrații și alte rezultate asemănătoare se poate consulta [1]. 4 . APLICAȚII PROBLEMA 1. Într-un triunghi ABC are loc inegalitatea

2

33sinsinsin CBA .

Soluție. Folosim funcția din exemplul 5 și aplicăm inegalitatea lui Jensen pentru n=3:

23

3sin

3sinsinsin

CBACBA .

PROBLEMA 2. Dacă 0,...,, 21 naaa și au suma n, atunci

21

1...1

1

1

naa n

.

Page 28: RMM 14.pdf

S.S.M.ROMÂNIA - Filiala Mehedinți 2014

28 REVISTA MEHEDINȚEANĂ DE MATEMATICĂ NR. 14

Soluție.x

xfRf

1

1)(,),1(: ,este convexă, fiind o translație a exemplului 8.

)(...)( 1 nafaf2...1

11

1...1

111

n

naan

aa nn

PROBLEMA 3.Fie nixRxx in ,1,1,,...,1 . Să se arate că

n

nn xxn

xx ...111...

11

11

.

IMO,Shortlist Soluție. Conform Teoremei 6 este suficient să demonstrăm pentru n=2. Funcția este

xxfRf

11)(,),1(: , și o aplicăm numerelor a2, b2 mai mari sau egale cu 1:

0)1()(1

21

11

1 222

abba

abba

Evident adevărată.

PROBLEMA 4.Fie Rcba ,, . Arătați că 1888 222

abc

cacb

bbca

a .

IMO,2001

Soluție. Asumăm fără a restrânge generalitatea că a+b+c=1, folosim x

xf 1)( , convexă

(compuneri între exemplele 4 si 8). Din inegalitatea lui Jensen: af(a2+8bc)+bf(b2+8ca)+cf(c2+8ab)f(M) unde M= 32 24)8( aabcbcaa

Rămâne de arătat că f(M)1 sau că 1M sau că 0)()(24 233 bacaaabc . Aceasta este evidentă. Egalitatea are loc pentru a=b=c. PROBLEMA 5.Dacă triunghiul ABC este ascuțitunghic, atunci

sin(cosA)+sin(cosB)+sin(cosC)<23

Soluție. Funcția f(x)=cosx este concavă pe [0,] iar g(x)=sinx este concavă, descrescătoare; deci ℎ = 푔 ∘ 푓 este concavă. Avem:

23

6sin3

21sin3)

3(3)

3(3)()()(

hCBAhChBhAh .

PROBLEMA 6. Dacă triunghiul ABC este ascuțitunghic, atunci:

8

332

cos2

cos2

cos CBA .

Soluție.h(x)=g(f(x))=lncosx

.8

33ln3

cosln32

cosln2

cosln2

cosln

CBACBA

PROBLEMA 7. În triunghiul ascuțitunghic ABC are loc inegalitatea

81

2sin

2sin

2sin

CBA .

Indicație h(x)=lnsinx este concavă. PROBLEMA 8. În triunghiul ascuțitunghic ABC are loc inegalitatea

Page 29: RMM 14.pdf

S.S.M.ROMÂNIA - Filiala Mehedinți 2014

29 REVISTA MEHEDINȚEANĂ DE MATEMATICĂ NR. 14

tg2A+tg2B+tg2C9. Indicație.h(x)=tg2x este convexă.(f(x)=tgx si g(x)=x2 ). PROBLEMA 9. Cu notațiile obișnuite, în orice triunghi avem: rhhh cba 9 .

Indicație.x

xf 1)( este convexă. Notăm

cba hx

hx

hx

cbacq

cbabq

cbaaq 1,1,1.,,, 321321

.

Bibliografie: [1]PantelimonG.Popescu, I.V.Maftei, Jose Luis Diaz-Barrero, Mariana Dinca – “Inegalitati Matematice - Modele inovatoare”, EDP-2007 [2] Phan Kim Hung - “Secrets in inequalities”, vol.1, Basic Inequalities, Ed.Gil 2007 [3] M.Becheanu, B.Enescu - “Inegalitati elementare si mai putin elementare”, Ed. Gil 2002

Asupra numărului de permutări fără puncte fixe

Prof. Daniel Sitaru, Colegiul Naţional Economic „Theodor Costescu”

Definiție: Fie permutarea :

푝: {1,2, …푛} → {1,2, …푛};푛 ∈ 푁∗ ;푝 = 1 2 …푝(1) 푝(2) …

푛푝(푛)

Spunem că permutarea 푝 admite o coincidență în 푖 ; 푖 ∈ {1,2, …푛} dacă: 푝(푖) = 푖 . Numărul 푖 cu această proprietate se numește punct fix al permutării 푝. Prin !푛-(left factorial) vom nota numărul de permutări fără puncte fixe al mulțimii {1,2, … 푛} . Prima demonstrație a faptului că:

!푛 = 푛! ∙(−1)푘!

a fost dată în 1713 de P.R.de Montmort și găsită independent de L.Euler în 1753. În cele ce urmează prezint o demonstrație a acestei relații bazată pe principiul includerii și excluderii urmată de câteva proprietăți și aplicații. Proprietatea 1:

!푛 = 푛! ∙(−1)푘!

Demonstrație: Fie 퐴 mulțimea celor (푛 − 1)! permutări care admit un punct fix în 푖. Din principiul includerii și excluderii avem:

Page 30: RMM 14.pdf

S.S.M.ROMÂNIA - Filiala Mehedinți 2014

30 REVISTA MEHEDINȚEANĂ DE MATEMATICĂ NR. 14

|퐴 ∪ 퐴 ∪ …∪ 퐴 | = |퐴 |− 퐴 ∩ 퐴 + ⋯+ (−1) 퐴

Pe de altă parte: 퐴 ∩ 퐴 ∩ …∩ 퐴 = (푛 − 푘)!

deoarece o permutare din mulțimea: 퐴 ∩ 퐴 ∩ …∩ 퐴

prezintă în pozițiile 푖 , 푖 , … , 푖 puncte fixe în număr de (푛 − 푘)! . Astfel 푘 poziții 푖 , 푖 , … , 푖 pot fi alese din mulțimea celor 푛 poziții în 퐶 moduri , deci :

퐴 = 퐶 (푛 − 1)!− 퐶 (푛 − 2)! + ⋯+ (−1) 퐶

Numărul permutărilor fără puncte fixe se obține scăzând din 푛! −numărul tuturor permutărilor, numărul permutărilor care admit măcar un punct fix:

!푛 = 푛!− 퐶 (푛 − 1)!− 퐶 (푛 − 2)! + ⋯ (−1) 퐶 (푛 − 푘)! … + (−1) 퐶

!푛 = 푛! 1 −11! +

12! −

13! + ⋯+

(−1)푘! + ⋯

(−1)푛!

!푛 = 푛! ∙(−1)푘!

Proprietatea 2: Pentru orice 푛 ∈ 푁∗ avem:

!푛 = 푛 ∙ ! (푛 − 1) + (−1) Demonstrație:

푛 ∙ ! (푛 − 1) + (−1) = 푛(푛 − 1)! ∙(−1)푘! + (−1) =

= 푛(푛 − 1)! ∙(−1)푘! + (−1) = 푛!

(−1)푘! +

(−1)푛! =

= 푛! ∙(−1)푘! = !푛

Proprietatea 3: Pentru orice 푛 ∈ 푁 ;푛 ≥ 2 avem:

!푛 = (푛 − 1) ∙ [! (푛 − 1) + ! (푛 − 2)] Demonstrație:

(푛 − 1) ∙ [! (푛 − 1) + ! (푛 − 2)] = (푛 − 1) ∙ ! (푛 − 1) + (푛 − 1) ∙ ! (푛 − 2) =

= (푛 − 1) ∙ (푛 − 1)! ∙(−1)푘!

+ (푛 − 1) ∙ (푛 − 2)! ∙(−1)푘!

=

= 푛! ∙ (−1)푘! − (푛 − 1)! ∙

(−1)(푛 − 1)! = 푛! ∙

(−1)푘! + (−1) =

= 푛! ∙(−1)푘! +

(−1)푛! = 푛! ∙

(−1)푘! = !푛

Page 31: RMM 14.pdf

S.S.M.ROMÂNIA - Filiala Mehedinți 2014

31 REVISTA MEHEDINȚEANĂ DE MATEMATICĂ NR. 14

Valorile lui !푛 (numite și numărul de deranjamente al unei permutări cu "푛" elemente sau numerele lui Montmart) pentru 푛 ∈ {1,2,3, … ,10} sunt:

! 1 = 0 ; ! 2 = 1 ; ! 3 = 2 ; ! 4 = 9 ; ! 5 = 44 ; ! 6 = 265 ; ! 7 = 1854 ; ! 8 = 14833 ; ! 9 = 133496 ; ! 10 = 1334961

Aplicația 1: În câte moduri pot fi aranjate pe un raft patru cărți în așa fel încât nici o carte să nu își păstreze locul inițial? Soluție:! 4 = 9 Aplicația 2: În câte moduri pot fi dispuși cinci operatori pe o linie de asamblare astfel încât nici un operator să nu repete operația efectuată? Soluție: ! 5 = 44 Bibliografie: 1. N.Teodorescu, V.Mangu, A.Negru, C.Cărbunaru, M.Trifu: “Matematica

în gimnaziu și liceu”-EDP-Bucuresti-1989 R.L.Graham, D.E.Knuth, O.Patashnik: “Concrete Mathematics”-Addison-Wesley, Reading,MA,USA-1994

Temă pentru grupa de performanță la clasa a- XI -a

Calculul limitelor unei clase de șiruri cu ajutorul limitelor de funcții

Prof. Dr. Stretcu Daniel,Colegiul Național ”Gheorghe Țițeica”

O clasă specială de șiruri se rezolvă cu ajutorul următoarei proprietăți: PROPRIETATE: Fie 푓,푔 ∶ (푎,푏)\{0} → ℝ ,푎 < 0 < 푏 , 푔(푥) > 0 ,∀푥 ∈ (푎,푏)\ {0 }ș푖 lim

( )( )

= 1..

Dacă 푎 , este un șir, 푎 ≠ 0 și limn→∞

푎 , = 0 ,∀푘 ≤ 푛, 푘,푛 ∈ ℕ,

atunci limn→∞

∑ 푓 푎 , = limn→∞

∑ 푔 푎 , ,dacă ultima limită există.

DEMONSTRAȚIE: Folosim proprietatea inegalitaților: Dacă 푚 < < 푀 , 푦 > 0 , 푘 = 1,푛 atunci 푚 < ⋯

⋯< 푀 (1)

Din lim→

( )( )

= 1 și limn→∞

푎 , = 0 , folosind definiția limitei unei funcții rezultă:

1 − 휀 < ,

,< 1 + 휀 ,∀휀 > 0 (2)

Folosind (1) și (2) , obținem : 1 − 휀 < ∑ ,∑ ,

< 1 + 휀 ,∀푛 > 푀 (휀)

De unde rezultă proprietatea cerută. Exemplu rezolvat:

Să se calculeze: limn→∞

∑ 푠푖푛

Page 32: RMM 14.pdf

S.S.M.ROMÂNIA - Filiala Mehedinți 2014

32 REVISTA MEHEDINȚEANĂ DE MATEMATICĂ NR. 14

Soluție: Fie 푎 , = şi 푓:ℝ∗ → ℝ,푓( ) = 푥 , 푔: ℝ∗ → ℝ,푔( ) = 푥

Avem: lim→

( )

( ) = 1şi lim

n→∞푎 , = 0.

Aplicând proprietatea demonstrată obţinem : limn→∞

∑ 푠푖푛 = limn→∞

∑ 푓(푎 , ) =

limn→∞

∑ 푔(푎 , ) = limn→∞

∑ = limn→∞

∑ 푘 = limn→∞

( ) = .

Exerciţii propuse: Să se calculeze următoarele limite:

1) limn→∞

∑ 푠푖푛 ;

2) limn→∞

∑ 푠푖푛 ;

3) limn→∞

∑ 푠푖푛 ;

4) limn→∞

∑ 푡푔 ;

5) limn→∞

∑ 푡푔 ;

6) limn→∞

∑ 푎푟푐푠푖푛

7) limn→∞

∑ 푎푟푐푡푔

8) limn→∞

∑ sin 1

n2+knk=1 ;

9) limn→∞

∑ tg 1

n2+knk=1 ;

10) limn→∞

∑ arcsin 1

n2+knk=1 ;

11) limn→∞

∑ arctg 2

n2+knk=1 ;

12) limn→∞

∑ (푒 -1)nk=1 ;

Observație: Pentru exercițiile 14-21, se transformă produsele în sume folosind: 푎 = 푒 și se utilizează :lim

x→0

( ) = 1

Bibliografie: Revista Preuniversitară, București , 1990, pg. 46-47 ,autor Conf. dr. I. Bacalu

Page 33: RMM 14.pdf

S.S.M.ROMÂNIA - Filiala Mehedinți 2014

33 REVISTA MEHEDINȚEANĂ DE MATEMATICĂ NR. 14

Puncte laticiale în plan și spațiu

Prof. Daniel Sitaru, Colegiul Naţional Economic „Theodor Costescu”

Definiție: Se numește punct laticial în plan orice punct care are ambele coordonate exprimate prin numere întregi. Analog, se definesc punctele laticiale din spațiul tridimensional. Notăm: 퐿 = {(푥, 푦)|푥, 푦 ∈ 푍} ; 푆 = {(푥,푦, 푧)|푥, 푦, 푧 ∈ 푍} . Proprietatea 1: Orice triunghi din plan cu toate vârfurile puncte laticiale are dublul ariei număr întreg. Demonstrație : Dacă 퐴(푥 ,푦 ); 퐵(푥 ,푦 ); 퐶(푥 ,푦 ) ∈ 퐿 ; 퐴[퐴퐵퐶] = |∆| ;

∆=푥 푦 1푥 푦 1푥 푦 1

; 2퐴[퐴퐵퐶] = |∆| ∈ 푍

Proprietatea 2: Nu există nici un octogon regulat în plan, cu latura 푙 ∈ 푄 cu toate vârfurile puncte laticiale. Demonstrație :

퐴[퐴 퐴 …퐴 ] = 8 ∙ 퐴[푂퐴 퐴 ] = 8 ∙12 ∙ 푠푖푛

휋4 ∙

푙4푠푖푛

=

= 8 ∙푙4 ∙ 푐푡푔

휋8 = 2 ∙ 푙 (√2 − 1)

푙 ∈ 푄 ⟹ 퐴[퐴 퐴 …퐴 ] ∈ 푄 ⟹ √2 − 1 ∈ 푄 . FALS! Proprietatea 3: Fie 퐷 mulțimea formată din toate dreptele din plan care trec prin cel puțin două puncte din mulțimea 퐿. Pe orice dreaptă din mulțimea 퐷 se află o infinitate de puncte din mulțimea 퐿. Demonstrație : Dacă 푑 ∈ 퐷 trece prin 퐴(푥 ,푦 ) și (푥 ,푦 ) , atunci 퐶(2푥 − 푥 , 2푦 − 푦 ) simetricul lui 퐴 față de 퐵 se află pe dreapta 푑. În acest fel pe 푑 există o infinitate de puncte din mulțimea 퐿. ( raționamentul se repetă cu simetricul lui 퐵 față de 퐶... ) Proprietatea 4: Orice punct din plan care are ambele coordonate numere raționale se află pe o dreaptă din mulțimea 퐷. Demonstrație : Fie 푀 , punct cu ambele coordonate raționale (dacă coordonatele nu au același numitor, găsim numitorul comun și îl notăm cu “n”). Punctul 푀 se află pe dreapta 푑:푦 = 푥 care trece prin 푂(0,0) și 퐴(푝, 푞) deci este din mulțimea 퐷. Proprietatea 5: Există puncte în plan care nu se află pe nici o dreaptă din mulțimea 퐷. Demonstrație : Fie 퐴(푥 ,푦 ); 퐵(푥 ,푦 ) puncte de pe o dreaptă 푑 ∈ 퐷 ; 푥 ,푥 , 푦 , 푦 ∈ 푍 ; 푥 ≠ 푥 sau 푦 ≠ 푦 . Fie 푃(√5,√7). Daca 퐴,퐵,푃 coliniare atunci:

Page 34: RMM 14.pdf

S.S.M.ROMÂNIA - Filiala Mehedinți 2014

34 REVISTA MEHEDINȚEANĂ DE MATEMATICĂ NR. 14

푥 푦 1푥 푦 1√5 √7 1

= 0, de unde:

푥 푦 − 푥 푦 + √5(푦 − 푦 ) + √7(푥 − 푥 ) = 0, 푦 − 푦 = 0 ;푥 − 푥 = 0. FALS! Punctul 푃 verifică proprietatea. Proprietatea 6: Pentru orice 푛 ∈ 푁∗ există în plan un cerc care conține în interiorul său exact "푛" puncte laticiale. Demonstrație : Fie (푎, 푏); 푁(푐, 푑); 푀,푁 ∈ 퐿 ;푀 ≠ 푁 . Arătăm că 푀,푁 nu se pot afla la aceeași distanță de punctul (√3 , ) . Presupunem prin absurd că 푀푃 = 푁푃 . Rezultă:

(푎 − √3) + (푏 − ) = (푐 − √3) + (푑 − ) ,

푎 + 푏 − − 푐 − 푑 + − 2√3(푎 − 푐) = 0 , 푎 − 푐 = 0

푎 + 푏 − − 푐 − 푑 + = 0 ,

(푏 − 푐 ) − (푏 − 푑) = 0 ,

(푏 − 푑) 푏 + 푑 − = 0 ,

Dar +푑 ≠ ; 푏,푑 ∈ 푍 ⟹ 푏 = 푑 . FALS! deoarece 푀 ≠ 푁. Fie 푀 punctul laticial cel mai apropiat de 푃, 푀 următorul (ș.a.m.d.) . Din raționamentul de mai sus 푀 ≠ 푀 ≠ 푀 ≠ ⋯ ≠ 푀 ≠ 푀 ≠ ⋯. Cercul cu centrul în punctul 푃 și de rază cuprinsă strict între distanțele de la 푃 la 푀 și 푀 conține în interiorul său exact punctele 푀 ,푀 , … ,푀 . Proprietatea 7: Pentru orice 푛 ∈ 푁∗ există în spatiul tridimensional o sferă care conține în interiorul său exact "푛" puncte laticiale. Demonstrație : Fie (푎, 푏, 푐); 푁(푑, 푒, 푓); 푀,푁 ∈ 푆 ;푀 ≠ 푁 . Arătăm că 푀,푁 nu se pot afla la aceeași distanță de punctul (√3,√5 , ) . Presupunem prin absurd că 푀푃 = 푁푃 . Rezultă: (푎 − √3) + (푏 − √5) + (푐 − ) = (푑 − √3) + (푒 − √5) + (푓 − ) ,

푎 + 푏 + 푐 − 푑 − 푒 − 푓 − + − 2√3(푎 − 푑)− 2√5(푏 − 푒) = 0 , 푎 − 푑 = 0푏 − 푒 = 0

푎 + 푏 + 푐 − 푑 − 푒 − 푓 − + = 0,

푎 = 푑 ,푏 = 푒 , (푐 − 푓) 푐 + 푓 − = 0,

Dar 푐 + 푓 ≠ pentru că 푐, 푓 ∈ 푍. Rezultă : 푐 = 푓,푀 = 푁, FALS! deoarece 푀 ≠ 푁. Fie 푀 punctul laticial cel mai apropiat de 푃, 푀 următorul (ș.a.m.d.) . Din raționamentul de mai sus 푀 ≠ 푀 ≠ 푀 ≠ ⋯ ≠ 푀 ≠ 푀 ≠ ⋯. Sfera cu centrul în punctul 푃 și de rază cuprinsă strict între distanțele de la 푃 la 푀 și 푀 conține în interiorul său exact punctele 푀 ,푀 , … ,푀 .

Page 35: RMM 14.pdf

S.S.M.ROMÂNIA - Filiala Mehedinți 2014

35 REVISTA MEHEDINȚEANĂ DE MATEMATICĂ NR. 14

Bibliografie: 1.Daniel Sitaru,Claudia Nănuți - Probleme de concurs – Editura Ecko-Print, Drobeta Turnu Severin-2011 2.Daniel Sitaru,Claudia Nănuți - Matematici pentru olimpiade – Editura Ecko-Print, Drobeta Turnu Severin-2014 3. Dumitru Bușneag, Ioan Maftei - Teme pentru cercurile și concursurile de matematică ale elevilor - Editura Scrisul românesc-Craiova-1983

Temă pentru grupa de performanță la clasa a- XII –a Teoremele lui Lagrange și Cauchy în algebra superioară

Prof. Dana Heuberger, Colegiul Naţional „Gheorghe Şincai”- Baia Mare

Abstract: In this paper we’ll solve some interesting contest problems in the group theory, using Lagrange’s theorem and Cauchy’s theorem.

În acest articol vom evidenţia rolul teoremelor lui Lagrange şi Cauchy, două dintre cele mai cunoscute teoreme de teoria grupurilor, în problemele de concurs cu structuri algebrice. Cu ajutorul lor vom afla mai multe informaţii despre ordinele elementelor şi subgrupurilor unui grup oarecare, nu neapărat comutativ şi le vom aplica apoi în probleme de concurs. 1.1. Definiţie. Fie H un subgrup al grupului ,G . Definim relaţiile de echivalenţă ,H H (la stânga, respectiv la dreapta) pe G , astfel:

1Hx y x y H şi 1

Hx y yx H . Mulţimile xH şi Hx se numesc clasa de echivalenţă la stânga, respectiv la dreapta a lui x în raport cu H , deoarece: 1, Hy xH h H y xh x y H x y şi 1, Hy Hx h H y hx yx H x y Mulţimile / , , ,HG H xH yH şi / , , ,HG H Hx Hy se numesc mulţimile claselor de echivalenţă la stânga, respectiv la dreapta în raport cu H . Mulţimea I G se numeşte sistem de reprezentanţi pentru H dacă I este formată din câte un reprezentant al fiecărei clase de echivalenţă. 1.2. Observaţii. 1) Ca de obicei, dacă ,G este un semigrup (adică operaţia este o lege de compoziţie asociativă pe G ), H este o submulţime a lui G şi a G , notăm

aH ah h H G şi Ha ha h H G .

2) Funcţia : / /H Hf G G , 1f xH Hx este bijectivă, iar / / :not

H HG G G H şi se numeşte indicele subgrupului H în grupul G . 3) xH yH xH yH .

Page 36: RMM 14.pdf

S.S.M.ROMÂNIA - Filiala Mehedinți 2014

36 REVISTA MEHEDINȚEANĂ DE MATEMATICĂ NR. 14

Într-adevăr, dacă a xH yH , atunci 1 2,h h H , astfel încât 1 2a xh yh . Atunci, 1

2 1x yh h şi cum 12 1h h H , rezultă că x yH , aşadar xH yH .

Incluziunea cealaltă se demonstrează analog. 1.3. Teorema lui Lagrange. Fie ,G un grup finit şi H un subgrup al lui G . Atunci: a) ord ord ;H G

b) ord ord ord / HG H G . cu altă notaţie, :G H G H

Demonstraţie: Fie H relaţia de echivalenţă la stânga din definiţia 1.1. şi I un sistem de reprezentanţi pentru H . Conform observaţiei 1.2., mulţimea claselor de echivalenţă la stânga în raport cu H este o partiţie a mulţimii G , adică

x IxH G

şi clasele sunt disjuncte două

câte două. Mai mult, funcţia :f H xH , f h xh este bijectivă.

Rezultă că ,x y G , xH yH H . Aşadar, ord ord ord / HG H G , unde / HG e mulţimea claselor de echivalenţă modulo H , numită şi mulţime cât a lui G în raport cu relaţia de echivalenţă H . 1.4. Consecinţe.Fie ,G un grup finit şi H un subgrup al lui G . Atunci: 1) pentru orice x G , ;xH H 2) pentru orice x G , ord ord ;x G

3) pentru orice x G , ord Gx e , unde e G este elementul neutru al grupului; 4) orice grup de ordinul 푝 ∈ ℕ, cu p prim, este ciclic, deci este izomorf cu ℤ , + . 1.5. Teorema lui Cauchy. Fie ,G un grup finit cu elementul neutru e G şi p un

număr prim, ordp G . Atunci, numărul soluţiilor ecuaţiei px e este un multiplu nenul al lui p . O demonstraţie elementară a teoremei lui Cauchy se poate studia în lucrarea [1]. 1.6. Consecinţă.Dacă ,G este un grup finit şi p este un număr prim, cu ordp G , atunci există x G , astfel încât ord x p . 1.7. Consecinţă.În condiţiile consecinţei 1.6., numărul subgrupurilor de ordin p ale lui G este congruent cu 1 mod p . Demonstraţie: Din teorema lui Cauchy, există elemente de ordinul p ale grupului G . Notăm cu 푘 ∈ ℕ∗ numărul subgrupurilor de ordinul p ale lui G . Numărul p fiind prim, acestea sunt ciclice. Avem i iH x , 1,i k şi i jH H e , , 1, 2, , ,i j k i j .

Fie H mulţimea soluţiilor ecuaţiei px e . Rezultă 1 2 kH H H H , deci 1 2ord ord 1 1kH H H H k p .

Din teorema lui Cauchy, deoarece numărul soluţiilor ecuaţiei px e este un multiplu nenul al lui p , rezultă 1 1 0 modk p p , deci 1 modk p . 1.8. Observaţii. 1. Numărul elementelor de ordin diferit de 2 ale unui grup finit este impar.

Page 37: RMM 14.pdf

S.S.M.ROMÂNIA - Filiala Mehedinți 2014

37 REVISTA MEHEDINȚEANĂ DE MATEMATICĂ NR. 14

Într-adevăr, elementele de ordin 3 ale grupului se cuplează în perechi 1,g g , deci sunt în

număr par. Lor li se mai adaugă elementul neutru, care este de ordin 1. 2. În orice grup finit de ordin par, există elemente de ordin 2. Pentru argumentare, se poate folosi observaţia anterioară sau consecinţa 1.6. 1.9. Propoziţie. Fie ,G un grup finit, astfel încât x G , 2x e . Atunci: a) ,G este grup comutativ;

b) există 푘 ∈ ℕ, astfel încât ord 2kG .

Demonstraţie: Prima parte a propoziţiei este un exerciţiu foarte cunoscut, prezent în manuale, care este adevărat şi pentru cazul în care grupul G nu este finit. Presupunem că ordinul grupului G nu este o putere a lui 2. Atunci ∃푝 ∈ ℕ un număr prim, 3p , astfel încât ordp G . Atunci, din consecinţa 1 a

teoremei lui Cauchy, rezultă că există a G cu ord a p . Avem pa e şi cum 2a e ,

obţinem , 2 1pa a e , fals. Aşadar ∃푛 ∈ ℕ astfel încât ord 2nG . 1.10. Generalizare. Fie ,G un grup finit şi 푝 ∈ ℕ un număr prim astfel încât

x G , px e . Atunci ordinul lui G este o putere a lui p . Iată acum câteva probleme interesante care ilustrează rezultatele anterioare:

E.1.Fie ,G un grup finit şi 1 2,H H subgrupuri ale lui G . Fie mulţimea 1 2 1 2,H H x y x H y H .

Dacă 1 21max ord , ord ord2

H H G , demonstraţi că 1 2H H G .

Soluţie: Presupunem că 1 2ord ordH H , deci 1ord2nH , unde ord G n .

Atunci, deoarece din teorema lui Lagrange avem că 1ord ordH G , rezultă că 1ord H n , deci 1H G . Aşadar 1 2 2 2,H H GH x y x G y H . Elementul neutru e al grupului G se află şi în 2 ,H deci, x G , 2x x e GH , aşadar 2G GH şi cum şi 2GH G , rezultă că 2 1 2G GH H H . E.2.Fie ,G un grup cu n elemente, 푝 ∈ ℕ, 푝 ≥ 2 şi mulţimea pH x x G G .

Arătaţi că , 1p n H G . Soluţie: „” Presupunem că există , ,x y G x y , astfel încât p px y . Din , 1p n rezultă că există ℎ,푘 ∈ ℕ, astfel încât 1p h n k .

Atunci, deoarece ord Gx e , obţinem: hph nk ph nk ph px x x x x x .

Analog rezultă hpy y şi cum p px y , deducem că x y , fals.

Aşadar H este o submulţime cu n elemente a grupului G . Cum G are n elemente, rezultă H G . „” Deoarece mulţimea G este finită, afirmaţia H G este echivalentă cu faptul că funcţia

:f G G , pf x x este bijectivă. Presupunem că , 1p n . Atunci, există numărul prim q astfel încât ,q p n .

Page 38: RMM 14.pdf

S.S.M.ROMÂNIA - Filiala Mehedinți 2014

38 REVISTA MEHEDINȚEANĂ DE MATEMATICĂ NR. 14

Din teorema lui Cauchy, există x G cu ord x q . Atunci, deoarece p q t , cu 푡 ∈ ℕ ,

avem tp qf x x x e f e şi cum f este injectivă, rezultă că x e ,

contradicţie cu faptul că x are ordinul q . Rezultă că , 1p n . Observaţie. Problema se poate reformula astfel: Fie ,G un grup cu n elemente, 푝 ∈ ℕ, 푝 ≥ 2 şi funcţia :f G G , pf x x . Atunci, , 1p n f este bijectivă. E.3. Fie n un număr natural liber de pătrate şi G un grup neabelian de ordinul n . Demonstraţi că există două elemente din G , de acelaşi ordin, care nu comută.

Marian Andronache Soluţie: Presupunem că orice două elemente ale grupului care au acelaşi ordin comută. Pentru fiecare factor prim p al lui n notăm p

pH x G x e . Deoarece am presupus că , ,pu v H uv vu , rezultă că pH este închisă faţă de operaţia grupului. Deoarece pH este finită, rezultă că pH este un subgrup abelian al lui G . Fie ,p q

doi factori primi distincţi ai lui n şi fie px H , qy H . Notăm 1 1a xyx y .

Avem 1 1 1q qxyx xy x xx e , deci 1qxyx H şi cum 1

qy H , rezultă că qa H . Analog arătăm că pa H . Aşadar p qa H H e , deci xy yx . Fie 1 2 kn p p p , unde 1 2, , , kp p p sunt factorii primi distincţi ai lui n . Din teorema lui Cauchy rezultă că există 1 2, , , ka a a G care au respectiv ordinele

1 2, , , kp p p . Deoarece elementele 1 2, , , ka a a comută două câte două, rezultă că 1 2ord ka a a n , deci grupul G este ciclic. Cum orice grup ciclic este comutativ, aceasta

contrazice ipoteza. Aşadar există două elemente de acelaşi ordin ale grupului care nu comută. E.4. Fie 푚 ∈ ℕ,푚 > 2 şi G un grup finit astfel încât \x G e , ord x m . Arătaţi că G nu se poate scrie ca o reuniune de m subgrupuri proprii ale sale. Soluţie: Să presupunem că există subgrupurile proprii ale lui G , 1 2, , , mH H H , astfel încât 1 2 mG H H H . Deoarece există i ix H cu ord ix m , rezultă că pentru orice 1,i m avem iH m . Fie :i it G H indicele lui iH în G , adică numărul claselor de echivalenţă (la stânga) determinate de subgrupul iH . Deoarece iH G , avem că 1it ,

1,i m . Pe de altă parte, deoarece elementul neutru aparţine oricărui subgrup, avem:

1 2 mG H H H . Fie 1 2min , , , mt t t t . Deoarece 1t , putem alege un divizor prim p al său. Din teorema lui Lagrange, ordp G şi din teorema lui Cauchy, există g G cu ord g p . Din ipoteză rezultă că p m , deci 1,i m , it m . Din teorema lui Lagrange ştim că 1,i m , i iG t H .

Rezultă: 1 21

1 1 11 1mm

H H Ht t G

, contradicţie.

E.5. Fie G un grup finit cu elementul neutru e .

Page 39: RMM 14.pdf

S.S.M.ROMÂNIA - Filiala Mehedinți 2014

39 REVISTA MEHEDINȚEANĂ DE MATEMATICĂ NR. 14

a) Arătaţi că dacă grupulG este abelian şi are un număr impar de elemente, atunci produsul tuturor elementelor sale este egal cu e . b) Arătaţi că dacă pentru 푛 ∈ ℕ grupul G este abelian şi are 4 2n elemente, atunci produsul tuturor elementelor sale este diferit de e .

Ion Savu Soluţie: a) Deoarece ordinul oricărui element al grupului divide ordinul grupului, care este impar, rezultă că a G , 2a e , deci \a G e , 1a a . Împărţim elementele diferite

de e ale grupului în perechi disjuncte de tipul 1,a a . Deoarece grupul este comutativ, obţinem că produsul tuturor elementelor sale este egal cu e . b) Fie 1 2 4 1, , , , nG e x x x . Presupunem că produsul tuturor elementelor sale este egal cu e . Din teorema lui Cauchy, există un element \a G e de ordinul 2. Împărţim cele 4 2n elemente ale lui G în perechile disjuncte ,e ae , 1 1 2 2, , , ,n nx ax x ax . Alegem câte un element al fiecăreia dintre perechile anterioare şi obţinem astfel o mulţime A G cu 2 1n elemente. Avem \G A a A .

Obţinem: 2 1 2n

x G x A x A x A x A x A

e x x x x ax a x

, deci 2 2

x A

a x y

, unde am

notat cu x A

y x

. Fie = ord 4 2 ordd y n G .

Dacă d este impar, atunci 2d da y e , contradicţie cu ord 2a . Dacă 2d l , cu l impar, atunci 2l l da y y e , contradicţie cu ord 2a . Aşadar presupunerea făcută este falsă, de unde rezultă concluzia. E.6. Fie ,G un grup de ordinul G p q , cu ,p q numere prime, p q . Arătaţi că nu există subgrupuri distincte de ordinul q ale lui G . Soluţie:Deoarece q este prim, din teorema lui Cauchy rezultă că există cel puţin un subgrup de ordinul q al lui G . Presupunem că ar exista două subgrupuri distincte de ordinul q ale lui G . Deoarece q este prim, acestea sunt ciclice: 2 1

1 , , , , qG e a a a ,

2 12 , , , , qG e b b b . Presupunem că există 1, 1k q astfel încât 2

ka G . Atunci avem

2ka G . Cum ord ka q , rezultă că 1 2

ka G G şi deoarece 1G şi 2G au acelaşi

număr de elemente, obţinem că 1 2G G , fals. Aşadar 1 2G G e . Considerăm mulţimile 2 1

2 2 2 2, , , , qG a G a G a G . Presupunem că există , 1, 2, , 1k t q , k t şi 1 2 2,b b G cu 1 2

k ta b a b . Atunci 1

2 1 2k t k ta b b G a e q k t , fals.

Aşadar mulţimile anterioare sunt disjuncte două câte două, au câte q elemente şi sunt incluse în G , de unde: 2...

q ori

q q q q p q

, fals (din ipoteză avem că p q ).

Deducem că grupul G are un singur subgrup de ordinul q . E.7. Se consideră 푛 ∈ ℕ, 푛 ≥ 4 şi grupul ,G cu n elemente, având elementul neutru e . Fie subgrupurile 1H şi 2H ale lui G , astfel încât 1 2H H e .

Page 40: RMM 14.pdf

S.S.M.ROMÂNIA - Filiala Mehedinți 2014

40 REVISTA MEHEDINȚEANĂ DE MATEMATICĂ NR. 14

Dacă mulţimea 1 2H H are cel puţin 22n

elemente, demonstraţi că 1H G sau 2H G .

Dana Heuberger Soluţie: Presupunem că 1H G şi 2H G . Notăm 1H a şi 2H b .

Dacă 1a , atunci subgrupul 2 1 2H H H are cel puţin 22n

elemente şi din teorema lui

Lagrange rezultă că 2H G , contradicţie. Aşadar 2a . Analog rezultă că 2b .

Din ipoteză deducem: 1 2 1 2 12 2n nH H a b

, deci 22n a b 1

Cum 1 2H H e , notând \i iH H e , 1, 2i , obţinem: 1 2 1 1H H a b .

Mai mult, 1 2 1 2\H H G H H , deci 1 2 12nH H .

De aici, folosind şi relaţia 1 , deducem că 1 1 1 32na b a b ,

deci 2 4 0ab a b , adică 2 2 0a b , fals, deoarece 2a şi 2b . Aşadar 1H G sau 2H G . Observaţii:1. Problema anterioară este o generalizare a unei probleme a aceleiaşi autoare, apărută pe lista scurtă a Olimpiadei Naţionale din anul 2005.

2. Enunţul nu rămâne adevărat în situaţia în care în 1 2H H sunt cel puţin 12n

elemente,

deci nu se poate îmbunătăţi. Într-adevăr, pentru 6n , considerând grupul 3σ , σ, τ, ε, φ, θe cu ord σ ord τ ord ε 2 şi ord φ ord θ 3 , putem alege subgrupurile 1 , σH e ,

2 , φ, θH e , cu 1 2H H e şi 1 2 1 42nH H

.

E.8. Considerăm în grupul 999 ,S un subgrup abelian G de ordinul 1111. Arătaţi că există 1, 2, , 999i astfel încât G să avem i i .

Berkeley, Preliminary Exams Soluţie:Avem 11 101G şi folosind teorema lui Lagrange deducem că elementele grupului pot avea unul dintre ordinele 1, 11, 101 sau 1111. Presupunem că G nu este ciclic. Din teorema lui Cauchy rezultă că există ,x y G cu

ord 11, ord 101x y . Deoarece xy yx , rezultă că ord 11 101 1111x y , deci G este ciclic, contradicţie cu presupunerea făcută. Aşadar grupul G este ciclic, deci există σ G cu 2 1110, σ, σ , , σ σG e .

Arătăm că există 1, 2, , 999i pentru care σ i i . Considerăm descompunerea lui 999σ S în cicluri disjuncte de lungime 1 , 1 2σ kc c c . Avem: 1 2ord ord ord 999kc c c şi 1 2c.m.m.m.c. ord , ord , , ord 1111kc c c . Presupunem că 1, 2, ,i k , ord lungimea 1i ic c . Aşadar 1< ord 999ic şi ord 1111ic , deci 1, 2, ,i k , ord 11,101ic . Presupunem că există a cicluri de lungime 11 şi b cicluri de lungime 101.

Page 41: RMM 14.pdf

S.S.M.ROMÂNIA - Filiala Mehedinți 2014

41 REVISTA MEHEDINȚEANĂ DE MATEMATICĂ NR. 14

Obţinem 11 101 999a b , deci 101 999 11a , adică 999 11 101, 202, 303, 404, 505, 606, 707, 808, 909a ,

de unde rezultă că 푎 ∉ ℕ, fals. În concluzie, există cel puţin un ciclu de lungime 1, adică un punct fix al permutării σ , deci şi al permutărilor 2 1110, σ , , σe .

E.9. Dacă grupul necomutativ ,G are n elemente, atunci 4nZ G .

Ca de obicei, ,Z G g G x G gx xg este centrul grupului G .

Marian Andronache Demonstraţie: Fie \a G Z G şi C a g G ag ga comutatorul său. Ştim că Z G şi C a sunt subgrupuri ale lui G . Deoarece a Z G , avem C a G . Aşadar C a este un subgrup propriu al lui G . Din teorema lui Lagrange obţinem că

2G

C a . Deoarece a Z G şi a C a , avem Z G C a , deci Z G e un subgrup

propriu al lui C a .

Din teorema lui Lagrange deducem că 2 4

C a GZ G .

Observaţie. Există grupuri necomutative, cu n elemente, care au 4nZ G .

De exemplu, pentru 8n , cele două grupuri necomutative de ordinul 8 – cel diedral şi cel al cuaternionilor – verifică cerinţa.

E.10. Fie grupul ,G cu n elemente, unde푛 ∈ ℕ, 푛 ≥ 5. Dacă oricum am alege 32nk

elemente ale sale, printre acestea există două elemente din Z G , atunci grupul G este comutativ.

Dana Heuberger Soluţie: Fie \M G Z G . Presupunem că M şi M conţine cel puţin 1k elemente. Adăugând elementul neutru la 1k elemente ale lui M , obţinem k elemente ale grupului, printre care, din ipoteză, găsim două care fac parte din Z G . Aşadar mulţimea M conţine cel puţin un element din Z G , fals. În concluzie, mulţimea M are cel mult 2k elemente, deci mulţimea Z G are cel puţin 2n k elemente.

Dar 3 32 2n nk

, aşadar în Z G avem cel puţin 2 12nn k elemente.

Folosind E.9. obţinem: 2 14 2n nZ G n k şi apoi 1

4 2n n , deci 4n , fals.

Observaţie. Pentru 8n şi 4 82nk

, grupul cuaternionilor

1, 1, , , , , ,H i i j j k k , cu 2 2 2 1i j k , ij ji k , jk kj i şi ki ik j verifică ipoteza problemei, dar nu şi concluzia acesteia, deoarece 1,1Z H şi grupul nu este comutativ. Aşadar enunţul problemei nu poate fi îmbunătăţit.

Page 42: RMM 14.pdf

S.S.M.ROMÂNIA - Filiala Mehedinți 2014

42 REVISTA MEHEDINȚEANĂ DE MATEMATICĂ NR. 14

Bibliografie: [1] Heuberger D., Pop V.,Matematică de excelenţă, pentru concursuri,

olimpiade şi centre de excelenţă, Clasa a XII-a, volumul I: ALGEBRĂ, Editura Paralela 45, 2014

[2] Heuberger D., „Sisteme maximale de elemente ale unor grupuri finite”, Argument, nr. 12, 2010, Editura Ceconii, Baia Mare, 2010, pp. 27-31

[3] Ion I.D., Radu N, Algebră, Editura Didactică şi Pedagogică, Bucureşti, 1981 [4] Lăduncă L.G., Borne pentru matematicieni. Algebră-Analiză. Clasele IX-XII, Editura Taida, Iaşi, 2010 [5] Pop V. (coord), Băeţica C., Mincu D., Pletea A., Pop V. Roman M., Teme şi probleme pentru concursurile internaţionale studenţeşti de matematică, vol. I. Editura Studis, Iaşi, 2013

Transport de structură

Prof. Angela Niţoiu - Colegiul Tehnic “Decebal”

Scopul acestei teme este de a furniza o metodă mai simplă de a demonstra că o mulţime are structură de grup în raport cu o operaţie oarecare, decât metoda verificării efective a axiomelor din definiţie. Pentru aceasta, va fi suficient să identificăm o funcţie bijectivă convenabilă de la mulţimea respectivă la un grup cunoscut, datorită următorului rezultat: Teoremă: Fie (퐺,∗) un grup, M o mulţime nevidă şi 푓:퐺 → 푀o funcţie bijectivă. Atunci: a) există o unică lege ,,o ” pe M astfel încât ∀ 푥, 푦 ∈ 퐺, 푓(푥 ∗ 푦) = 푓(푥)표 푓(푦); b) (M, o) este un grup izomorf cu (G, ∗ ). ( Spunem că legea ,,o ” este obţinută prin transportul legii ,,∗” de la G la M, prin funcţia f). Demonstraţie: Fie 훼,훽 ∈ 푀. Definim 훼표훽 = 푓 푓 (훼) ∗ 푓 (훽) . 푓 (훼) ∈ 퐺, 푓 (훽) ∈ 퐺 ⇒ 푓 (훼) ∗ 푓 (훽) ∈ 퐺 ⇒ 훼표훽 ∈ 퐺, deci ,,o ” este lege de compoziţie. Asociativitatea: Fie 훼,훽,훾 ∈ 푀, (훼표훽)표훾 = 푓 푓 (훼표훽) ∗ 푓 (훾) = 푓 푓 푓 푓 (훼) ∗

푓 (훽) ∗ 푓 (훾) = 푓 푓 (훼) ∗ 푓 (훽) ∗ 푓 (훾)∗=

푎푠표푐. 푓 푓 (훼) ∗ (푓 (훽) ∗

푓 (훾)) = 푓 푓 (훼) ∗ 푓 푓 푓 (훽) ∗ 푓 (훾) = 푓(푓 (훼) ∗ 푓 (훽표훾)) =

훼표(훽표훾). Elementul neutru: Dacă 푒 ∈ 퐺 este elementul neutru, notăm 푒 = 푓(푒) ∈ 푀. Fie 훼 ∈푀.훼표푒 = 푓 푓 (훼) ∗ 푓 (푒 ) = 푓(푓 (훼) ∗ 푒) = 푓 푓 (훼) = 훼. Analog ⇒푒 표훼 = 훼 şi deci 푒 este elementul neutru al lui (M, o).

Page 43: RMM 14.pdf

S.S.M.ROMÂNIA - Filiala Mehedinți 2014

43 REVISTA MEHEDINȚEANĂ DE MATEMATICĂ NR. 14

Elemente simetrizabile: Fie 훼 ∈ 푀 ă

∃! 푥 ∈ 퐺, 푓(푥) = 훼.Notăm 훼 = 푓(푥 ), unde 푥 este simetricul din G al lui x. Avem: 훼표훼 = 푓 푓 (훼) ∗ 푓 (훼 ) = 푓(푥 ∗ 푥 ) = 푓(푒) = 푒 şi analog obţinem 훼 표훼 = 푒 , deci 훼 este simetricul din M al lui 훼. În concluzie, (M, o) este grup. Mai mult, ∀ 푥,푦 ∈ 퐺, 푓(푥)표푓(푦) = 푓 푓 푓(푥) ∗ 푓 푓(푦) = 푓(푥 ∗ 푦). Demonstrăm acum că ,,o ” este unica lege cu proprietatea din enunţ. Fie ,, ” o lege de compoziţie pe M astfel încât 푓(푥 ∗ 푦) = 푓(푥)푓(푦),∀푥, 푦 ∈ 퐺. Pentru 훼,훽 ∈ 푀,∃!푥, 푦 ∈ 퐺 astfel încât 푓(푥) = 훼,푓(푦) = 훽. Avem: 훼훽 = 푓(푥)푓(푦) = 푓(푥 ∗ 푦) = 푓 푓 (훼) ∗ 푓 (훽) = 훼표훽, deci legile coincid. Observaţii a) Dacă grupul (퐺,∗) este comutativ, atunci şi grupul (M, o)este comutativ. b)f este un izomorfism de la G la M. c) Dacă pentru grupul (퐺,∗) şi mulţimea nevidă M avem funcţia bijectivă 푔:푀 → 퐺, atunci există o unică lege ,,o ” pe M astfel încât ∀푥, 푦 ∈ 퐺,푔(푥표푦) = 푔(푥) ∗ 푔(푦), şi anume: ∀훼,훽 ∈ 푀,훼표훽 = 푔 (푔(훼) ∗ 푔(훽)) iar (M, o) este un grup izomorf cu grupul (퐺,∗). Exemple: 1) a) Să se arate că funcţia 푓: (−1,1) → 퐑, 푓(푥) = 푡푔 este bijectivă. b) Să se înzestreze mulţimea 퐺 = (−1,1) cu o structură de grup comutativ. Soluţie: a)f este strict crescătoare, deci injectivă. f e continuă pe (-1, 1) şi

lim↘

푓(푥) = −∞ , lim↗푓(푥) = ∞,

deci, f e şi surjectivă. b) Conform observaţiei c), considerând grupul comutativ (R, +) şi definind legea „o” astfel: 푥표푦 = 푎푟푐푡푔 푡푔 + 푡푔 ,∀푥, 푦 ∈ (−1,1), obţinem că (G,o) este grup abelian şi (G,o)≈(퐑, +). 2) Determinaţi grupul (퐺,∗) ştiind că funcţia 푓: (0,∞) → 퐺,푓(푥) = 푥 + 1 este un izomorfism de grupuri de la (퐑∗ , ∙ ) la (퐺,∗). Soluţie: f bijectivă ⇒Im f = G ; dar Im f = (1, ∞) şi deci G = (1, ∞). Grupul (퐺,∗) se obţine prin transport de structură al grupului (퐑∗ , ∙ ) prin funcţia f şi deci legea este: 푥 ∗ 푦 = 푓(푓 (푥) ∙ 푓 (푦)),∀ 푥,푦 ∈ (1,∞). Avem 푓 : (1,∞) → (0,∞),푓 (푦) = 푦 − 1 şi deci

푥 ∗ 푦 = 푓 (푥 − 1)(푦 − 1) = (푥 − 1)(푦 − 1) + 1 = 푥푦 − 푥 − 푦 + 2,∀푥, 푦 ∈ (1,∞). Bibliografie: Dana Heuberger, Vasile Pop, Gheorghe Boroica, Nicolae Muşuroia, Adrian Magdaş, Ioana Magdaş, Viorel Lupşor- Matematică pentru grupele de performanţă- clasa a XII-a, ed. Dacia Educaţional, Cluj- Napoca, 2004

Page 44: RMM 14.pdf

S.S.M.ROMÂNIA - Filiala Mehedinți 2014

44 REVISTA MEHEDINȚEANĂ DE MATEMATICĂ NR. 14

PROBLEME PROPUSE Elevii vor rezolva probleme de la clasa pe care o urmează şi de la clasa inferioară. Se pot rezolva şi problemele date la Etapa Locală a Olimpiadei de matematică. Soluţiile redactate pe foi format A4 se vor preda profesorului îndrumător.

Ciclul Primar

P.1. Deschizând manualul de matematică la întâmplare , constatați că suma numerelor ce indică cele două pagini succesive este 297. Aflați numerele înscrise pe cele două pagini.

Prof. Șimandan Gileta P.2. Fie numerele a=푥푦69 și b=푥56푥 . Determinați x și y astfel încât: a) a = b b) a > b c) a < b.

Prof.Cora Ionela Dăniasă P.3. Câte pagini are o carte dacă pentru numerotarea ei s-au folosit 444 cifre ?

Prof. Șimandan Gileta P.4. Optimea unui număr este egală cu treimea altui număr. Aflati numerele dacă diferența lor este 20.

Prof. Dr.Gheorghe Căiniceanu P.5. 9 stilouri şi 8 pixuri costă 69 RON şi 3 stilouri şi 5 pixuri costă 30 RON. Cât costă un stilou? Dar un pix?

RMM -7-2007 P.6. Într-o ogradă sunt raţe şi oi, în total 16 capete şi 44 picioare. Câte raţe şi câte oi sunt în ogradă?

RMM- 7-2007 P.7. Mă gândesc la un număr din care scad 350, apoi triplez rezultatul, scad 2350, înjumătăţesc rezultatul şi obţin 925. La ce număr mă gândesc?

RMM-7-2007 P.8. Micşoraţi de 9 ori suma dintre triplul numărului 96 mărit de 7 ori şi jumătatea sfertului numărului 8 888 micşorat cu 346. Ce rezultat aţi obţinut?

RMM-7-2007 P.9. Pe fiecare dintre primele 4 cărți numărul de jos este într-o aceeași legătura ascunsă cu numărul de sus. Oare care este al doilea număr scris pe cea de-a 5-a carte ?

RMM-7-2007 P.10. Executând un panou pentru cabinetul de matematică Flavius constată că îi lipsesc șase etichete reprezentând aceeași cifră. Ce cifră îi lipsește,dacă el a putut scrie doar șirul :

RMM-7-2007 P.11. Suma a trei numere este 418.Să se afle numerele știind că primul este de două ori mai mare decât al treilea,iar al doilea este cu 2 mai mic decât sfertul primului.

RMM-7-2007 P.12. O gospodină a cumpărat 10 Kg prune și 5 Kg struguri plătind în total 20 000 lei. Câți lei costă 1 Kg de prune și câți lei costă 1 Kg de struguri, dacă prețul unui Kg de prune este de 3 ori mai mic decât al unui Kg de struguri ?

32

6

25

10

53

15

63

18

74

?

1 7 2 1 2 2 + + =

Page 45: RMM 14.pdf

S.S.M.ROMÂNIA - Filiala Mehedinți 2014

45 REVISTA MEHEDINȚEANĂ DE MATEMATICĂ NR. 14

RMM-7-2007 P.13. Află cele trei numere a, b, c, ştiind că a : b = 4 rest 2, b : c = 4 rest 2, iar a – c = 880.

RMM-7-2007 P.14. Suma a cinci numere consecutive pare este de 6 ori mai mare decât primul număr şi cu 100 mai mare decât el. Află cele cinci numere.

RMM-7-2007 P.15. Dacă se împarte un număr x la 8, atunci numărul se micşorează cu 245. Află valoarea lui x.

RMM-7-2007 P.16. Elena rezolvă 24 de probleme într-o săptămână, Mihai rezolvă cu 8 mai multe, iar Ana cu 12 probleme mai puţin decât Elena şi Mihai la un loc. - află câte probleme vor rezolva, în acelaşi ritm, în primul semestru ( 17 săpt.). - scrie rezolvarea problemei printr-un singur exerciţiu cu mai multe operaţii şi rezolvă.

RMM-7-2007 P.17. Aflaţi numerele naturale a, b, c care îndeplinesc simultan condiţiile:

a) al doilea număr este de trei ori mai mare decât primul, b) al treilea număr este jumătatea celui de-al doilea, c) suma numerelor este mai mare decât 7 şi cel mult egală cu 44.

RMM-6-2006 P.18. Vlad are un număr de bile. El observă că le poate grupa câte 6 dar şi câte 10, iar diferenţa dintre grămezi este 2. Câte bile are Vlad?

RMM-6-2006 P.19. Două veveriţe consumă rezerva de alune în 6 luni. În cât timp vor termina rezerva trei veveriţe?

RMM-6-2006 P.20. Radu are de 7 ori mai multe timbre decât colegul său. Dacă ar fi avut cu 48 de timbre mai puţine, iar colegul său cu 36 de timbre mai multe, atunci Radu ar fi avut de trei ori mai multe timbre decât colegul său. Câte timbre are fiecare?

RMM-6-2006 P.21. La concursul „Cangurul” pentru fiecare problemă din cele 20 de probleme propuse, rezolvată corect, se acordă 10 puncte, iar pentru fiecare problemă nerezolvată se scad 5 puncte. Ana a obţinut 125 puncte. Câte probleme a rezolvat corect ?

RMM-6-2006 P.22. Aflaţi numerele naturale a, b, c care îndeplinesc simultan condiţiile:

a + c = 20, a – b = 5, b – c = 5. RMM-6-2006

P.23. Diferenţa a două numere este 226 iar primul mărit cu 2 este de 8 ori mai mic decât al doilea. Care sunt numerele?

RMM-6-2006 P.24. Răsturnatul semisumei a două numere este 24, iar a patra parte din primul este egală cu a treia parte din al doilea. Care sunt numerele?

RMM-6-2006 P.25. Tatăl are 46 de ani iar fiul are 19 ani. Cu câţi ani în urmă tatăl era de patru ori mai în vârstă decât fiul ?

RMM-6-2006 P.26. Produsul vârstelor a trei fraţi este 72, iar suma vârstelor este mai mică decât 15. Câţi ani are fiecare, ştiind că nu sunt gemeni ?

RMM-6-2006

Page 46: RMM 14.pdf

S.S.M.ROMÂNIA - Filiala Mehedinți 2014

46 REVISTA MEHEDINȚEANĂ DE MATEMATICĂ NR. 14

Clasa a V-a

V.1. Arătați că numărul 35 ∙ 55 ∙ 77 este pătrat perfect. Prof. Elena Rîmnicianu

V.2. Să se rezolve ecuațiile: a) (x+2x+3x+...+403x):202=2015 b) (x-1)+(x-2)+...+(x-403)=2015

Prof. Elena Rîmnicianu V.3. Produsul a două numere naturale consecutive este mai mic decat dublul numărului x cu 4028. Să se determine cele două numere știind că x =1+2+3+…+2014.

Prof. Elena Rîmnicianu V.4. Arătați că există un multiplu al lui 2015 care să nu aibă nici o cifră nenulă.

Prof. Daniel Sitaru V.5. Să se scrie în ordine descrescătoare și să se determine termenii din mijloc ai șirului de fracții:

914 ;

1021 ;

1128 ; … … ;

2952016

Prof. Daniel Sitaru V.6. Câte numere 푁 = 푎푏푎7푎 sunt divizibile cu 3? Dar cu 7? Dar cu 21?

Prof. Daniel Sitaru V.7. Să se scrie numărul 7 ca sumă a șapte numere naturale consecutive.

Prof. Daniel Sitaru V.8. Numerele 1,2,3, … ,64 se dispun pe o tablă de șah în așa fel încât pe fiecare coloană suma numerelor să fie aceeași și numărul 7 să fie pe un pătrat alb. Se cere suma numerelor de pe pătratele negre.

Prof. Daniel Sitaru V.9. Să se arate că numerele 푎푎푎푏푏푏 si 푏푏푏푎푎푎 nu pot fi simultan pătrate perfecte.

Prof. Daniel Sitaru V.10. Să se arate că există 푛 ∈ 푁∗ încât numărul scris cu “n” cifre de 1 să se dividă cu 2017.

Prof. Daniel Sitaru V.11. Fie = 푎푏푐푑푒 ; 푁 = 푏푐푑푒푎 ; 푃 = 푐푑푒푎푏 . Să se arate că dacă oricare dintre numerele 푀,푁,푃 este divizibil cu 41 atunci 푀 + 푁 + 푃 este divizibil cu 41 .

Prof. Daniel Sitaru V.12. Să se arate că numărul: 푁 = 2 ∙ 3 + 1 este divizibil cu 17 .

Prof. Claudia Nănuți V.13. Să se determine toate numerele naturale de maxim șase cifre care au cifrele direct proporționale cu rangul lor de la dreapta spre stânga.

Prof. Claudia Nănuți V.14. Câte perechi de numere naturale (m; n) există, astfel încât 2m+3n=2015 și câtul împărțirii lui m la n este 9?

Prof. Ionică Constantin V.15. Determinați suma tuturor resturilor împărțirilor a 100 de numere naturale consecutive la 19, știind că cel mai mic se împarte exact la 19.

Prof. Ionică Constantin V.16. Numerele naturale a, b, c și d împărțite la 5 dau câturi numere impare consecutive și resturi nenule diferite. a) Arătați că (a + b + c + d) ⋮ 10. b) Determinați valoarea minimă a sumei (a + b + c + d).

Prof. Ionică Constantin

Page 47: RMM 14.pdf

S.S.M.ROMÂNIA - Filiala Mehedinți 2014

47 REVISTA MEHEDINȚEANĂ DE MATEMATICĂ NR. 14

V.17. Aflați numerele naturale x, y și z astfel încât 4072 + 1 ⋅ 2 ⋅ 3 ⋅ … ⋅ x = y . Prof. Palașcă Vladimira

V.18. Să se rezolve în mulțimea numerelor naturale ecuația: (1 + 4 + 4 + ⋯+ 4 + 4 ) ⋅ x = 4 + 4 + 4 + ⋯+ 4 + 4 .

Prof. Palașcă Vladimira V.19. Se dă numărul a= 20142015......0111234567891 .

a)Stabiliţi care este a 2015-a cifră a numărului a. b)Verificaţi dacă a este pătrat perfect.

Prof.Bondoc Gabriela V.20. Arătaţi că numărul A=1!+2!+3!+4!+....+2014! nu este pătrat perfect.

Prof.Bondoc Gabriela V.21. Se dă mulţimea A={ Nnnn |352 3 }

a)Aflaţi n pentru care 8003 A . b)Arătaţi că A nu conţine pătrate perfecte. Prof.Bondoc Gabriela

V.22. Să se determine cel mai mic număr natural care nu este nici număr prim nici pătrat perfect și care nu are niciun factor prim mai mic decât 30.

Prof. Dr.Gheorghe Căiniceanu V.23. Determinați numerele naturale de forma A=2015abc , care împărțite la 201 dau restul 15.

Prof. Dan Nedeianu

Clasa a VI-a

VI.1. Să se arate că numerele 20141007 749 și 20141007 5225 sunt prime intre ele. Prof. Elena Rîmnicianu

VI.2. Aflați numărul natural x știind că: 2016 x - 2016 x-1 = 2015. Prof. Elena Rîmnicianu

VI.3. Știind că unghiul A ia valoarea din proporția: 2015...321

2015352016

A , să se afle

suplementul complementului unghiului A. Prof. Elena Rîmnicianu

VI.4. Să se determine toate numerele naturale 푎푎푎푏 cu proprietatea 푎푎푎푏 = 푐푐푐푑푑푑푑푏 Prof. Daniel Sitaru

VI.5. Fie 푎, 푏, 푐,푑 ∈ ℤ și 2 3 5 7 3 5 7 2 5 7 2 3 ; ; ;

20 20 20a b c d a b c d a b c dA B C

7 2 3 520

a b c dD .Să se arate că dacă oricare trei dintre numerele 퐴,퐵,퐶,퐷 sunt

întregi atunci și cel de-al patrulea număr este întreg. Prof. Daniel Sitaru

VI.6. Fie (푂퐴 bisectoarea unghiului ∢퐴 푂퐴 cu măsura de 128°32 16 . (푂퐴 este bisectoarea ∢퐴 푂퐴 , (푂퐴 este bisectoarea ∢퐴 푂퐴 ....(푂퐴 este bisectoarea ∢퐴 푂퐴 . Aflați cel mai mic 푛 pentru care 푚(∢퐴 푂퐴 ) < 1

Prof. Daniel Sitaru VI.7. Să se arate că oricum am alege 2015 numere întregi există unele dintre acestea cu suma dintre ele divizibilă cu 2015 .

Prof. Daniel Sitaru

Page 48: RMM 14.pdf

S.S.M.ROMÂNIA - Filiala Mehedinți 2014

48 REVISTA MEHEDINȚEANĂ DE MATEMATICĂ NR. 14

VI.8. Să se determine al 26-lea termen al șirului: 1 2 1 3 2 1 4 3 2 1 5 4, , , , , , , , , , , ,1 1 2 1 2 3 1 2 3 4 1 2

Prof. Daniel Sitaru VI.9. Să se arate că diferența pătratelor a două numere prime mai mari decât 3 este divizibilă cu 12.

Prof. Daniel Sitaru VI.10. Să se arate că nu există 푥,푦, 푧 ∈ ℕ încât:푥 + 푦 + 푧 = 2029

Prof. Daniel Sitaru

VI.11. Fie 푎, 푏, 푐,푑 ∈ ℕ. Să se arate că dacă: 푎 − 3푏 + 2푐 ≥ 0,푏 − 3푐 + 2푑 ≥ 0, 푐 − 3푑 + 2푎 ≥ 0,푑 − 3푎 + 2푏 ≥ 0

atunci 푎 = 푏 = 푐 = 푑 . Prof. Claudia Nănuți

VI.12. Să se afle un număr natural 푛 astfel încât restul împărțirii numărului 푁 = 푎푎푎푏푏푏푐푐푐7 la 푛 și restul împărțirii sumei cifrelor sale la 푛 să coincidă.

Prof. Claudia Nănuți VI.13. Astăzi este ziua de naștere a unui copil, dar și a bunicului său. Bunicul are atâția ani câte luni are nepotul său, iar suma vârstelor lor este de 78 ani. Câți ani are fiecare?

Prof. Ionică Constantin VI.14. Determinați numerele de forma 42xyz pătrate perfecte și divizibile cu 5.

Prof. Ionică Constantin VI.15. Fie x, y, z ∈ ℕ astfel încât 4(2x− 3z) = 5(x + y).Să se demonstreze că produsul y(x + z) este divizibil cu 15.

Prof. Ionică Constantin VI.16. Într-o clasă sunt x fete și y băieți, numerele x și y fiind direct proporționale cu numerele 2 și 3. Urmează ca un elev sau o elevă să se transfere în altă clasă și astfel numărul fetelor va reprezenta 60% din cel al băieților. Câți elevi sunt acum în clasă?

Prof. Palașcă Vladimira VI.17. Să se rezolve în ℤ ecuația: xy + 2x = 3y + 9.

Prof. Palașcă Vladimira VI.18. Se consideră numărul B=2015+20152+20153+....+20152015.

a) Calculaţi numărul B. b) Determinaţi restul împărţirii lui 20152015 la 2014.

Prof.Bondoc Gabriela VI.19. Să se determine cel mai mic număr natural de trei cifre care are exact 5 divizori .

Prof. Dr.Gheorghe Căiniceanu

VI.20. Determinați toate perechile (a,b) de numere naturale, cu 0 ≤ a ≤ b ≤ 30, a + b este număr prim și a + b divide ab + 4.

Prof. Dan Nedeianu VI.21. Pe un cântar se află ciuperci. Fiind proaspăt culese, 99% reprezintă apă. După o perioadă, o parte din cantitatea de apă s-a evaporat, procentul de apă devenind de 98%. Dacă inițial cântarul indica 1kg, ce greutate va indica după evaporare?

Prof. Piț-Rada Marica

Page 49: RMM 14.pdf

S.S.M.ROMÂNIA - Filiala Mehedinți 2014

49 REVISTA MEHEDINȚEANĂ DE MATEMATICĂ NR. 14

Clasa a VII-a

VII.1. Arătați că numărul: 222

22

2020202606060670707071212121213131313

este număr rațional.

Prof. Elena Rîmnicianu VII.2. În paralelogramul ABCD, măsura unghiului B este egală cu suplementul unghiului de 600,

AB = 12 cm, iar BC este egal cu două treimi din AC. Dacă M este mijlocul laturii AB, să se afle DM, perimetrul şi aria paralelogramului.

Prof. Elena Rîmnicianu VII.3. Știind că 2 < 푥 < 4, arătați că < <

Prof. Palașcă Vladimira VII.4. Arătați că dacă numerele raționale a și b îndeplinesc condițiile: a + b < 4 și

ab − 2a − 2b + 4 > 0, atunci a < 2 și b < 2. Prof. Palașcă Vladimira

VII.5. Demonstrați că dacă numerele întregi x, y, z verifică relația: xy + 2z(x − y) = 4z + 7, atunci |x + y| = 8.

Prof. Ionică Constantin VII.6. Să se determine n ∈ ℕ pentru care √n + 101 + √n + 10 ∈ ℚ.

Prof. Ionică Constantin

VII.7. Aflați elementele mulțimii A = x ∈ ℤ / ∈ ℤ

Prof. Ionică Constantin

VII.8. Aflați Nn și Zx astfel încât 243

312

nnx .

Prof.Bondoc Gabriela

VII.9. Să se arate că 7 are un multiplu care se scrie numai cu cifre de 9. Generalizare : arătați că orice număr prim mai mare sau egal cu 7 are un multiplu care se scrie numai cu cifre de 9.

Prof.Dr.Gheorghe Căiniceanu

VII.10. Să se arate că dacă ∈ 푁 ;푛 ∈ 푁∗atunci ∈ 푁 . Prof. Daniel Sitaru

VII.11. Fie , b, c, d ∈ (0 ,∞) ; a + b + c + d = 2. Să se arate că: 푎(푏 + 푐 + 푑) + 푏(푎 + 푐 + 푑) + 푐(푎 + 푏 + 푑) + 푑(푎 + 푏 + 푐) ≤ 4

Generalizare: Fie 푥 , 푥 , … ,푥 ∈ (0,∞) ; 푥 + 푥 + ⋯+ 푥 = 2 ; 푛 ∈ ℕ. În aceste condiții:

푥 (푥 + 푥 + ⋯+ 푥 ) + 푥 (푥 + 푥 + ⋯+ 푥 ) + ⋯+ 푥 (푥 + 푥 + ⋯+ 푥 ) ≤ 푛 Prof. Daniel Sitaru

VII.12. A) Să se determine 푎,푏, 푐,푑, 푒, 푓 ∈ 푁∗ astfel încât: 2014 = 푎 + 푏 − 푐 ; 2015 = 푑 + 푒 − 푓 .

B) Să se arate că dacă 푛 ∈ 푁,푛 ≥ 7 există 푎,푏, 푐 ∈ 푁∗ încât 푛 = 푎 + 푏 − 푐 . Prof. Daniel Sitaru

VII.13. Fie 퐴퐴′ mediană în triunghiul 퐴퐵퐶; 퐴′ ∈ (퐵퐶). Fie 푀 ∈ (퐵퐴′); 푁 ∈ (퐴′퐶). Paralela prin 푀 la 퐴퐴′ intersectează 퐴퐵 şi 퐴퐶 în 푆, respectiv 푇. Paralela prin 푁 la 퐴퐴′ intersectează 퐴퐶 şi 퐴퐵 în 푆′, respectiv 푇′.Să se arate că: 푀푆 + 푀푇 = 푁푆 ′ + 푁푇′

Prof. Daniel Sitaru

Page 50: RMM 14.pdf

S.S.M.ROMÂNIA - Filiala Mehedinți 2014

50 REVISTA MEHEDINȚEANĂ DE MATEMATICĂ NR. 14

VII.14. Se dă un trapez de baze 퐴퐵 = 17푐푚, 퐶퐷 = 8푐푚 și 푀푁 ∥ 퐴퐵 ;

; ; 3 .BNM AD N BCCN

Să se calculeze 푀푁.

Prof. Daniel Sitaru VII.15. Determinați toate numerele de trei cifre cu proprietatea că modulul diferenței dintre număr

și inversatul său este număr par. Prof. Daniel Sitaru

VII.16. Arătați că dacă media aritmetică a primelor “n” zecimale ale numărului √3 − 1 este cuprinsă între 4 si 4 atunci și media aritmetică a primelor “n” zecimale ale numărului 2 − √3 are aceeași proprietate.

Prof. Daniel Sitaru VII.17. Să se determine 푥, 푛 ∈ ℕ∗ încât:

푥0, (푥) +

푥0,0(푥) +

푥0,00(푥) + ⋯+

푥0, 000 … 0(푥)

"n-1"

= 3789

Prof. Daniel Sitaru VII.18. Să se arate că dacă 푎,푏, 푐 ∈ ℝ și

1 − 4푎 + 3푏 ≥ 0,푎 − 4푏 + 3푐 ≥ 0, 푏 − 4푐 + 3푎 ≥ 0, 푐 − 4 + 3푎 ≥ 0 atunci: 푎 = 푏 = 푐 = 1 .

Prof. Claudia Nănuți

VII.19. Fie 푥 ∈ 푅 ; 푖 ∈ 1,6 ; 6

2

, 11i

i ji j

x

. Să se arate că:

6 2 2

, 1, 1

105 ( )i j i j i ji j i ji j

x x min x x

Prof. Claudia Nănuți VII.20. Fie = 9 ∙ 111 … 1

푎푎푎…푎 푏푏푏… 푏 + 8 .Să se afle 푎,푏 ∈ ℕ încât 퐶 să fie un cub

perfect. Prof. Daniel Sitaru

VII.21. Se dă triunghiul ABC ascuțitunghic și punctele A’ ϵ [BC], B’ ϵ [AC], C’ ϵ [AB] astfel încât AA’∩BB’∩CC’={M},iar ' ' ' 1.MA MB MC Să se calculeze MA + MB + MC – MA ∙ MB ∙ MC.

Prof. Dan Nedeianu VII.22. Calculaţi suma :

( √ )√+

√ ∙ (√ √ ) + … +

√ (√ √ ) .

Prof. Draga Tătucu Mariana VII.23. Determinaţi 푥,푦 ∈ 푄 cu 푦 ≥ 0 astfel încât are loc √3 ∙ 푥 − 2푥 + 2√3 − 5 + 푦 =2015 .

Prof. Draga Tătucu Mariana VII.24. Aflaţi cea mai mică valoare a numărului

푛 = |푎 − 5푏 + 2| + |8 − 3푎 + 푏| + |2 + 푎 + 2푏| + |푎 + 2푏 + 4| , unde 푎,푏 ∈ 푅 . Prof. Draga Tătucu Mariana

VII.25. Fie triunghiul echilateral 퐴퐵퐶 cu 퐴퐵 = 12 푐푚 . Fie punctele 푀,푁,푃 astfel ca 퐴 ∈ (퐶푁), 퐶 ∈ (퐵푀) şi 퐴퐵 ∩ 푀푁 = {푃} . Calculaţi 퐴푃, stiind ca AN = CM = 12 cm.

Prof. Draga Tătucu Mariana

Page 51: RMM 14.pdf

S.S.M.ROMÂNIA - Filiala Mehedinți 2014

51 REVISTA MEHEDINȚEANĂ DE MATEMATICĂ NR. 14

VII.26. Fie trapezul 퐴퐵퐶퐷 cu 퐴퐵||퐶퐷 . Dacă 퐴퐵 > 퐶퐷 şi 퐴퐵,퐶퐷 sunt invers proporţionale cu 1 si 3, iar 퐴퐵 + 퐶퐷 = 20 푐푚 . Aflaţi aria trapezului ştiind că 퐴퐷 = 6 푐푚 şi = 8 푐푚 .

Prof. Draga Tătucu Mariana VII.27. Rezolvați ecuația în ℝ ∶ [푥 + 4] + 2[푥 − 2] − = [2 + 푥] .

Prof. Draga Tătucu Mariana

Clasa a VIII-a

VIII.1. Fie x,y,z,t numere reale pozitive astfel încât xy=zt=2014. Să se demonstreze că :

161 (1+x)(1+y)(1+z)(1+t)2014.

Prof. Elena Rîmnicianu

VIII.2. Stabiliți dacă numărul a= 5085044

5125084

5165124 …+

201220084

201620124

aparține intervalului (504

1;1008

1 ).

Prof. Elena Rîmnicianu VIII.3. Dacă x, yR și x(-3;1006), y(-5 ;1008), arătați că numărul

2 2( 2014) ( 8)a x y x y este natural. Prof. Elena Rîmnicianu

VIII.4. Să se determine elementele mulțimii A={x∈ 푍/ ∈ 푍} Prof. Elena Rîmnicianu

VIII.5. Demonstrați că: a) |1007푥 + 1| + 1007|2 − 푥| ≥ 2015 , ∀푥 ∈ 푅 b) |504푥 − 2| + 504|4− 푥| ≥ 2014,∀푥 ∈ 푅

și calculați valorile întregi ale lui x pentru care se realizează egalitatea. Prof. Elena Rîmnicianu

VIII.6. Să se afle numerele naturale pătrate perfecte de forma: 25푛 − 2016. Prof. Elena Rîmnicianu

VIII.7. Determinați soluțiile naturale ale ecuației: 4√x + 1 + 8 y − 1 + 12√z− 2 = x + y + z + 54

Prof. Ionică Constantin

VIII.8. Să se arate că + + ≥ 82, ∀x, y, z ∈ (2,∞). Prof. Ionică Constantin

VIII.9. Aflați x, y ∈ ℝ astfel încât √x − 1024 + y − 1024 = . Prof. Ionică Constantin

VIII.10. Să se rezolve în mulțimea numerelor întregi ecuația: √x − 55 + √68− x = |2x + 11|.

Prof. Palașcă Vladimira VIII.11. Determinați numerele a și b știind că există numerele reale x și y astfel încât :

x + y + 2ax − 10y + a + 26 + x + y − 8x − 2by + b + 20 = 3 . Prof. Palașcă Vladimira

VIII.12. A) Un poliedru este format prin reunirea punctelor unei prisme și ale unei piramide care au o față comună. Câte muchii are poliedrul dacă se știe că are 9 fețe și 9 vârfuri?

Page 52: RMM 14.pdf

S.S.M.ROMÂNIA - Filiala Mehedinți 2014

52 REVISTA MEHEDINȚEANĂ DE MATEMATICĂ NR. 14

B) Pentru un poliedru format dintr-o prismă triunghiulară regulată cu înalțimea 3 și un tetraedru regulat de muchie 3, care au o bază comună, aflați aria totală.

Prof. Dr.Gheorghe Căiniceanu VIII.13. Fie VABCD şi V’ABCD piramide patrulatere cu aceeaşi bază şi vârfurile de o parte şi de alta a planului (ABCD). Pe cele opt feţe laterale se înscriu numerele de la 1 la 8, o singură dată pe fiecare faţă. Să se studieze dacă este posibil ca suma numerelor de pe cele patru feţe care se întâlnesc într-un vârf să fie aceeaşi pentru orice vârf.

Prof. Daniel Sitaru VIII.14. Punctele unei sfere de rază 4 cm se colorează cu trei culori diferite. Să se arate că există două puncte pe sferă, de aceeași culoare și aflate la o distanță de 6, 6√2 sau 3√6 cm unul de altul.

Prof. Daniel Sitaru VIII.15. Fie (푂퐴 , (푂퐴 , … , (푂퐴 ;푛 ≥ 2 ;푛 ∈ 푁 semidrepte astfel încât:

푚(∢퐴 푂퐴 ) = 2푥,∀푖 ∈ 1,푛 − 1 ; 푥 ∈ (0°, 90°) Să se arate că: 1 2 2 3 1 1 1 22 ( )n n n nA A A A A A A A sinx OA OA OA

Prof. Daniel Sitaru VIII.16. Fie 훼 ,훽 plane paralele cu baza 퐴퐵퐶퐷 a unei piramide patrulatere regulate 푉퐴퐵퐶퐷, duse

la 15

, respectiv la 45

din înălțimea piramidei față de vârf. Se cere raportul dintre volumul trunchiului

de piramidă cu bazele situate în planele 훼 ,훽 și volumul piramidei inițiale. Prof. Daniel Sitaru

VIII.17. Fie 푉퐴 퐴 퐴 o piramidă triunghiulară regulată. Se ridică într-un punct 푀 al bazei o perpendiculară pe planul bazei care intersectează planele fețelor laterale în 퐵 ,퐵 ,퐵 . Să se demonstreze că suma 푀퐵 + 푀퐵 + 푀퐵 este constantă.

Prof. Claudia Nănuți VIII.18. Să se afle 푎,푏, 푐,푑 ∈ ℕ∗ încât:

푎 + 푏 + 푐 + 푑 + 푏푐푑 + 푎푐푑 + 푎푏푑 + 푎푏푐 = 8√푎푏푐푑 Prof. Daniel Sitaru

VIII.19. Fie 푥, 푦, 푧 ∈ (0 ,∞). Să se arate că:

푥푦푥 + 푦+

푦푧푦 + 푧 +

푥푧푥 + 푧 ≥ 푥푦 + 푦푧 + √푥푧 −

푥 + 푦 + 푧2

Prof. Daniel Sitaru

VIII.20. Fie 푎 = 20 + 20 + ⋯+ √20" "

; 푏 = 42 + 42 + ⋯+ √42" "

.

Rezolvați ecuația: 3 4 3 1 [ ]3 3

x xx b a , unde[푥]este partea întreagă a numărului

real 푥. Prof. Daniel Sitaru

VIII.21. Să se demonstreze că: 2 2 2 2 1499 143 195 4056195 15

Prof. Daniel Sitaru VIII.22. Fie 푎, 푏, 푐 ∈ (0,∞). Să se arate că: 2 2 25 3 4 2( 2 3 2 )a b c ab bc ac

Prof. Daniel Sitaru

VIII.23. Fie 푎 ∈ (0,2). Să se arate că: 2 2

1 12015 2015 24 14(2 )a a

Page 53: RMM 14.pdf

S.S.M.ROMÂNIA - Filiala Mehedinți 2014

53 REVISTA MEHEDINȚEANĂ DE MATEMATICĂ NR. 14

Prof. Daniel Sitaru VIII.24. Triunghiul 퐴퐵퐶 având centrul de greutate 퐺 se proiectează pe un plan în triunghiul 퐴′퐵′퐶′ cu centrul de greutate 퐺′. Dacă: 퐴퐴 = 푥;퐵퐵 = 푦;퐶퐶 = 푧;퐺퐺 = 푔, să se arate că:

푥 + 푦 + 푧 ≥푔3

Prof. Daniel Sitaru VIII.25. Să se afle 푎,푏, 푐,푑 ∈ (0,∞) astfel încât 푎 + 푏 + 푐 + 푑 + 푒 = 1 și suma 푎푏 + 푏푐 + 푐푑 + 푑푒 să fie maximă.

Prof. Daniel Sitaru VIII.26. Să se arate că există două mulțimi disjuncte A și B astfel încât

1 1 1 1 1, , ,..., ,50 51 51 52 52 53 160 161 161 162

A B

,iar suma

elementelor lui A să fie egală cu suma elementelor lui B. Prof. Dan Nedeianu

VIII.27. Să se determine 푥, 푦 ∈ ℤ astfel încât (x2 –y2 )2-3y=1. Prof. Dan Nedeianu

VIII.28. Să se demonstreze că are loc inegalitatea:

cbaacaccbcbbaba 222222 cu egalitate pentru a = b = c, oricare ar fi numerele reale a, b, c.

Prof.Piț-Rada Ionel-Vasile

Clasa a IX-a

IX.1. Fie ,0,...,, 201421 xxx astfel încât 1... 201421 xxx . Să se arate că

,2015,1... 22014

22

21 k

kxxx .

Prof. Ovidiu Ticuși IX.2. Se dau numerele reale pozitive nenule a,b,c astfel ȋncȃt abc=1 şi a+b+c˃(1 /a)+(1/b)+(1/c).Să se arate că cel puţin unul din numerele a,b,c este strict mai mare decȃt 1.

Prof.Dr.Tomiţă Vasile IX.3. Sa se determine toate numerele întregi k pentru care ecuația 022 kxx admite rădăcini raționale.

Prof. Dr.Gheorghe Căiniceanu IX.4. Să se rezolve ecuațiile:

a) + + + = 4, 푢푛푑푒[푎] = 푝푎푟푡푒푎 î푛푡푟푒푎푔ă 푎 푙푢푖 푎.

b) + + + ⋯+ = 푛, 푢푛푑푒 푘, 푛 ∈ ℕ푐푢 푘 ≤ 푛; Prof.Adela Grecu

IX.5. Fie A,B,C măsurile, în radiani, ale triunghiului ABC. Să se arate că: 2 2 2 13

A B C

Prof. Claudia Nănuți IX.6. Fie 푆 = 1 + 2 + ⋯+ 푛 ;푛 ∈ ℕ∗ . Să se arate că: 1 + 2 + ⋯+ 푛 divide 푆 .

Prof. Daniel Sitaru

Page 54: RMM 14.pdf

S.S.M.ROMÂNIA - Filiala Mehedinți 2014

54 REVISTA MEHEDINȚEANĂ DE MATEMATICĂ NR. 14

IX.7. Să se determine numerele naturale a , a , … , a ; n ∈ N∗ știind că: (푎 푎 + 푎 푎 + ⋯+ 푎 푎 ) = + + ; ∀푛 ∈ 푁∗ .

Prof. Daniel Sitaru IX.8. Să se demonstreze că există în plan un cerc care conține în interiorul său exact 2014 puncte laticiale. ( Prin punct laticial se înțelege un punct care are ambele coordonate exprimate prin numere întregi).

Prof. Daniel Sitaru IX.9. Să se determine funcțiile 푓:ℝ → ℝ cu proprietatea:

푥 푓(푦) + 푦 푓(푥) = 2014푓(푥푦); ∀푥,푦 ∈ ℝ Prof. Daniel Sitaru

IX.10. Să se rezolve sistemul:1

11

xyzx yzxy z

Prof. Claudia Nănuți IX.11. Fie :ℝ → ℝ , astfel încât : 2 3 4 0, f f f x f f x f x x x R Să se arate că funcția 푓 este injectivă.

Prof. Daniel Sitaru IX.12. Fie 퐴퐵퐶 un triunghi echilateral înscris într-un cerc de rază 푟 și 푀 un punct pe cercul circumscris triunghiului. Să se arate că: 3 2MA MB MC r

Prof. Claudia Nănuți IX.13. Fie 퐴퐵퐶퐷 un pătrat înscris într-un cerc de rază 1 și 푀 un punct pe cercul circumscris pătratului. Să se arate că: 4 2MA MB MC MD

Prof. Claudia Nănuți

IX.14. Să se arate că oricum am alege cinci puncte de coordonate întregi există cel puțin un punct de coordonate întregi situat în interiorul unui segment determinat de două dintre punctele date.

Prof. Daniel Sitaru IX.15. Să se compare numerele: 푠푖푛(푐표푠2015) și 푐표푠(푠푖푛2015).

Prof. Daniel Sitaru IX.16. Să se arate că nu există nici un triunghi 퐴퐵퐶 în care să fie adevărată relația:

푝 = 푟 푡푔퐴2 + 푡푔

퐵2 + 푡푔

퐶2

Prof. Daniel Sitaru IX.17. Să se demonstreze că:

푡푔1° + 푡푔2° + ⋯+ 푡푔44° + 푡푔1°푡푔44° + 푡푔2°푡푔43° + ⋯+ 푡푔22°푡푔23° = 22 Prof. Daniel Sitaru

IX.18. Fie 푎, 푏, 푐, 푑 ∈ (0,∞). Să se arate că:

1 +푎푏 + 1 +

푏푐 + 1 +

푐푑 + 1 +

푑푎 ≥ 16

Prof. Daniel Sitaru

IX.19. Fie 푎, 푏, 푐 ∈ [1,3]. Să se arate că:

(푎 + 푏 + 푐)1푎 +

1푏 +

1푐 ≤ 81

Prof. Daniel Sitaru

Page 55: RMM 14.pdf

S.S.M.ROMÂNIA - Filiala Mehedinți 2014

55 REVISTA MEHEDINȚEANĂ DE MATEMATICĂ NR. 14

IX.20. Fie 퐴 = {1,2,3, … ,2015}. Să se afle probabilitatea ca alegând o progresie aritmetică de trei termeni, cu rație strict pozitivă, cu elemente din 퐴 aceasta să conțină numărul 2015.

Prof. Daniel Sitaru

IX.21. Să se demonstreze că: 2015 2015 02015 2015

sin tgcos ctg

Prof. Claudia Nănuți

IX.22. Să se rezolve ecuația 푥 = 3 ; 푥 ∈ ℤ unde x este partea întreagă a numărului real x . Prof. Dan Nedeianu

IX.23. În ∆ABC, de ortocentru H se construiesc punctele A’,B’,C’ care sunt simetricele punctului H față de mijloacele segmentelor [BC], [AC], respectiv [AB]. Dacă ' ' ' 0AA BB CC

,să se

determine 푘 ∈ ℝ astfel ca AB AC k AH

. Prof. Dan Nedeianu

Clasa a X-a

X.1. Să se arate că 2015 20151log 2 log 2014! 0,1

2014

Prof. Ovidiu Ticuși

X.2. Să se arate că dacă 푥 ∈ [0,1] atunci: 2 1 2 12 2 1 ( 2 1) 2 22 1 2 1

x x

Prof. Daniel Sitaru X.3. Fie: 퐴 = {(푎, 푏)|2푎 + 2푏 + 5푎푏 + 2푎 + 푏 = 6 ;푎,푏 ∈ 푍} Câte soluții are ecuația 퐴 = 퐴 ∪ 퐴 ∪ 퐴 ∪ 퐴 ?

Prof. Daniel Sitaru X.4. Fie 푎,푏, 푐,푑 ∈ (1 ,∞) încât: ≥ ≥ .Să se arate că : ≥

Prof. Daniel Sitaru

X.5. Să se arate că: 0 1 1 2 1 2 1 ;n n nn n n n n nC C C C C C n N

Prof. Daniel Sitaru X.6. Să se arate că dacă 푎,푏, 푐 ∈ (0,1) atunci: 푎푏푐 ≤ 푎 ∙ 푏 ∙ 푐 Generalizare pentru 푥 ,푥 , … ,푥 ∈ (0,1).

Prof. Daniel Sitaru X.7. Să se arate că dacă 푥 ∈ (0,1) ∪ (1 ,∞) atunci:

1log 3 ∙ log 9 +

1log 9 ∙ log 27 + ⋯+

1log 3 ∙ log 3 =

20142015

1log 3

Prof. Daniel Sitaru X.8. Să se determine 푚,푚,푝,푞, 푟 ∈ ℕ astfel încât: 푚! + 푛! + 푝! + 푞! = 푟! Generalizare: Să se determine 푥 ,푥 , … ,푥 , 푥 ∈ ℕ astfel încât: 푥 ! + 푥 ! + ⋯+ 푥 ! = 푥 !

Prof. Daniel Sitaru

X.9. Să se demonstreze că: 퐶 + 퐶 + 퐶 + ⋯+ 퐶 < 7! + 8! + 9! + ⋯+ 100! Prof. Daniel Sitaru

X.10. Să se rezolve ecuația: 1 + 2 + ⋯+2015 = (퐶 ) Prof. Claudia Nănuți

Page 56: RMM 14.pdf

S.S.M.ROMÂNIA - Filiala Mehedinți 2014

56 REVISTA MEHEDINȚEANĂ DE MATEMATICĂ NR. 14

X.11. În câte moduri pot fi stocate 7 fișiere a câte 520MB pe 4 hard-discuri având fiecare 2GB și astfel încât fiecare hard-disc să conțină cel puțin un fișier?

Prof. Daniel Sitaru

X.12. Să se arate că: *1 2 3 2015 !

1! 2! 3! 2015!

N

Prof. Claudia Nănuți X.13. Să se demonstreze că: 푒 ∙ 휋 > 푥 ; ∀푥 ∈ [푒 ,휋]

Prof. Daniel Sitaru

X.14. Fie 푎, 푏 ∈ [0,∞). Să se arate că ∀푥, 푦 ∈ ℝ : 푎 ∙ 푏 ≤ 푒 √ Prof. Daniel Sitaru

X.15. Fie 푎 ∈ (1,∞); 푏 ∈ (0,∞). Să se arate că: 2 3641 1 127

b b ba a alog a log a log a b

Prof. Claudia Nănuți X.16. Fie 푥 ∈ (1,∞);푦 ∈ (0,∞);푛 ∈ ℕ∗. Să se arate că:

푙표푔 (푥 − 1)푙표푔 (푥 + 1)푙표푔 (푥 + 1) ∙ … ∙ 푙표푔 푥 + 1 < 2 ( ) ∙푦

푛 + 1 Prof. Daniel Sitaru

X.17. Să se rezolve ecuația: 5 + 12 = 푥 Prof. Daniel Sitaru

X.18. Fie 푛 ∈ ℕ∗ și 푥 ∈ , 10 ; 푖 ∈ 1,푛 . Să se arate că: 2

1

10 20i i

nlg x lgx

ii

x n

Prof. Daniel Sitaru X.19. Să se determine ecuația perpendicularei duse din punctul de afix 푧 = 3 + 4푖 pe dreapta: (−1 + 3i)푧 + (−1 − 3푖)푧 + 5 = 0 . Să se determine afixul piciorului perpendicularei.

Prof. Daniel Sitaru X.20. Să se determine puterea punctului de afix z = 4 + 2i

față de cercul de ecuație : (z− 3 + i)(z + 3 − i) = 9 . Prof. Daniel Sitaru

X.21. Să se determine ecuația axei radicale a cercurilor de ecuații: 푧푧 + (1 + 푖)푧 + (1 − 푖)푧+ 3 = 0 ;푧푧 + (3 − 2푖)푧 + (3 + 2푖)푧 + 5 = 0

Prof. Daniel Sitaru X.22. Să se determine ecuația dreptei determinate de punctele de afixe 푧 = 2 + 푖 ; 푧 = 3 − 2푖 .

Prof. Claudia Nănuți X.23. Să se determine 푚 ∈ ℝ încât punctele de afixe 푧 = 1 + 푖 ; 푧 = 푚 + 2푖 ; 푧 = 6 − 푖 să fie coliniare.

Prof. Claudia Nănuți X.24. Să se afle aria triunghiului determinat de punctele de afixe

푧 = 3 − 푖 ; 푧 = 4 + 푖 ; 푧 = 5 − 7푖 Prof. Claudia Nănuți

X.25. Să se determine unghiul format de dreptele de ecuații: (2 + 푖)푧 + (2 − 푖)푧 + 3 = 0 ; (1 − 푖)푧 + (1 + 푖)푧 − 1 = 0

Prof. Claudia Nănuți

X.26. Să se determine 푚 ∈ ℝ încât dreptele de ecuații: 3푖푧 − 3푖푧 + 1 = 0 ; (푚 − 2푖)푧 + (푚 + 2푖)푧 − 1 = 0 să fie paralele.

Prof. Claudia Nănuți

Page 57: RMM 14.pdf

S.S.M.ROMÂNIA - Filiala Mehedinți 2014

57 REVISTA MEHEDINȚEANĂ DE MATEMATICĂ NR. 14

X.27. Să se determine distanța de la punctul de afix 푧 = 5 − 푖 la dreapta de ecuație: (−4 + 2푖)푧 + (−4 − 2푖)푧 + 3 = 0 .

Prof. Claudia Nănuți X.28. Fie 푙푔푎, 푙푔푏, 푙푔푐, 푙푔푑 ; 푎, 푏, 푐, 푑 ∈ (1,∞) lungimile laturilor consecutive ale unui patrulater inscriptibil. Să se arate că:

푙푔푏푐푑푎 푙푔

푎푐푑푏 푙푔

푎푏푑푐 푙푔

푎푏푐푑 ≤ [푙푔(푎푑) ∙ 푙푔(푏푐)]

Prof. Daniel Sitaru

X.29. Să se rezolve ecuația: 2015

0

pk k x kp x n p n

k

C A A A

Prof. Daniel Sitaru X.30. Fie numerele reale a, b, c, x, y, z cu a, b, c>0 si 푎푥 + 푏푦 + 푐푧 ≥ 0. Demonstrați inegalitatea: 2014 1 2014 1 2014 1 0.x y za b c

Prof.Leonard Giugiuc (Prelucrare după Cristinel Mortici)

X.31. Fie 푧 ∈ ℂ un număr complex cu proprietatea că 푧 + ≤ 3.

3|1| 22

zz . Sa se arate ca 5|1|

zz .

Prof. Dr.Gheorghe Căiniceanu X.32. Să se demonstreze că, dacă 푎,푏, 푐 ∈ (0,1) 푐푢 푎푏 < 푐,푏푐 < 푎, 푐푎 < 푏, 푎푡푢푛푐푖:

log 푎 + log 푏 + log 푐 ≤ log √푏푐 + log √푐푎 + log √푎푏 Prof. Vasile Grecu

X.33. Să se rezolve ecuația lg(30+lg x) = lg 4 + lg (lg푥) ,푥 ∈ . Prof. Dan Nedeianu

X.34. Dacă z1,z2,…,z2014∈ ℂ∗astfel ca |z1|2+|z2|2+…+|zn|2+ z12+z2

2+…+z20142=0 . Să se arate că

z1z2z3…z2014∈ ℝ Prof. Dan Nedeianu

Clasa a XI-a

XI.1. Fie 퐴 ∈ 푀 (ℤ); 퐴 = 퐼 . Să se afle 푋 ∈ 푀 (ℤ) încât 퐴푋퐴 = 푋퐴 푋 și 푋 = 퐼 . Prof. Daniel Sitaru

XI.2. Fie 퐴,퐵 ∈ 푀 (ℂ). Pentru 퐴 = (푎 ) , notăm 퐴 = (푎 ) , . Să se arate că dacă 푑푒푡퐴 ∙ 푑푒푡퐵 ≠ 0 atunci 푟푎푛푔 퐴 + 퐵 = 푟푎푛푔((퐴) + (퐵) )

Prof. Daniel Sitaru XI.3. Fie (푋 ) un șir de matrici din 푀 (ℂ) astfel încât există un șir de numere reale nenule (푎 )

cu proprietatea: 푋 푋 = 푎 푋 + 푎 푋 ; 푖 ∈ 1,푛 Să se arate că toate matricile șirului au același rang.

Prof. Daniel Sitaru XI.4. Să se demonstreze că ecuația 푠푖푛푥 = are exact 13 soluții.

Prof. Daniel Sitaru XI.5. Să se calculeze:

lim→

푛 + 푛 + ⋯+ 푛 , 푛 ∈ ℕ,푛 ≥ 2 Prof. Daniel Sitaru

Page 58: RMM 14.pdf

S.S.M.ROMÂNIA - Filiala Mehedinți 2014

58 REVISTA MEHEDINȚEANĂ DE MATEMATICĂ NR. 14

XI.6. Fie 퐴,퐵,퐶 ∈ 푀 (ℝ); 푑푒푡퐴 ≠ 0 și 퐴퐵 = 퐶 . Să se determine 푋 ∈ 푀 (ℝ)încât 퐵퐴 = 푋 .

Prof. Daniel Sitaru

XI.7. Fie 퐴,퐵,퐶,퐷 ∈ 푀 (ℂ); 퐴 + 퐵 = 퐶 + 퐷 ; 퐴 + 퐵 = 퐶 + 퐷 ;퐴퐵 = 퐵퐴 și 퐶퐷 = 퐷퐶. Să se arate că: 퐴 + 퐵 = 퐶 + 퐷 ; ∀푚 ∈ ℕ∗

Prof. Claudia Nănuți XI.8. Fie ,푔: (0,∞) → ℝ;푓(푥) = 푎푟푐푡푔푥 ;푔(푥) = . Să se arate că:

∀푛 ∈ 푁∗ ; 푓(푘)푔(푘) > 푓(푘)푔(푛 − 푘 + 1)

Prof. Claudia Nănuți XI.9. Să se determine formula termenului general și limita șirului:

푎 = 푒 ; 푎 = 푒 ; 푎 ∙ 푎 = 푎 ;푛 ≥ 2 . Prof. Daniel Sitaru

XI.10. Să se arate că șirul: 2 2 2 2

1 1 1 1 ; 12 5 8 (3 1)na n

n

este convergent.

Prof. Daniel Sitaru XI.11. Să se compare numerele: a = (100!) și b = (4!)

Prof. Daniel Sitaru XI.12. Fie : [0 , 2] → ℝ , o funcție continuă astfel încât: 푓(0) = 푓(2) . Arătați că există 푐 ∈ [0 ,1] încât 푓(푐) = 푓(푐 + 1).

Prof. Daniel Sitaru

XI.13. Să se calculeze:

5

3 5 7

50

7 5 3

7

lim3x

log log logcosx

log log logcosx

Prof. Daniel Sitaru

XI.14. Să se arate că dacă:푎,푏 ∈ 0, și (푐푡푔푎) + (푐푡푔푏) ≤ 2 ,∀푥 ∈ ℝatunci 푐푡푔푎 = 푡푔푏. Prof. Claudia Nănuți

XI.15. Fie 푝,푞,푟 ∈ (0,1)∪ (0,∞); 푥,푦, 푧 ∈ ℝ încât:

푝 = 푞푟 ; 푞 = 푝푟 ; 푟 = 푝푞 și =1 1 −푥1 −푦 1−푧 1 1

.Să se arate că 퐴 nu este inversabilă.

Prof. Daniel Sitaru XI.16. Să se calculeze limita șirului:

x =1푙푛푛

1

2푘 − 2√2푘 − 1−

1

2푘 + 2√2푘 − 1

Prof. Claudia Nănuți

XI.17. Să se arate că există 푛 ∈ ℕ∗ încât: 1 1 1 20151 2sin sin sinn

unde [푥] ,reprezintă partea întreagă a numărului real 푥 . Prof. Daniel Sitaru

Page 59: RMM 14.pdf

S.S.M.ROMÂNIA - Filiala Mehedinți 2014

59 REVISTA MEHEDINȚEANĂ DE MATEMATICĂ NR. 14

XI.18. Fie 2 23

1 11( 1)

n

nk

xk k

. Să se calculeze: lim n

n

lnxn

Prof. Daniel Sitaru

XI.19. Să se calculeze: 21

1lim1

n

nk

k coskn k cosk

Prof. Daniel Sitaru XI.20. Fie 퐴,퐵 ∈ 푀 (ℂ) încât:

푑푒푡(퐴 + 2013퐵) = 푑푒푡(퐵 + 2013퐴) 푑푒푡(퐴 + 2014퐵) = 푑푒푡(퐵 + 2014퐴) 푑푒푡(퐴 + 2015퐵) = 푑푒푡(퐵 + 2015퐴)

Să se arate că: 푑푒푡퐴 = 푑푒푡퐵. Prof. Claudia Nănuți

XI.21. Fie numerele reale 푎,푏, 푐 > 0 푎푠푡푓푒푙 푖푛푐푎푡 푎 + 푏 + 푐 = 3.

Demonstrați inegalitatea: 2014 2014 2014

2013 2013 2013 2013 2013 2013

3 .2

a b cb c c a a b

Prof.Leonard Giugiuc XI.22. Să se arate că șirul 2)( nna ,dat de )1(log na nn este monoton și mărginit .Să se determine limita șirului.

Prof.Dr.Gheorghe Căiniceanu XI.23. Să se arate că dacă 푎,푏, 푐 ∈ (0,1) atunci:

a) log 푏 + log 푐+log 푎 ≥ 3

b) log 푏 log 푐 log 푎log 푎 log 푏 log 푐log 푐 log 푎 log 푏

≥ 9(log + log + log )

Prof. Vasile Grecu

XI.24. Calculați :3 3 3 3 3 3

1 1 1 1 1lim ...11 2 3n n nn n n

.

Prof.Dan Nedeianu XI.25. Determinați matricea 퐴 ∈ 푀 (ℤ) astfel încât 2

3A I A . Prof. Dan Nedeianu

Clasa a XII-a

XII.1. Fie 푓,푔:ℝ → ℝ ;푓,푔 continue pe ℝ și ,푏 ∈ ℝ . Știind că:

∀푥 ∈ ℝ ,푓(푥) = 푔(푎 − 푥) si 푓 (푥) + 푔 (푥) = 푏 să se calculeze:

퐼 =푏푓 (푥) + 푔(푥)푓(푥) + 푔(푥) + 푏 푑푥

Prof. Daniel Sitaru

XII.2. Fie 퐴,퐵 ∈ 푀 (ℤ); 푑푒푡퐵 ≠ 0 ; 퐵∗ = 퐵. Să se arate că 푑푒푡 퐴 + 퐵 ≠ 0 . Prof. Daniel Sitaru

XII.3. Fie (퐺 ,∙) un grup și 푥, 푦 ∈ 퐺 astfel încât 푥 = 푒 și 푦 = 푥푦푥 . Să se arate că: 푦 = (푥 ) 푦 푥 ; 푛 ∈ 푁∗.

Prof. Daniel Sitaru

Page 60: RMM 14.pdf

S.S.M.ROMÂNIA - Filiala Mehedinți 2014

60 REVISTA MEHEDINȚEANĂ DE MATEMATICĂ NR. 14

XII.4. Fie 푓: [−1,1] → ℝ,푓(푥) = 푎푟푐푠푖푛 푐표푠(푎푟푐푠푖푛푥) + 푎푟푐푐표푠(푠푖푛(푎푟푐푐표푠푥))

Să se calculeze: 퐼 = ∫ 푓(푥)푙푛(1 + 푡푔푥)푑푥 Prof. Daniel Sitaru

XII.5. Fie 퐺 = 퐴(푥) 퐴(푥) =푥 00푥

0 푥 푥 0

0 푥 푥 0

푥00푥

; 푥 ∈ ℝ .

Să se arate că (퐺 ,∙) este grup abelian și (퐺 ,∙) ≅ (ℝ , +) . Prof. Daniel Sitaru

XII.6. Fie 퐺 = 퐴(푥) 퐴(푥) =푥 00푥

0 푥 푥 0

0 푥 푥 0

푥00푥

; 푥 ∈ ℝ∗ .

Să se arate că (퐺 ,∙) este grup abelian și (퐺 ,∙) ≅ (ℝ∗ ,∙) . Prof. Daniel Sitaru

XII.7. Fie 퐺 = ℝ/ − ; 푥 ∗ 푦 = 푥 + 푦 + 2015푥푦 . Să se arate că (퐺 ,∗) este grup abelian și (ℝ∗,∙) ≅ (퐺,∗).

Prof. Claudia Nănuți

XII.8. Să se arate că: 3

12

( 1) 12

( 2)

arctg xI dxarctg

x

Prof. Daniel Sitaru

XII.9. Să se calculeze: lim →∫

Prof. Claudia Nănuți

XII.10. Fie 1

*

0

; 0 ; 0,1 ; n

nxI dx ax b x n

ax b

N

Să se calculeze: lim→

(푠푖푛 푛 ∙ 퐼 ) Prof. Claudia Nănuți

XII.11. Fie polinoamele 푃 (푥) = 1 ; 푃 (푥) = ∫ 푃 (푦) 푑푦 ; 푎 ∈ ℝ ;푛 ≥ 1 . Să se arate că polinomul: 푇 (푥) = ∑ 푃 (푥) nu poate avea rădăcini multiple.

Prof. Claudia Nănuți XII.12. Să se demonstreze că:

2015 푑푥 2015 푑푥 ≥ 1

Prof. Daniel Sitaru

XII.13. Să se rezolve în 푀 (ℂ)Χ푀 (ℂ) sistemul: 푋 + 푌 = 푂푋 + 푌 = 퐼

Prof. Daniel Sitaru

XII.14. Fie 퐻 un subgrup cu șase elemente al grupului (ℂ∗ ,∙) . Arătați că:퐻 = ±1 , ± ± 푖 √ Prof. Claudia Nănuți

Page 61: RMM 14.pdf

S.S.M.ROMÂNIA - Filiala Mehedinți 2014

61 REVISTA MEHEDINȚEANĂ DE MATEMATICĂ NR. 14

XII.15. Fie 푝 ∈ ℤ. Notăm 푝ℤ = {푝푞|푞 ∈ ℤ}. Fie 푝 ,푝 , … ,푝 ;푛 ≥ 2 numere prime distincte.

Să se arate că: 11

n n

i iii

p p

Z Z

Prof. Claudia Nănuți

XII.16. Să se calculeze: 9 51I x x dx Prof. Claudia Nănuți

XII.17. Fie 푎 > 0 și 푓: [−푎, 푎] → ℝ o funcție Rolle cu proprietatea că: 0

0

a

a

f x dx f x dx

Să se arate că ∃푏 ∈ (−푎,푎); 푓 (푏) = 0 Prof. Claudia Nănuți

XII.18. Fie 퐺 = 푀(푥) =2푥 + 1 0 6푥

0 0 0푥 0 3푥 + 1

푥 ∈ ℝ/ − .

Să se arate că (퐺 ,∙) este grup abelian și (ℝ∗,∙) ≅ (퐺 ,∙) . Prof. Daniel Sitaru

XII.19. Să se demonstreze că: , : ; 2 ( )Aut f f f f x y xy f x f y Z Z Z Prof. Daniel Sitaru

XII.20. Considerăm inelul unitar (퐴, +,∙).Știind că A are exact 7 elemente inversabile și mai puțin de 7elemente neinversabile ,demonstrați că A este corp.

Prof.Leonard Giugiuc (Prelucrare dupa Marcel Țena)

XII.21. Fie xexf sin)( , Rf ]2

,0[: . Să se arate că:

1) f este descrescătoare 2) 4

6

( )12 12

f x dxe e

.

Prof. Dr. Gheorghe Căiniceanu

XII.22. Să se arate că: ∫ (1 + x + x )(e + e + e )dx ≥ (e − 1) Prof.Vasile Grecu

XII.23. Să se calculeze: a) 2

1

1 2 910 10 10 10x x x x dx

b) 1

0

1 2 1910 10 10 10x x x x dx

Prof. Adela Grecu

XII.24. Să se calculeze: ∫ dx Prof. Adela Grecu

XII.25. Să se calculeze: ∫ ( ) dx Prof.Vasile Grecu

XII.26. Să se calculeze: 2

3

1 sincos

x xdx

x

.

Prof. Dan Nedeianu

Page 62: RMM 14.pdf

S.S.M.ROMÂNIA - Filiala Mehedinți 2014

62 REVISTA MEHEDINȚEANĂ DE MATEMATICĂ NR. 14

XII.27. În grupul (G,∙) elementele a,b G satisfac relațiile a5=e și ab=b2a. Determinați ordinul elementului b în grupul G (adică să se determine 푝 ∈ ℕ∗ mimin astfel încât bp=e).

Prof. Dan Nedeianu

Concursul naţional de matematică aplicată „Adolf Haimovici”-2014 Clasa a IX-a Servicii

1. Să se rezolve ecuaţia 1 8 15 ... 204x .

Prof.Claudia Nănuți

2. Să se demonstreze că 1 1 1 1 1 1 1 11 ... ... .2 3 4 2013 2014 1008 1009 2014

Prof.Daniel Sitaru

3. Fie n N , 2n . Să se rezolve în mulţimea R sistemul

1 1 2

32 1 2

33 1 2

31 2

... 1

... 2

... 3.....................................

...

n

n

n

n n

x x x x

x x x x

x x x x

x x x x n

.

Prof.Claudia Nănuți-RMM13- 2013 4. Fie ABCDEF un hexagon convex, iar M, N, P, Q respectiv punctele de intersecţie ale segmentelor ce unesc mijloacele laturilor opuse în patrulaterele ABCD, ABDE, ABEF, ABFC. Să se arate că patrulaterul MNPQ este paralelogram.

GM 11-2013

Clasa a X-a Profil Servicii 1. Să se rezolve ecuaţiile: a) 1 2 1x x . b) 3 31 2 1x x

Prof.Claudia Nănuți

2. Să se arate că dacă 2, , ,x y z e e , atunci 1 1 1 81lnln ln ln 8

xyzx y z

.

Prof.Daniel Sitaru-RMM13- 2013

3. Fie n N , 2n . Să se rezolve în mulţimea R sistemul:

1 2 3 4

2 1 3 4

3 1 2 4

1 2 3 1

1...22...33...4

.................................

...1

n

n

n

n n

x x x x x

x x x x x

x x x x x

nx x x x xn

Prof.Claudia Nănuți-RMM13- 2013

Page 63: RMM 14.pdf

S.S.M.ROMÂNIA - Filiala Mehedinți 2014

63 REVISTA MEHEDINȚEANĂ DE MATEMATICĂ NR. 14

4. Să se determine funcţia * *:f N N , ştiind că 222 2 2 ( ) 1

1 (1) 2 (2) ... ( )4

n f nf f n f n

,

*n N . GM 11-2013

Clasa a XI-a Profil Servicii

1. Să se rezolve ecuaţia

11 1 1 11

1 1 11 11 0

11 11 1 1

x x x

x x x

x x x

.

Prof.Claudia Nănuți-RMM13- 2013

2. Să se calculeze 20

sin sinlimx

ax bxx

,

0

ln 1 arctglim

ln 1 tgx

axbx

*,a b R .

Prof.Daniel Sitaru

3. Fie 2, ( )A B M C astfel încât ( ) 0Tr AB . Să se arate că 2014 2014AB BA . Prof.Daniel Sitaru-RMM-13- 2013

4. Să se determine asimptotele graficului funcţiei : , 1 1,f R , 2

( )1

xf xx

.

Prof.Claudia Nănuți Clasa a XII-a Profil Servicii

1. Fie : 0,nf R , 2n , 2 3( ) log log ... logn nf x x x x . Să se calculeze

1

1lim ( )e

nnf x dx

n .

Prof.Claudia Nănuți-RMM13- 2013 2. Să se calculeze: a) x x dx , 0,x .

b) sinsin cos

x dxx x , 0,

2x

.

Prof.Daniel Sitaru 3. Fie 1 0, 1G . Definim pe 1G legea de compoziţie *x y x y , unde x este partea

fracţionară a lui x. Fie 2 cos sin | 0, 2G t i t t . Să se arate că 1, *G şi 2,G sunt grupuri abeliene izomorfe.

Prof.Daniel Sitaru-RMM13- 2013 4. Definim pe R legea de compoziţie * 2 2 6x y xy x y . Să se rezolve ecuaţiile: a) *x x x . b) * *x x x x . c) * * *x x x x x .

Prof.Claudia Nănuți

Page 64: RMM 14.pdf

S.S.M.ROMÂNIA - Filiala Mehedinți 2014

64 REVISTA MEHEDINȚEANĂ DE MATEMATICĂ NR. 14

Clasa a IX-a Profil Ştiinţe ale naturii

1. Să se determine 0,x dacă 2 2

1 4 2015... 12014 2014 2

x x x x .

2. Se dă 1 2 ...2! 3! ( 1)!n

nSn

, unde ! 1 2 3 ...n n . Să se arate că 11( 1)!nSn

,

*n N . 3. Fie triunghiul ABC şi punctele M şi N pe laturile AB şi AC, astfel încât 5AB MA

, ||MN BC .

Să se determine x real, ştiind că 3 7CN x AN

.

4. Fie ABCD un patrulater ortodiagonal, M mijlocul laturii AB , N mijlocul laturii BC , P

mijlocul laturii CD şi Q mijlocul laturii AD . Dacă 8AC cm , 6BD cm , iar

MP NQ O , aflaţi modulul vectorului 2014v MN MO MQ

.

Clasa a X-a Profil Ştiinţe ale naturii

1. Se consideră expresia 3

11 2 32 4

3 2 3( , ) x yxE x y x y

x x y

. Calculaţi (2, 16)E .

2. a) Arătaţi că expresia 2 4 8 16

3 9 27 81

log log log loglog log log log

x x x xx x x x

este constantă pentru orice

0, 1x . b) Dacă 12log 3a şi 12log 5b , calculaţi 15log 20 în funcţie de a şi b.

3. a) Fie z C astfel încât 2 1 1zz

. Calculaţi 12z .

b) Fie 1 2,z z C , 1 2z z . Arătaţi că dacă 1 2z z , atunci *1 2

1 2

z z iRz z

.

4. Arătaţi că dacă 3 3 3 27b c c a a b

a b c

cu , , 0,a b c , atunci a b c .

Clasa a XI-a Profil Ştiinţe ale naturii

1.a) Fie 2( )X M R . Să se demonstreze că 22 2( ) det( )X Tr X X X I O , unde ( )Tr X este

suma elementelor de pe diagonala principală. b) Fie 2( )Y M R astfel încât 2Y t Y , *t R . Să se arate că 1n nY t Y , *n N .

c) Să se determine 2( )X M R astfel încât 2014 2014 20132014 2013

X

.

2. Fie , nA B M R două matrice distincte astfel încât 2B B A . Este matricea 2A B B inversabilă?

Page 65: RMM 14.pdf

S.S.M.ROMÂNIA - Filiala Mehedinți 2014

65 REVISTA MEHEDINȚEANĂ DE MATEMATICĂ NR. 14

3. Calculaţi: a) 0

sin 2014 arcsinlimtg2015 arctg2015x

x xx x

. b)

2014 52014 1lim2014 3

x

x

xx

.

4. Fie funcţia :f R R cu proprietatea că 3 2( ) 2014f x x x , x R . a) Să se arate că (0) 0f . b) Să se arate că

0lim ( ) 0x

f x

.

Clasa a XII-a Profil Ştiinţe ale naturii

1. Se consideră matricea 2 21 1

A

şi mulţimea 2( ) | 1G X a I aA a R .

a) Să se demonstreze că ( ) ( ) ( )X a X b X ab a b , , 1a b R . b) Să se demonstreze că mulţimea G formează grup abelian în raport cu înmulţirea matricelor. c) Să se calculeze (1) (2) ... (2014)X X X .

2. Pe mulţimea 0, 1, 2, 3, 4H definim legea de compoziţie , 2,, 3 şi 2

x y x yx y x y x y

y x x y

.

Precizaţi proprietăţile acestei legi de compoziţie.

3. Se consideră funcţia : 1,f R , ln( ) xf xx

.

a) Să se calculeze ( )f x dx . b) Să se calculeze 3 2 ( )x f x dx . 4. Fie funcţia :f R R , ( ) 1 sinf x x . a) Să se arate că orice primitivă a funcţiei f este crescătoare pe R. b) Să se calculeze ( ) ( )f x f x dx .

O l i m p i a d a d e M a t e m a t i c ă - fazalocală2014-

Clasa a-V-a

Subiectul I. Determinaţi 1.Se consideră mulţimile 100...,,3,2,1A şi 5 3 |B n n N . a) Să se determine numărul elementelor mulţimii A B . b) Să se calculeze suma elementelor mulţimii A B . c) Câte elemente din mulţimea A B sunt pătrate perfecte?

Prof.Dana Paponiu 2.Un elev a rezolvat 28 de probleme în cinci zile. În ziua a cincea a rezolvat de şase ori mai multe probleme faţă de prima zi. Să se afle câte probleme a rezolvat elevul în ziua a patra, ştiind că în fiecare zi a rezolvat un număr de probleme cel puţin egal cu cel din ziua precedentă.

Prof.Dan Nedeianu – RMM- 2013

Page 66: RMM 14.pdf

S.S.M.ROMÂNIA - Filiala Mehedinți 2014

66 REVISTA MEHEDINȚEANĂ DE MATEMATICĂ NR. 14

3.Se împarte la 11 numărul 17747 1 nn , unde n este un număr natural. a) Să se determine câtul şi restul împărţirii. b) Să se determine ultimele două cifre ale câtului dacă n este un număr natural multiplu de 4.

Prof.Dana Paponiu 4. Se consideră numerele naturale n, ab şi cd care satisfac relaţia 2 2 4ab cd n . a) Arătaţi că ab cd şi ab este un număr impar. b) Să se determine valorile posibile ale lui n şi să se precizeze câte dintre acestea satisfac, în plus, condiţia 12 n este divizibil cu 5.

Prof.Dana Paponiu - GM 11-2013

Clasa a VI-a 1. a) Aflaţi numărul abc ştiind că 1332abc bca cab şi a, b, c sunt numere prime diferite două câte două. Prof. Angela Niţoiu –RMM- 2013 b) Arătaţi că numărul 3 2 3 16 6 1n nA se divide cu 43, n N .

GM-6-7-8-2013 2. Să se determine *,m n N astfel încât 20141 1 2 1 2 3 ... 1 2 3 ... n m .

Prof.Daniel Sitaru 3. Pe o dreaptă d se consideră punctele 0A , 1A , 2A , …, 2014A astfel încât 1A este mijlocul

segmentului 0 2A A , 2A este mijlocul segmentului 0 3A A , 3A este mijlocul segmentului

0 4A A , ..., 2013A este mijlocul segmentului 0 2014A A . Ştiind că lungimea segmentului

2013 2014A A este 20122 cm, aflaţi lungimile segmentelor 0 1A A , 0 2014A A şi verificaţi dacă

lungimea segmentului 2010 2014A A este divizibilă cu 15. ***

4. Avem la dispoziţie un raportor care are o singură gradaţie la 19 . Să se arate că utilizând acest raportor putem construi orice unghi având măsura 1 , 2 , 3 , ..., 359 . De câte utilizări ale raportorului este nevoie pentru a desena un unghi de 60?

Prof.Daniel Sitaru – GM- 11-2012 Clasa a VII-a

1. a) Să se arate că 2014 2015 N . b) Determinaţi un număr raţional p Q astfel încât 2014 2015 p Q .

Prof.Emilia Răducan

2. Să se rezolve în Z ecuaţia

Prof.Elena Râmniceanu –RMM- 2013 3. În triunghiul dreptunghic ABC, cu ˆ( ) 90m A , bisectoarea unghiului A intersectează pe BC în M, iar N este mijlocul lui AB.

a) Demonstraţi că dacă 2

ABMN , atunci triunghiul ABC este isoscel.

b) În condiţiile de la punctul a), arătaţi că 14MNB ABCA A .

GM 9-2013

1 1 1 2014... .( 1) ( 1)( 2) ( 2013)( 2014) 2015x x x x x x

Page 67: RMM 14.pdf

S.S.M.ROMÂNIA - Filiala Mehedinți 2014

67 REVISTA MEHEDINȚEANĂ DE MATEMATICĂ NR. 14

4. Se dă pătratul ABCD şi fie E şi F mijloacele laturilor [ ]AB , respectiv [ ]AB . Să se arate că: a) CE DF . b) Dacă P este simetricul lui E faţă de A, arătaţi că CDPE este paralelogram. c) Dacă CE DF M , arătaţi că AD AM .

*** Clasa a VIII-a

1.Fie 1 1 1...3 2 1 2 5 2 2 3 2 1 2 ( 1)

nSn n n

, *n N . Să se calculeze

20141 S . Prof.Daniel Sitaru –GM12-2013

2. Fie *2A n n N şi *2 2B n n N , unde x reprezintă partea întreagă

a lui x. Calculaţi A B . Prof.Daniel Sitaru -GM9-2013

3. Aflaţi aria totală a unei piramide patrulatere regulate VABCD în care muchia laterală VA are lungimea de 12 cm şi măsura unghiului VAC este de 30 .

Prof.Daniel Sitaru 4. Pe planul triunghiului OBC se ridică în O, o perpendiculară pe care se ia un punct A. Fie M şi M1 ortocentrele triunghiurilor ABC, respectiv OBC, AD şi BE înălţimi în triunghiul ABC şi BE1 înălţime în triunghiul OBC. a) Să se arate că 1 ( )MM ABC .

b) Dacă 5OB cm , 4 2OC cm , 7BC cm şi 2 21OA cm , să se calculeze AD. Prof.Emilia Răducan

Clasa a IX-a -M1 1.a) Se consideră mulţimea | 2014M x N x şi mulţimile 2 1 |A a a N M ,

3 1 |B b b N M şi 5 1 |C c c N M . Să se calculeze cardA , cardB şi cardC . Prof.Emilia Răducan

b) Fie a, b, c şi d numere reale pozitive astfel încât 1a b c d . Să se arate că 2 2 2 2 1

16a b c d .

Prof.Daniel Sitaru 2. Să se rezolve ecuaţia 2 4 3 0x x , unde x reprezintă partea întreagă a numărului real x.

Prof. Daniel Sitaru 3.Fie ABC un triunghi cu AB c , BC a , AC b . Medianele AM, BN şi CP taie cercul circumscris triunghiului în punctele D, E respectiv F. Ştiind că 0MD NE PF

, să se arate că

2 2 2

2 2 2 2 2 2 2 2 22 2 2 2 2 2a b cAM BN CP

b c a a c b a b c

0

.

GM 12-2013 4. Se dă ABC , [ ]M AB , [ ]N AC şi P BC , astfel încât [ ]C BP . Dacă , , R astfel încât AM MB

, BP CP

, CN NA

, să se arate vectorial că dacă M, N, P coliniare, atunci 1 .

Prof.Doru Preşneanu- RMM13- 2013

Page 68: RMM 14.pdf

S.S.M.ROMÂNIA - Filiala Mehedinți 2014

68 REVISTA MEHEDINȚEANĂ DE MATEMATICĂ NR. 14

Clasa a X-a M1

1. a) Determinaţi a N astfel încât 3

3 3 33

1 2 49 9 9 2 1

a

.

b) Să se arate că 3 33 3 31 2 4 2 19 9 9 .

Prof.Carmen Chirfot

2. a) Determinaţi a R , ştiind că 22( 1)( 2)( 3)( 4) 5 1t t t t t t a , pentru orice t R .

b) Să se rezolve ecuaţia 10 1 10 2 10 3 10 4 1 0x x x x . Prof.Claudia Nănuţi -RMM13- 2013

3. Să se rezolve ecuaţia 5 31 log log 4 5x x . GM12-2013

4. Considerăm numărul complex cos10 sin10z i .

a) Să se arate că 14

7

1 72cos18

zz

.

b) Să se calculeze suma 2 4 6

2 3

2014 1 2014 1 2014 1...

z z zz z z

36

18

2014 1zz

, unde [ ]t reprezintă partea întreagă a numărului real t.

Prof.Dan Nedeianu Clasa a XI-a M1

1.Fie matricile 3, , ( )A B C M R şi x cifră nenulă, 100 10 110 1 1001 100 10

A

, 1 1

1 11 1

xB x

x

,

11 1 1 11

1 1 11 11

11 11 1 1

x x x

C x x x

x x x

.

a) Să se arate că A B C . b) Să se rezolve ecuaţia det( ) 0C .

Prof.Claudia Nănuţi -RMM13- 2013 2. Fie a R şi n N , 2n .

a) Să se arate că 1 22

a a a .

b) Să se arate că şirul 1n nx

,

1

1 n

nk

kx an n

este convergent. ([ ]a reprezintă partea întreagă a

lui a). Prof.Dr.Gheorghe Căiniceanu

Page 69: RMM 14.pdf

S.S.M.ROMÂNIA - Filiala Mehedinți 2014

69 REVISTA MEHEDINȚEANĂ DE MATEMATICĂ NR. 14

3. Fie 2( )A M R şi propoziţiile p: ( ) 1Tr A ( ( )Tr A este suma elementelor de pe diagonala principală a matricei A) şi q: det( ) 2A , r: 2det 4A I . Să se arate că dacă două dintre propoziţii sunt adevărate, este adevărată şi a treia.

Prof.Dr.Daniel Stretcu

4. Fie ,4 2

. Considerăm şirul 1n na

definit prin 1a tg şi 2 2 2

1 cos sinn na a ,

1n . Să se calculeze lim 1nnn a

.

GM 11- 2013 Clasa a XII-a M1

1. Pe mulţimea R definim legea de compoziţie 20152015 2015*x y x y . Să se arate că , *R este

grup abelian şi că , * ,R R . Prof.Claudia Nănuţi

2. Calculaţi 2sin 3cos3sin 2cos

x x dxx x , 0,

2x

fără a utiliza o schimbare de variabilă.

GM11-2013

3. Fie 1 2 20120, , , ...,2013 2013 2013

G

. Definim pe G legea de compoziţie *x y x y , unde

x este partea fracţionară a lui x. Să se arate că , *G este grup abelian şi că 2013, * ,G Z . Prof.Daniel Sitaru -RMM 13-2013

4. Fie n N , 3n şi 3 1

12

n

nk

kI x dxn

.

a) Să se calculeze 3I .

b) Să se arate că 5 6 5 42 2n

n nI , 3n .

Prof. Dr.Gheorghe Căiniceanu

Olimpiada de matematică- etapa judeţeană, 8 martie 2014

Clasa a IX-a Clasa a X-a Lungu Vlad C.N. “Traian” I Popescu Cristiana C.N. “Traian” I

Chivără Diana C.N. “Traian” II Burtea Catalin C.N. “Traian” II Peleașă Teodora C.N. “Traian” II Ghilicică Vlad C.N. “Traian” III

Radu David C.N. “Traian” III Florea Bianca C.N. “Traian” M Calagiu Andreea C.N. “Traian” M Badea Simona C.N. “Titeica” M

Munteanu Alexandru C.N. “Traian” M Opranescu Veronica C.N. “Titeica” M Fleancu Ingrid C.N. “Traian” M Meterez Alexandru C.N. “Titeica” M Zanfir Mihaela C.N. “Traian” M Socolescu Andrei C.N. “Traian” M

Chesoi Andi C.N. “Traian” M

Page 70: RMM 14.pdf

S.S.M.ROMÂNIA - Filiala Mehedinți 2014

70 REVISTA MEHEDINȚEANĂ DE MATEMATICĂ NR. 14

Clasa a XI-a Paleacu Cosmin C.N. “Traian” M Neciu Denisa C.N. “Titeica” I Rusu Ștefania C.N. “Titeica” M

Glăvan Steliana C.N. “Titeica” II

Clasa a XII-a Andreescu Roxana C.N. “Traian” III

Nicolicioiu Armand C.N. “Titeica” III Măgăreață Georgiana C.N. “Titeica” I Dîrpes Iulian C.N. “Titeica” M Mema Serban C.N. “Traian” II

Bănica Teodor C.N. “Traian” M Pîrvulescu Loredana C.N. “Titeica” III Iloaia–Pîrvulescu Bianca C.N. “Traian” M Stoican Theodor C.N. “Traian” M

Bărzuică Alexandru C.N. “Titeica” M Dobrescu Maria C.N. “Titeica” M Buligă Alexandru C.T.” Dl.Tudor” M Mareși Cristian C.”M.Basarab” M

Pleșa Bianca C.N.”T.Lalescu” M

Clasa a V-a Clasa a VI-a Șoșea Miruna C.N. “Traian” I Bușe Iasmina C.N. “Traian” I Vasile Marian C.N. “Traian” II Crăciunescu Ion E. Șc.”P.Dumitriu” II

Semen Valentin Ion C.N. “Traian” III Jianu Diana C.N. “Titeica” III Tătucu Miruna Maria C.N. “Traian” III Grecu Bogdan C.N. “Traian” M Dumitru Alexandra C.N. “Traian” M Gardner Benjamin C.N. “Traian” M

Țuțurigă Cristina C.N. “Traian” M Pâlșan Bianca Șc.”A.Voinescu” M Apetri Anastasia C.N. “Traian” M Tudor Bogdan C.N. “Traian” M

Ispas Andrei C.N. “Traian” M Ceaușene Patricia C.N. “Traian” M Vlăducu Marius Șc.”A.Voinescu” M Curea Minodora Șc.”P.Sergescu” M Parvan Andreea Șc.”P.Dumitriu” M Petolea Andreea Șc.”P.Sergescu” M

Raicu Iulia C.N. “Traian” M Semen Andrei Șc.”P.Șeicaru” M Paraschiv Ivona C.N. “Traian” M Tîrîși Claudiu L. Șc.Gen.Nr.14 M Purcaru Carol C.N. “Titeica” M Chiriță Medeea C.N. “Traian” M Sîrbu Maria C.N. “Titeica” M Guran Sarah C.N. “Traian” M

Alicsandroniu Dragos C.N.”Odobleja” M Munteanu Cătălina Șc.Vînju Mare M Lunguleasa Eugen C.N. “Traian” M Sărăcin Răzvan C.N. “Titeica” M Popa Alexandra Șc.”P.Sergescu” M Bușe Iasmina C.N. “Traian” M

Cismaru Ioana Maria C.N. “Traian” M Copilu Ștefan C.N. “Titeica” M

Dorobanțu Andrei C.N. “Traian” M Filip Vladimir C.N.”Odobleja” M

Răchitan Maria C.N. “Traian” M

Clasa a VIII-a Clasa a VII-a Marghescu Bogdan C.N. “Traian” I Șeitan Radu C.N. “Traian” I

Giurescu Malina C.N. “Traian” II Paicu Madalina Șc.”P.Sergescu” II Iloaia Pirvulescu C.N. “Traian” II Fulga Fabian C.N. “Traian” III Antonescu Denis C.N. “Traian” III Gîndacu Elvis C.N. “Traian” M

Bobîrsc Laura Șc.”P.Sergescu” III Rogobete Ana C.N. “Traian” M Burdescu Alexandru C.N. “Traian” M Dumitrache Francesca C.N. “Traian” M

Mareș Andrei C.N. “Traian” M Popescu Răzvan C.N. “Traian” M

Page 71: RMM 14.pdf

S.S.M.ROMÂNIA - Filiala Mehedinți 2014

71 REVISTA MEHEDINȚEANĂ DE MATEMATICĂ NR. 14

Nețoiu Andrei C.N. “Traian” M Croitoru Robert C.N. “Traian” M Mazilescu Andru C.N. “Traian” M Drugă Ana Maria C.N. “Titeica” M Iacobescu Florea Șc.Gen.Nr.6 M Bucă Adina Șc.”A.Voinescu” M Sbîrcea Anidora Lic.”V.Gomoiu” M Ianculovici Isabela C.N. “Traian” M

Andrei Eliza C.N. “Traian” M Lăcătușu Roxana C.N. “Traian” M Schverin Christian Șc.”P.Șeicaru” M Voinicu Anastasia C.N. “Traian” M

Sbîrcea Vlad C.N. “Titeica” M Vuc Daria Șc.”P.Șeicaru” M Socolescu Cristian C.N. “Traian” M

Moraru Andrei C.N. “Traian” M Rovența Robert C.N. “Traian” M

Concursul Național de Matematică aplicată ”Adolf Haimovici” Etapa Județeană-8.03.2014

OLIMPIADA NAȚIONALĂ DE MATEMATICĂ-2013 – 2014-Brașov

Lungu Vlad X C.N. “Traian” ARGINT Prof.Dr.Căiniceanu Gheorghe

Crăciunescu Emanuel VI Șc.Orsova ARGINT Prof. Vasilcanu Teodor

Marghescu Bogdan VIII C.N. “Traian” BRONZ Prof. Dr.Prajea Manuela

Clasa a IX-a Clasa a X-a Lupu Aurel C.N.”Odobleja” M Butaru Daniel C.N.”Odobleja” I

Năstăsie Sergiu C.N.”Odobleja” M Cîrneci Natalia C.N.”Odobleja” M Costăchioiu Andreea C.N. “Traian” M

Lupșescu Valeriu C.N.”T.Lalescu” M

Clasa a XI-a Clasa a XII-a Strhăianu Raluca C.N.Economic II Trăilescu Liliana Nela C.N.Economic I

Budencea Iozefina C.N.Economic III Brebu Agnana C.N.Economic II Trinca Irina Mădălina C.N.Economic III Pena Lavinia C.N.Economic II

Ioancea Alexandra C.N.Economic M Țuțunel Oana Roxana C.N.Economic III Scarlat Mariana C.N.Economic M Răescu Adelina C.N.”Odobleja” I

Lunguleasa Ionela C.N.”Odobleja” III Cismaru Mădălina C.N.”Odobleja” II Bușoi Gabriela C.N. “Titeica” M Pîlsu Georgiana C.N.”Odobleja” II

Dorobanțu Adina C.N.”Dierna” I Popa Mihaela C.N.”Odobleja” III Mladinovici Ionela C.N.”Odobleja” II Nărămzoiu Claudia Lic.Cujmir III

Page 72: RMM 14.pdf

S.S.M.ROMÂNIA - Filiala Mehedinți 2014

72 REVISTA MEHEDINȚEANĂ DE MATEMATICĂ NR. 14

CONCURSUL “Lumina Math”-2013-2014

Antonescu Denis VIII C.N.

“Traian” M

Marghescu Bogdan VIII C.N. “Traian” I

Bolocan Monica Alexandra

VII C.N. “Titeica”

M

Mazilescu Andru VIII C.N. “Traian”

III

Bușe Iasmina VI C.N.

“Traian” II

Ofițeru Cristian VII C.N. “Titeica” M

Ceaușene Patricia VI C.N.

“Traian” M

Picior Cătălin VII C.N. “Traian” III

Croitoru Robert VIII C.N.

“Traian” M

Pîlșan Bianca VI Șc.”A.Voinescu” III

Dănoiu Constantin Eugen

VI C.N. “Traian”

M

Popa Delia VII C.N.”Odobleja”

M

Dumitrache Francesca VII C.N.

“Traian” III

Șeitan Radu VII C.N. “Traian” M

Florea Andrei Bogdan VII C.N.

“Titeica” III

Semen Valentin Ion V C.N. “Traian” III

Fulga Fabian VII C.N.

“Traian” M

Șoșea Miruna V C.N. “Traian” M

Giurescu Mălina VIII C.N.

“Traian” III

Ștefan Bogdan VII C.N. “Traian” M

Grecu Bogdan VI C.N.

“Traian” II

Stretcu Cătălin VII C.N. “Titeica” M

Ilioaia-Pîrvulescu Andreea

VIII C.N. “Traian”

II

Tudor Bogdan VI C.N. “Traian”

M

Iordăchescu Anca VIII Sc.Nr. 5 M

Vasile Marian V C.N. “Traian” M

Jianu Diana VI C.N.

“Titeica” M

Vițian Denis VI Șc.”A.Voinescu” M

Mareș Călin VIII C.N.

“Traian” II

Vladu Cristina VII Șc.”A.Voinescu” M

CONCURSUL “CANGURUL”- 2014

Popescu Cristiana C.N. “Traian”

Diplome de onoare la Barajul Național Radu David Alexandru C.N. “Traian” Grecu Bogdan C.N. “Traian”

CONCURSUL “VIITORI OLIMPICI”- 2014

Bușe Iasmina C.N. “Traian” VI I – AUR Prof.Dr.Căiniceanu Gheorghe Grecu Bogdan C.N. “Traian” VI III Prof.Dr.Căiniceanu Gheorghe Puiu Mihaela Șc.Gen.3 IV III Prof. Florica Vasile Puiu Maria Șc.Gen.3 IV III Prof. Florica Vasile Șeitan Radu C.N. “Traian” VII M Prof. Paponiu Dana

Vasile Marian C.N. “Traian” V M Prof. Dr.Prajea Manuela

Page 73: RMM 14.pdf

S.S.M.ROMÂNIA - Filiala Mehedinți 2014

73 REVISTA MEHEDINȚEANĂ DE MATEMATICĂ NR. 14

OLIMPIADA NAȚIONALĂ DE LINGVISTICĂ

Bușe Iasmina C.N. “Traian” VI III Prof.Dr.Căiniceanu Gheorghe Grecu Octavian C.N. “Traian” VI M Prof.Dr.Căiniceanu Gheorghe

Raicu Iulia C.N. “Traian” V M Prof. Dr.Prajea Manuela

CONCURSUL “MICUL ARHIMEDE” –Craiova- 2013

Dumitrache Francesca C.N. “Traian”

VII M Prof. Paponiu Dana

CONCURSUL “PURPLE COMET”- UNIVERSITATEA WISCONSIN- 2014

Sc.gen.5,Coordonator Prof. Piț-Rada Marica Echipa VIP formată din elevii: Iordăchescu Anca, Neagoe Ramona, Stoianovici Bianca, Savu Teodora, Paulescu Andrei "Honorable Mention - Middle School, 1-400" CNT,Coordonator Prof. Paponiu Dana Echipa: Dumitrache Francesca, Fulga Fabian, Gândacu Elvis, Picior Cătălin, Tabacu Teculescu Andra, Popescu Răzvan „Honorable Mention”, Midle School, 400-1000